Download Controlled Substance Management

Document related concepts

Compounding wikipedia , lookup

Drug design wikipedia , lookup

Neuropsychopharmacology wikipedia , lookup

Psychedelic therapy wikipedia , lookup

Stimulant wikipedia , lookup

Drug discovery wikipedia , lookup

Medical prescription wikipedia , lookup

Adherence (medicine) wikipedia , lookup

Pharmacognosy wikipedia , lookup

Bad Pharma wikipedia , lookup

Pharmaceutical industry wikipedia , lookup

Pharmacokinetics wikipedia , lookup

Medication wikipedia , lookup

Neuropharmacology wikipedia , lookup

Drug interaction wikipedia , lookup

Prescription costs wikipedia , lookup

Prescription drug prices in the United States wikipedia , lookup

Psychopharmacology wikipedia , lookup

Electronic prescribing wikipedia , lookup

Pharmacogenomics wikipedia , lookup

Polysubstance dependence wikipedia , lookup

Transcript
ASIPP’s Comprehensive Pain Medicine Board Review Course-2007
SECTION 12
Controlled Substance
Management
Directions: Choose the best answer
2310. The common belief that most people who misuse, divert,
or assign illicit use to controlled substances are felt to be
unemployed and from the inner city. The following is
true of demographics according to SAMHSA:
A. 75% of illicit drug users were employed full or part
time
B. The highest use of illicit controlled substances were
among American Indians and Alaskan natives.
C. Blacks and whites were approximately equal in the use
and misuse of controlled substances.
D. Inner city blacks were of the highest percentage of illicit
drug users.
E. 60% of illicit drug users were on Medicaid
2311. All of the following describe psychological components
of pain, EXCEPT:
A. Catastrophic thinking
B. Helplessness
C. Compulsive search for a job
D. Blame
E. Chronic maladaptive coping patterns
2312. Which one of the following is characteristic of
cytochrome P-450?
A. Located in the lipophilic environment of mitochondrial
membranes
B. Catalyzes O-, S-, and N-methylation reactions
C. Catalyzes aromatic and aliphatic hydroxylations
D. Catalyzes conjugation reactions
E. Activity is not inducible by drugs
2313. You have agreed to see a new patient who comes to
you through a referral from a family physician in a city
approximately 50 miles from your practice. The patient
gives you a referral package that does not contain any
reference to the patient’s recent history with the other
physician. The referral package also does not contain
any reference to the patient’s history of compliance with
the family physician’s treatment plan or instructions
regarding the patient’s use of controlled drugs. The
patient asks you to prescribe controlled substances to
treat his/her pain and reports that he/she is using a high
dosage of a specified Schedule II controlled substance.
The patient claims he/she has run out of medications
and will experience acute withdrawal symptoms if he/she
does not get the medications from you. What should
you do prior to minimize the potential for abuse and
diversion of these drugs by this patient?
A. Prescribe the patient a months’ worth of drugs and see
him/her back in your office after you obtain the medical records from the previous physician.
B. Call the previous physician and attempt to verify the
patient’s self-report of recent pain treatments and
medications, use an appropriate urine screen to verify
the presence of the drug the patient says he/she has
taken and to determine whether he/she is
C. Attempt to verify the patient’s self-report of recent pain
treatments and medications and perform an appropriate urine screen.
D. Accept the patient’s self-report and continue with treatment without verification through the prior provider.
E. Send the patient back to the previous provider for necessary controlled substances to treat acute withdrawal
symptoms and tell him/her that you will not prescribe
controlled substances until you receive all the necessary records from the prior provider.
466
Section 12 • Controlled Substance Management
2314. What document reflects the practitioner’s explanation 2320. Gabapentin (Neurontin®) exerts its analgesic affect by:
A. Inhibition of reuptake Serotonin and Norepinephrine
and the patient’s understanding of the risks, benefits,
B. Central modulation of the dorsal lateral funiculus at the
alternative treatments, and special issues concerning the
dorsal horn intermediary.
use of controlled substances?
C. GABA-A affinity and activation.
A. Narcotic contract
D. The analgesic effect is unknown.
B. History and Physical Evaluation form
E. NMDA modulation.
C. Pain scale evaluation and update report
D. Informed consent form
E. The approval letter from the patient’s health care benefit 2321. Drug interactions common to cyclobenzaprine
(Flexeril®) include all of the following except :
plan
A. MAOI agents
B. Barbiturates
2315. If a patient brings unused controlled substances back to
C. Tertiary tricyclic antidepressants
you at your office, you should do which of the following?
D. Zolpidem (Ambien®)
A. Dispose of the controlled substances after the patient
E. Alcohol
leaves your office and write down what you did in the
medical record.
B. Inventory the returned controlled substances and use
them with other patients who cannot afford to pay for 2322. Among the following neuroleptics, the agent most likely
associated with constipation, urinary retention, blurred
prescriptions for these drugs because they do not have
vision, and dry mouth is:
health insurance.
A. Chlorpromazine (Thorazine®)
C. It depends on regulations of State Board of Medical
B. Clozapine (Clozaril®)
Licensure
C. Olanzapine (Zyprexa®)
D. Flush the stuff down the toilet.
D. Sertindole (Serdolect®)
E. Call a DEA agent to come and get the drugs.
E. Haloperidol (Haldol®)
2316. Cocaine intoxication has become a common problem in
hospital emergency rooms. Which one of the following 2323. Fluoxetine (Prozac®) is classified as:
A. As an MAO inhibitor (MAOI)
drugs is not likely to be of any value in the management
B. As a tricyclic nonselective amine reuptake inhibitor
of cocaine overdose?
C. As a heterocyclic nonselective amine reuptake inhibiA. Dantrolene
tor
B. Diazepam
D. As a selective serotonin reuptake inhibitor
C. Lidocaine
E. As an alpha2-adrenergic receptor inhibitor
D. Naltrexone
E. Nitroprusside
2324. Among the following neuroleptics, the most likely
neuroleptic associated with skeletal muscle rigidity,
tremor at rest, flat facies, uncontrollable restlessness, and
spastic torticollis is
A. Clozapine
B. Haloperidol
C. Olanzapine
D. Sertindole
E. Ziprasidone
2318. N-acetyl benzoquinoneimine is the hepatotoxic
metabolite of which drug?
A. Sulindac
2325. The W.H.O. cancer pain relief ladder is:
B. Acetaminophen
A. Poorly validated with anecdotal reports of effectiveC. Isoniazid
ness.
D. Indomethacin
B. Emphasizes non-opioid treatment through the first
E. Procainamide
three steps.
C. Is inappropriate to utilize on a terminally ill patient.
2319. A 36-year-old male with a bipolar disorder is treated
D. Is a simple and effective tool to afford relief with a high
with lithium. Among the following adverse effects, the
level of confidence, in an overwhelming majority of
side effect attributed to lithium treatment is:
patients.
A. Browning of the vision
E. Suggests alternative therapy management in the early
B. Hypothyroidism
stages.
C. Agranulocytosis
D. Neuroleptic malignant syndrome
E. Pseudodepression
2317. All of the following are chronic pain related psychiatric
disorders, EXCEPT:
A. Anxiety
B. Depression
C. Sleep disorders
D. Post traumatic stress disorder
E. Increased sexual function
ASIPP
Section 12 • Controlled Substance Management
467
2326. The concept of substance dependency is categorized as a
B. Have anticonvulsant activity
risk relationship. The term pseudotolerance is:
C. May induce physical dependence
A. The need to increase dosage that is not due to tolerance,
D. Have a higher margin of safety than barbiturates
but to other factors such as disease progression, new
E. All of the above
disease, increased physical activity, lack of compliance.
2333. Axis IV provides important information about which of
B. An expected course in use of controlled substances.
the following?
C. The first sign of addiction.
A. Ability to pay for all necessary medical services.
D. An indication to the physician that dosing needs to be
B. Overall satisfaction with scope of ongoing medical
increased.
services.
E. A change of medication is necessary to ensure funcC. Desire to serve as a subject in an upcoming clinical trial
tional enhancement.
for a new medication.
D. Impact of psychosocial and environmental problems
2327. When selecting and dosing opioids:
that may impact the patient.
A. There is no ceiling dose for combination analgesics
E. Personality disorders
B. Use long acting opioids for as-needed pain
C. Use short acting opioids for around-the-clock pain
2334. Which of the following opioids is least likely to cause
D. There is no ceiling dose for pure agonists
bradycardia in high doses?
E. Agonist-antagonist opioids are appropriate breakA. Fentanyl®
through medications
B. Meperidine®
C. Morphine®
2328. Which of the following factors will determine the
D. Hydromorphone®
number of drug-receptor complexes formed?
E. Oxycodone®
A. Efficacy of the drug
B. Receptor affinity for the drug
2335. What are the correct statements about drug abuse in
C. Therapeutic index of the drug
patients receiving opioids?
D. Half-life of the drug
A. Illicit drug use and abuse of prescription controlled
E. Rate of renal secretion
substances is non-existent in patients receiving longacting opioids in appropriate doses.
2329. Which of the following opioids is not a good choice in
B. Short-acting opioids show significantly higher use of ilpatients with renal failure:
licit drugs and abuse of controlled prescription drugs.
A. Fentanyl®
C. Illicit drug use and controlled substance abuse in
B. Sufentanil®
chronic non-cancer patients is similar whether they
C. Morphine®
are on short-acting or long-acting opioids.
D. Alfentanil®
D. Patients on long-acting opioids are highly compliant
E. Meperidine®
and functional and therefore not necessary to monitor them.
2330. The Drug Abuse Warning Network (DAWN) is a network
E. Patients on short-acting opioids significantly abuse
that:
drugs. Thus, they should be monitored every month
A. Treats addicted patients
with urine drug testing.
B. Is a self help and support group
C. Collects information on hospital emergency depart- 2336. Choose the correct statement with regards to the
ment admissions for drug-related episodes
comparison of effectiveness and safety profiles of longD. Is an arm of the Drug Enforcement Agency
acting versus short-acting opioids in chronic non-cancer
E. Monitors physicians’ prescription habits
pain.
A. There was conclusive evidence from high quality ran2331. Which of the following statements concerning lithium
domized and systematic trials to determine that longis true?
acting opioids as a class were more effective and safer
A. Lithium is used to control agitation associated with
with no adverse events than short-acting opioids.
schizophrenia
B. There was no good-quality data available to assess comB. Retention of lithium is enhanced by a high-sodium
parative efficacy and adverse event risks in sub-popudiet
lations of patients with chronic non-cancer pain.
C. Early signs of lithium toxicity may include tremors
C. There was insufficient evidence from available trials to
D. The onset of lithium action occurs within 24 hours
determine whether long-acting opioids as a class are
E. All of the above
more effective or associated with fewer adverse events
than short-acting opioids.
2332. Benzodiazepines differ from barbiturates in that
D. There were approximately 20 randomized trials investibenzodiazepines :
gating long-acting oxycodone versus short-acting oxyA. Facilitate the action of aminobutyric acid (GABA) on
codone showing conclusive evidence that long-acting
neuronal chloride channels
oxycodone was superior to short-acting oxycodone.
ASIPP
468
Section 12 • Controlled Substance Management
E. Recent GAO reports, DEA statements, and media attention and case reports of abuse, addiction, and overdose from long-acting opioids represent a decreased
risk proportionate to prescribing pattern changes as
these are widely used.
B. They are safe in patients with a history of hypertension
C. They should be given morning and noon so as not to
disturb sleep
D. They are safe in patients with a history of delirium
E. They should be administered only at bedtime
2337. The most commonly used illicit drug is:
A. Marijuana
B. LSD
C. Ecstasy
D. Methamphetamine
E. OxyContin®
2343. Tolerance is:
A. A need to increase drug dosage to obtain the same effect.
B. A rapid immunity to opioids, secondary to cross-reactive antibodies.
C. The concept of understanding that a drug is necessary,
except in the community.
D. A patient’s ability to take the drug.
E. The physician’s willingness and acceptance to prescribe
the drug.
2338. Choose the correct statement about effectiveness of
one or more long-acting opioids in reducing pain and
improving functional outcomes.
A. There was insufficient evidence to prove that different
long-acting opioids are associated with different ef- 2344. A weak acid drug (A), with a pKa = 6, is given orally.
ficacy or adverse event rates.
Assuming that the pH of the stomach equals 3 and the pH
B. OxyContin was shown to be more effective with a lesser
of the blood equals 7, which of the following statements
side effect profile compared to morphine.
is true?
C. MS Contin was superior to transdermal fentanyl.
A. At equilibrium, there is roughly 1000 times more dissoD. The combination of OxyContin, Xanax, and Soma were
ciated drug than undissociated drug in the stomach
superior to all other drugs.
B. At equilibrium, the ratio of dissociated to undissociated
E. The combination of morphine, diazepam, and oxycodrug in the blood is approximately 10
done were superior to MS Contin alone.
C. At equilibrium, 10 times more undissociated drug than
dissociated drug is in the blood
2339. Which controlled substance produces a toxic metabolite
D. Drug concentrations on the blood side of the stomach
which may cause seizures with accumulation with
barrier will never reach the concentration of drug in
repeated dosing?
the stomach
A. Pentazocine
E. The drug will be more rapidly excreted if the urine is
B. Propoxyphene
made acidic
C. Morphine
D. Ketamine
2345. Identify the LEAST important advice in defending drug
E. Meperidine
charges:
A. Comply with all federal and state laws and regulations
2340. Which of the following is an action of a non-competitive
governing prescribing – have and follow a compliance/
antagonist?
risk management program
A. Alters the mechanism of action of an agonist
B. Keep current with and comply with DEA Policy StateB. Alters the potency of an agonist
ments
C. Shifts the dose-response curve of an agonist to the
C. Comply with Kentucky Board of Medical Licensure
right
Policies, Guidelines, and Newsletters, especially
D. Decreases the maximum response to an agonist
Guidelines for Use of Controlled Substances in Pain
E. Binds to the same site on the receptor as the agonist
Treatment
D. Keep up with your documentation
E. Check if the patients are paying for visits.
2341. When compared to the general population, the mortality
rate of a prescription drug abuse is about:
2346. Federal Control over the use of controlled substances is
A. The same
under the supervision of
B. 3 times higher
A. Balanced budget act of 1997
C. 6 times higher
B. Food and drug Cosmetic Act administered by the
D. 10 times higher
F.D.A.
E. 25 times higher
C. Judicial branch of the government
D. Executive branch of the government
2342. Which of the following statements is accurate with
E. National Narcotics Bureau (N.N.B.)
administration of psychostimulants in opioid induced
sedation?
2347. Which one of the following is the most frequently abused
A. They are safe in patients with a history of paranoid
opiate in the United States?
disorders
A. Morphine
B. Toradol
ASIPP
Section 12 • Controlled Substance Management
469
C. Hydrocodone
D. Oxycodone
E. Fentanyl lollypops
2352. Which of the following statements applies to a drug
exhibiting a saturated elimination process?
A. Upon multiple dosing, steady-state plasma concentrations will be reached in approximately 4 to 5 biologic
2348. Which of the following is NOT true about
half-lives
benzodiazepines when used long term in chronic pain?
B. The fraction of drug eliminated per unit time is conA. Decrease REM and slow wave sleep
stant
B. Decrease serotonin levels
C. The biologic half-life (t1/2) is affected by dose
C. May produce a dangerous withdrawal syndrome if sudD. First-order kinetics are operable
denly discontinued
E. The rate of drug elimination is dependent on plasma
D. Have a primary analgesic effect
drug concentration
E. May have a much higher rate of cognitive dysfunction
than opioids
2353. According to NIDA household surveys and the National
Comorbidity Survey, the prevalence of substance abuse
2349. The most frequent type of drug interaction that occurs
among U. S. adults is about:
in patients using drugs of the sedative hypnotic class is
A. 1%
A. Additive CNS depression
B. 3%
B. Antagonism of sedative or hypnotic actions
C. 7%
C. Competition for plasma protein binding
D. 17%
D. Induction of liver drug-metabolizing enzymes
E. 33%
E. Inhibition of liver drug-metabolizing enzymes
2354. According to the DSM-IV, which of the following is NOT
2350. A patient presents for treatment of mechanical and
a criterion for diagnosing substance DEPENDENCE?
radicular chronic spine pain resulting from severe
A. Continued use despite physical or psychological probdegenerative disease with multilevel stenosis. He has
lems caused by use.
had a 2 level fusion in the past, undergone multiple
B. Recurrent substance related legal problems
interventional techniques and physical therapy without
C. Development of tolerance to the substance
benefit. His radicular pain has improved somewhat
D. Attempts to cut down substance use
with gabapentin 400 mg QID but he continues to have
E. An episode of withdrawal
severe mechanical back pain. He has a past history of
cocaine and alcohol abuse, but has been clean and sober 2355. A patient complains of worsening chronic temporal
for 7 years. When considering chronic opioid therapy in
headache, despite daily treatment with aspirin,
this patient which of the following is the most correct
butalbital, caffeine and ergotamine. MRI of the head was
statement?
normal, but MRI of the neck demonstrated spondylosis.
A. Opioid therapy may be appropriate, but the patient
Headache most likely is due to:
must be advised he has increased risk of relapse or
A. Migraine
cross addiction.
B. Drug rebound phenomenon
B. Opioid therapy is not appropriate because of his past
C. Cervical spondylosis
history of substance abuse.
D. Pseudo-tumor cerebri
C. The patient is not at increased risk because opioids were
E. Vasodilation due to ergotamine
never his drug of choice.
D. Opioid therapy is acceptable, but only short-acting 2356. Which one of the following statements about pentazocine
agents such as hydrocodone or oxycodone should be
(Talwin®) is FALSE?
used.
A. Analgesia is at least equivalent to that of codeine
E. The physician could be arrested for “aiding and abetting
B. Causes sedation
addiction” if he or she prescribes controlled substances
C. Classified as a mixed agonist-antagonist
for the patient.
D. Full agonist at mu receptors
E. May interfere with the analgesic effects of morphine
2351. A substance abuser enters and completes a 6 months long
term treatment program. His or her chance at remaining
sober is about:
2357. Among persons with a known substance abuse disorder,
A. 5%
the substance most commonly abused is:
B. 10%
A. Marijuana
C. 30%
B. Prescription opiates
D. 60%
C. Cocaine
E. 90%
D. Benzodiazepines
E. Alcohol
ASIPP
470
Section 12 • Controlled Substance Management
2358. Which of the following is the most accurate definition of tolerance:
A. The medication stops working after a few months
B. Stopping the medication causes withdrawal symptoms
C. A given dose is less effective, increasing the dose restores the effect
D. Side effects of a given dose are less severe over time
E. A medication is less effective, changing to a different
medication restores the effect
2359. A patient who had called for an early refill of opioid
medication is rather agitated and jittery when seen in
the office. She states she had not run out of her opioid
medication but simply thought she needed a higher dose.
All of the following symptoms would suggest opiate
withdrawal EXCEPT :
A. Diarrhea
B. Piloerection
C. Pinpoint pupils
D. Sweating
E. Rhinorrhea
2360. Adults who have completed four years of college are:
A. Less likely to use an illicit drug
B. More likely to use an illicit drug
C. Adults with four years of college who are alcoholics are
more likely to use an illicit drug.
D. Adults who have not completed high school, are dropouts, or live in metropolitan areas are more likely to
use illicit drugs.
E. There is no evidence that education has an effect on the
statistical prevalence of illicit drug use.
2361. Neurochemically, ALL substances considered abusable
eventually activate:
A. Cholinergic pathways in the brainstem
B. Adrenergic pathways in the frontal lobes
C. GABA-B receptors diffusely throughout the brain
D. Serotonergic centers in the diencephalon
E. Dopaminergic neurons in the mesolimbic system
2362. Which of the following is NOT a condition to an oral
refill of a Schedule III or IV prescription?
A. The total quantity authorized, including the amount of
the original prescription, does not exceed five refills
nor extend beyond six months from the date of the
original prescription.
B. The pharmacist obtaining the oral authorization must
record on the reverse of the original prescription the
date, quantity of refill, and the number of additional
refills authorized, and must initial the prescription.
C. The quantity of each additional refill authorized is
equal to or less than the quantity authorized for the
initial filling of the original prescription.
D. The pharmacist must verify that the oral authorization
came from a physician or other practitioner by, for
example, calling the physician back at the number for
the physician listed in the telephone directory.
E. The prescribing practitioner must execute a new and
separate prescription for any additional quantities
ASIPP
beyond the five-refill, six-month limitation.
2363. According to the DSM-IV, which of the following is a
criterion for diagnosing substance ABUSE?
A. Using a substance for a purpose other than that described in the PDR
B. Substance use in hazardous situations
C. Development of tolerance to the substance
D. Attempts to cut down substance use
E. An episode of withdrawal
2364. A heroin addict comes to the emergency room in an
anxious and agitated state. He complains of chills, muscle
aches, and diarrhea; he has also been vomiting. His
symptoms include hyperventilation and hyperthermia.
He claims to have had an intravenous “fix” approximately
12 hours ago. The attending physician notes that pupil
size is greater than normal. What is the most likely cause
of these signs and symptoms?
A. The patient has overdosed with an opioid
B. These are early signs of the toxicity of MPTP, a contaminant in “street heroin”
C. The signs and symptoms are those of the abstinence
syndrome
D. In addition to opioids, the patient has been taking
barbiturates
E. The patient has hepatitis B
2365. Which one of the following drugs is most likely to
increase plasma levels of alprazolam, theophylline, and
warfarin:
A. Desipramine (Pamelor®)
B. Fluvoxamine (Luvox®)
C. Imipramine (Tofranil®)
D. Nefazodone (Serazone®)
E. Venlafaxine (Effexor®)
2366. A patient injured in an auto accident received 80 mg
of meperidine. He subsequently developed a severe
reaction characterized by tachycardia, hypertension,
hyperpyrexia, and seizures. When a questioned, the
uninjured spouse revealed that the patient had been
taking a drug for a psychiatric condition. Which of the
following drugs is most likely to be responsible for this
untoward interaction with meperidine?
A. Alprazolam (Xanax®)
B. Amitriptyline (Elavil®)
C. Lithium
D. Mirtazapine (Norbil®)
E. Phenelzine (Nardil®)
2367. Fentanyl patches have been used to provide analgesia.
The most dangerous adverse effect of this mode of
administration is
A. Cutaneous reactions
B. Diarrhea
C. Hypertension
D. Relaxation of skeletal muscle
E. Respiratory depression
Section 12 • Controlled Substance Management
471
2368. A recently bereaved 74-year-old female patient was 2374. A patient complains of inadequate analgesia and
treated with benzodiazepine for several weeks after the
increases his use of his medication. This behavior may
death of her husband, but she did not like the daytime
represent:
sedation it caused. She has no major medical problems but
A. Addiction
appears rather infirm for her age and has poor eyesight.
B. Drug abuse
Because her depressive symptoms are not abating, you
C. Tolerance
decide on trial of an antidepressant medication. Which
D. Drug diversion
one of the following drugs would be the most appropriate
E. All or any of the above
choice for this patient?
A. Amitriptyline
2375. Convulsions caused by drug poisoning are most
B. Mirtazapine
commonly associated with
C. Paroxetine
A. Phenobarbital
D. Phenelzine
B. Diazepam
E. Trazodone
C. Strychnine
D. Chlorpromazine
2369. Which of the following statements are true?
E. Phenytoin
A. Opioid calculators are very useful and reliable.
B. If a patient complains of breakthrough pain, you should 2376. Phencyclidine may best be characterized by which of the
double the dose of long acting opioid every day until
following statements?
pain relief.
A. It has opioid activity
C. Nerve pain might respond better to anticonvulsants
B. Its mechanism of action is related to its anticholinergic
than opioids
properties
D. Bone pain might respond better to anticonvulsants
C. It can cause significant hallucinogenic activity
than opioids
D. It causes significant withdrawal symptoms
E. Opioid hyperalgia may be due to M6G accumulation
E. Treatment of overdose is with an opiate
2370. A PET scan is performed on a known substance abuser 2377. Disadvantages of long-term morphine therapy in a
who receives their drug of choice during the procedure.
cancer patient are:
The PET scan would show activation of :
A. Withdrawal when drug is stopped
A. Nucleus Accumbens
B. Cognitive dysfunction
B. Ventral Tegmental Areas
C. Addiction
C. Central Nucleus of the Amygdala
D. Liver dysfunction
D. All of the above
E. Constipation
E. None of the above, they would actually be suppressed
2378. The most commonly used illicit drug in America is:
A. Oxycontin®
2371. When prescribing sublingual or buccal route of
B. Cocaine
administration, the best drug (55%) for absorption is
C. Morphine®
A. Methadone
D. Marijuana
B. Fentanyl
E. Alcohol
C. Morphine
D. Buprenorphine
2379. Cocaine, produced from the leaves of Erythroxylon
E. Demerol
species,
A. Produces bradycardia and vasodilation
2372. The management of phencyclidine toxicity is likely to
B. Is directly related chemically to opioid analgesics
include all of the following except:
C. Is metabolized by the microsomal metabolizing system
A. Naloxone for respiratory depression
D. Blocks nerve conduction effectively
B. Diazepam for seizures
E. Blocks norepinephrine receptors directly
C. Propranolol for adrenergic crisis
D. Haloperidol for disorganized, disruptive behavior
2380. Which of the following statements about the CSA is
E. Low-stimulus environment
true?
A. If a practitioner needs only a small number of dosage
2373. Which of the following is associated with crack
units of a controlled substance for office use , a pre(the free-base form of cocaine)?
scription order may be issued to permit a pharmacy
A. Flashbacks (recurrences of effects) may occur months
to dispense them.
after the last use of the drug
B. The Act prohibits treating a known addict with an
B. It may cause seizures and cardiac arrhythmias
opioid/opiate product for persistent (as opposed to
C. It acts by blocking adrenergic receptors
acute) pain.
D. It is the salt form of cocaine
C. The defines the term “narcotic” as those controlled subE. It is primarily administered intranasally
stances which are opioids or opiates, whether natural,
ASIPP
472
Section 12 • Controlled Substance Management
synthetic, or semi-synthetic.
D. Cocaine is classified by the Act as a C-I substance.
E. The Act permits DEA Diversion Investigators to inspect
your controlled substances records (inventories, storage facility, order forms, etc.) at any time during your
normal practice hours.
2381. Patients usually develop tolerance to all opioid effects
EXCEPT:
A. Sedation
B. Pruritus
C. Constipation
D. Pain relief
E. Respiratory depression
2382. Which of the following is not an accurate statement
regarding personal protective equipment (“PPE”)?
A. The PPE does not permit blood or other potentially infectious materials to pass through or reach employee’s
work clothes, skin, face, etc.
B. PPE must be readily accessible in the workplace or issued to the employee.
C. The employer is responsible for laundering PPE.
D. The employer is responsible for repairing PPE.
E. A physician practice may deduct the cost of an employee’s PPE from his or her paycheck.
2383. Which of the following is NOT an element of an effective
compliance program for a physician practice?
A. Regular auditing and monitoring
B. Education and training on the program for the physicians with an equity interest in the practice
C. Education and training for all personnel in the practice
D. Written practice standards including a code/standard
of conduct
E. A response mechanism and corrective action plan
2384. Which of the following characteristics of buprenorphine
best account for its effectiveness in the treatment of
opioid dependence and detoxification?
A. Sublingual formulation with naloxone
B. High opioid mu-1 receptor affinity and slow receptor
dissociation
C. Partial kappa receptor agonist activity
D. Low oral bioavailability necessitating parenteral administration
E. High intrinsic activity at the opioid mu receptor
2385. Compared to morphine, butorphanol would be expected
to display which one of the following pharmacologic
characteristics?
A. Kappa receptor antagonist activity
B. Analgesic ceiling effect
C. No respiratory depression
D. Histamine release
E. Mu receptor agonist activity
ASIPP
2386. According to the Federal Controlled Substances Act of
1970, a Schedule III drug could be considered to possess
which one of the following characteristics:
A. Lack of accepted safety and indication for medical use
B. Limited, if any, physical or psychological dependence
C. High potential for abuse
D. No withdrawal syndrome noted with abrupt discontinuation of use
E. Analgesic, anabolic, sedative, and/or hypnotic effects
2387. Compared to a short-acting, immediate release opioid, a
long-acting, sustained release opioid differs in which one
of the following characteristics?
A. Higher level of opioid receptor affinity and intrinsic
activity
B. Greater degree of tachyphylaxis
C. Faster development of tolerance
D. Protracted withdrawal syndrome following abrupt
discontinuation
E. No ceiling effect for analgesia
2388. Under the Controlled Substances Act (CSA), the
following drug schedules is thought to have the highest
potential for abuse?
A. Schedule II
B. Schedule III
C. Schedule V
D. Schedule I
E. Schedule IV
2389. A physician determines that an emergency situation
exists justifying a Schedule II emergency oral
prescription. Which of the following statements is true?
A. Within 7 days after the oral authorization to the pharmacist, the physician must deliver to the pharmacist
a written prescription for the emergency quantity
prescribed, and the written prescription must have
written on its face “Authorization for Emergenc
B. Within 14 days after the oral authorization to the pharmacist, the physician must deliver to the pharmacist
a written prescription for the emergency quantity
prescribed and the written prescription must have
written on its face “Authorization for Emergenc
C. Within 30 days after the oral authorization to the pharmacist, the physician must deliver to the pharmacist
a written prescription for the emergency quantity
prescribed and the written prescription must have
written on its face “Authorization for Emergenc
D. No further action is required.
E. Within 60 days after the oral authorization to the pharmacist, the physician must deliver to the pharmacist
a written prescription for the emergency quantity
prescribed and the written prescription must have
written on its face “Authorization for Emergenc
Section 12 • Controlled Substance Management
473
2390. Choose the most likely effect resulting from treatment 2396. Regarding low back pain in a primary care practice:
with 10 mg of diazepam 3 times daily?
A. Less than 10% of new low back pain patients followed
A. Retrograde amnesia
up with a doctor at 12 months.
B. Improved performance on tests of psychomotor funcB. Less than 10% of new low back pain patients still had
tion
low back pain at 12 months
C. Alleviation of the symptoms of major depressive disC. Less than 10% of new low back patients still had diforder
ficulties with ADLs at 12 months
D. Increased porphyrin synthesis
D. Less than 10% of new low back pain patients still had
E. Agitation and possible hyperreflexia with abrupt dispain complaints at 3 months
continuance after chronic use
E. Less than 10% of new low back pain patients followed
up with a doctor at 3 months
2391. A morbidly obese patient with low back pain complains
of not sleeping well and feeling tired during the day. His 2397. Which of the statements about tolerance is most true
wife wakes him up several times during the night due to
in a patient taking repeated daily doses of lysergic acid
his loud snores. Patient wants a prescription for sleep
diethylamide (LSD)?
medicine. Your next step is:
A. No tolerance develops
A. Prescribe Clorazepate
B. Tolerance develops in 3 to 4 days
B. Prescribe Flurazepam
C. Tolerance develops in 2 to 3 weeks.
C. Prescribe Secobarbital
D. Tolerance develops in 2 to 3 months.
D. Prescribe Triazolam
E. Tolerance develops in 6 months.
E. Refer to sleep disorder clinic
2398. A 17-year old who ingests “mushrooms” would present
2392. Choose the accurate statements concerning the
with symptoms best described as:
Barbiturates and Benzodiazepines:
A. Anticholinergic
A. Compared with benzodiazepines, barbiturates exhibit a
B. Adrenergic
flatter dose-response relationship
C. Cholinergic
B. Respiratory depression caused by barbiturate overdosD. Alpha adrenergic
age can be reversed by flumazenil
E. Dopaminergic
C. An increase in urinary pH will accelerate the elimination of phenobarbital
2399. To avoid criminal investigations into prescribing
D. Barbiturates may increase the half-lives of drugs mepatterns for controlled substances, a physician should . .
tabolized by the liver
.Choose the answer that best completes this sentence.
E. Symptoms of the abstinence syndrome are less severe
A. Warn patients to fill prescriptions at different drugduring the withdrawal from secobarbital than from
stores.
phenobarbital
B. Make sure there is a logical relationship between the
drugs prescribed and the treatment of the condition
2393. What is the first step in a patient on 300 mg Tramadol per
allegedly existing.
day with paroxetine(Paxil®) with inadequate pain relief ?
C. Issue prescriptions to patients known to be selling
A. Change the paroxetine
drugs to others.
B. Add a benzodiazepine
D. Prescribe controlled substances at intervals inconsistent
C. Increase the tramadol
with legitimate medical treatment.
D. Switch to a more potent opioid
E. Use street slang when talking about the drugs preE. Switch to an agonist/antagonist opioid
scribed.
2394. The greatest risk in a patient on 300 mg of daily Tramadol 2400. Which of the following statements is NOT true with
(Ultram®) and paroxetine (Paxil®) combination is:
regard to a continuing criminal enterprise?
A. Withdrawal
A. Conviction for being the manager or organizer of a
B. Increased depression
continuing criminal enterprise results in the person
C. Seizures
being sentenced to not less than 20 years and not more
D. Increased pain
than life imprisonment, a fine of $2 million or more,
E. GI bleeding
and forfeiture of assets under 21 USC 853.
B. To be convicted of being the manager or organizer of a
2395. Adverse effects of opioids that can be used to advantage
continuing criminal enterprise, one must obtain subinclude:
stantial income or resources.
A. Dysphoria
C. To be convicted of being the manager or organizer of a
B. Respiratory depression
continuing criminal enterprise, at least five other perC. Decreased GI motility
sons must be involved.
D. Pupil constriction
D. To be convicted of being the manager or organizer of a
E. Sexual dysfunction
continuing criminal enterprise, a person must violate
a provision of the Controlled Substances Act, the pun-
ASIPP
474
Section 12 • Controlled Substance Management
ishment for which is a misdemeanor.
E. To be convicted of being the manager or organizer of a
continuing criminal enterprise, a person must engage
in a continuing series of violations of the Controlled
Substances Act.
2401. A practitioner intending to dispense and prescribe
Schedule III, IV, or V controlled substances for
maintenance and detoxification treatment must submit
to the Secretary of the Department of Health and Human
Services a notification that he or she intends to do so. The
notification must state . . .Which one of the following
would not correctly complete this sentence?
A. That the practitioner has the capacity to refer patients
to whom the practitioner provides narcotic drugs for
appropriate counseling and other ancillary services.
B. If the practitioner is in solo practice, that s/he will not
treat more than thirty patients at any one time with
Schedule III, IV, or V drugs for detoxification or maintenance.
C. If the practitioner is in group practice, that the group
practice will not treat more than sixty patients at any
one time with Schedule III, IV, or V drugs for detoxification or maintenance.
D. That the practitioner is a qualified physician, as that
term is defined in the federal statutes.
E. If the practitioner is in group practice, the names and
DEA registration numbers for all practitioners in a
group practice.
unless renewed by the prescribing practitioner.
C. No Schedule III or IV drug may be dispensed without
a written or oral prescription, except when dispensed
directly by a physician or other practitioner to an ultimate user.
D. A pharmacist may dispense a Schedule III or IV drug
based on a fax to the pharmacy of a written, signed
prescription transmitted by the practitioner or one of
his/her staff.
E. A pharmacist may dispense a Schedule III or IV drug
based on an oral prescription received from the
prescribing practitioner, if the oral authorization is
promptly reduced to writing by the pharmacist.
2404. Compared with diazepam (Valium®),
midazolam(Versed®)
A. Is more lipid soluble
B. Has a longer elimination half-life
C. Has a larger volume of distribution
D. Has greater clearance
E. Undergoes slower hepatic metabolism
2405. According to the 2003 National Survey on Drug Use and
Health (NSDUH), 6.3 million persons age 12 or older
used prescription medications for non-medical reasons.
Which of the following is the most prevalent agent for
non-medical reasons?
A. Pain reliever
B. Tranquilizers
C. Stimulant
2402. Under the Drug Addiction Treatment Act of 2000,
D. Sedatives
practitioners who are qualified physicians may
E. NSAIDs
dispense and prescribe Schedule III, IV, or V controlled
substances specifically approved by the Food and Drug 2406. Meperidine
Administration for narcotic addiction treatment to a
A. Has been used for many years and is appropriate for
narcotic dependent person if the practitioner meets
chronic use
several requirements.Which one of the following
B. Is metabolized by CYP2D6
credentials will NOT make a physician a qualified
C. Is safe to use in elderly patients
physician?
D. May cause side effects that cannot be reversed by opioid
A. The physician holds a subspecialty board certification
antagonists.
in addiction psychiatry from the American Board of
E. Becomes more effective over time, because of an active
Medical Specialties.
metabolite
B. The physician holds an addiction certification from the
American Society of Addiction Medicine.
2407. Supportive management (“talking down”) of most acute
C. The physician holds a subspecialty board certification
reactions to lysergic acid diethylamide (LSD):
in addiction medicine from the American Osteopathic
A. Should be accompanied by an injection of diazepam
Association.
(Valium).
D. The physician has completed not less than 8 hours of
B. Should be attempted in a hospital or emergency room
training regarding the treatment and management of
setting.
opiate-dependent patients.
C. Is not indicated for these reactions.
E. The physician has published a peer-reviewed article on
D. Can usually be accomplished without medication or
the treatment of opiate-dependent patients.
hospitalization.
E. Should be accompanied by an injection of haloperidol
2403. Which of the following is NOT a true statement with
(Haldol).
respect to a Schedule III or IV drug?
A. Schedule III and IV prescriptions may not be filled or 2408. The following statements are true regarding opioidrefilled more than six months after the date the origiinduced constipation.
nal prescription was issued.
A. Treat constipation
B. Schedule III and IV prescriptions may not be refilled
B. To obtain a surgical consult to rule out complications
more than 10 times after the date of the prescription
C. To evaluate for drug abuse
ASIPP
475
Section 12 • Controlled Substance Management
D. Start on transdermal fentanyl
E. Start on methadone maintenance program
2409. Which of the following is true about the federal
Controlled Substances Act:
A. It prohibits self-prescribing.
B. It permits DEA Diversion Investigators to inspect patient records without patient consent, upon written
request.
C. It allows most practitioners to prescribe C-I substances
for IRB-approved research
D. It permits a “Do Not Fill Until” instruction on a prescription order.
E. It prohibits prescribing methadone without a special
registration as a methadone clinic.
2410. A meperidine (Demerol) dose equivalent to morphine 10
mg every 3 to 4 hours by injection is:
A. 100 mg every 3 hours
B. 75 mg every 3 hours
C. 50 mg every 3hours
D. 100 mg every 4 hours
E. 75 mg every 4 hours
2411. A 22 year old weight lifter who has been observed to have
a gradual behavior change marked by aggression and
mood swings may be demonstrating the effects of:
A. Increased protein intake
B. Creatine and chromium nutritional supplements.
C. Prednisone tablets
D. Anabolic steroids
E. Massive doses of vitamins
2412. The states of a patient in which repetitions of the same
dose of a drug has progressively less effect or in which the
dose needs to be increased to obtain the same degree of
pharmacological effect is most likely
A. Physical dependence
B. Synergistic effect
C. Additive effects
D. Disuse supersensitivity
E. Tolerance
A. Phencyclidine (PCP) intoxication.
B. Atropine intoxication
C. Benzodiazepine intoxication
D. Mescaline intoxication
E. Solvent intoxication
2415. All of the following statements are sources of confusion
in describing chronic pain EXCEPT:
A. Pain can be described in terms of its physiological underpinnings and its felt experience
B. Patients are ashamed of acknowledging the psychological contributors to their pain.
C. Practitioners are uncomfortable with acknowledging
psychological contributors
D. Pain is an unpleasant sensory and emotional experience associated with actual or potential tissue damage
or described in terms of such damage (IASP).
E. Pain is best described if only structural abnormalities
are utilized.
2416. Death from acute intoxication with phencyclidine is
most likely to occur as a result of:
A. Violence
B. Cholinergic crisis.
C. Hypertensive crisis.
D. Cardiac arrest.
E. Status epilepticus.
2417. A known heroin addict underwent emergency surgery
after a motor vehicle injury. He received morphine 10
mg IV, three doses in 2 hours which helped him only 15
minutes. The course of treatment in this patient is:
A. To titrate morphine
B. To obtain a surgical consult to rule out complications
C. To evaluate for drug abuse
D. Start on transdermal fentanyl
E. Start on methadone maintenance program
2418. Reported symptoms of Methylenedioxyamphetamine
(DMA) and its analog methylenedioxymethamphetam
2413. While on a maintenance dose of methadone 80 mg per
ine (MDMA) are reported to cause all of the following
day, a patient “shoots up” the heroin equivalent of 10 mg.
EXCEPT:
The most likely effect will be:
A. Mild stimulation
A. Euphoria and sedation
B. A feeling of well-being.
B. Sedation only.
C. Visual illusions or hallucinations.
C. Neither euphoria nor sedation
D. Auditory hallucinations.
D. Lacrimation, piloerection, and abdominal cramps
E. Anxiety
E. Respiratory distress, chest pain, and tachycardia
2419. Flashbacks can occur:
A. Long after the hallucinogenic intoxication has dis2414. A 15-year-old male high school freshman presents to an
sipated.
emergency department with a blank stare, belligerence,
B. Only in patients with pre-existing psychological probpsychomotor agitation, horizontal nystagmus, vertical
lems.
nystagmus, blood pressure 160/110 mm Hg, ataxia
C. As a result of impurities in street psychedelics.
dysarthria, and diminished responsiveness to pain.
D. Usually a half hour after the drug has been ingested.
He appears to be hallucinating, and as he is being
E. In patients with combined use of hallucinogens and
interviewed he assaults one of the attendants. The most
opioids
likely diagnosis is:
ASIPP
476
2420. The following pathology results from infective
endocarditis in heroin addicts, compared to endocarditis
in non-addicted patients:
A. The tricuspid valve is diseased
B. Mixed flora of bacteria
C. Staphylococcus aureus is found more often as a causative organism.
D. Surgical treatment is mandatory
E. Easily identified by an aortic murmur
2421. The Golden Crescent and the Golden Triangle refer to:
A. A geographic region of Africa where opium is grown.
B. A geographic region of Asia where marijuana is
grown.
C. Various names from heroin.
D. Symbols used in drug trafficking.
E. A geographic region of Asia where opium is grown.
2422. Identify evidence-based recommendations of
meperidine use in chronic pain.
A. Given no more frequently than every four hours for 6
months
B. Used in standard doses in the elderly indefinitely if
response is positive
C. Used in individuals with impaired renal function as
meperidine is shown to have only hepatotoxicity
D. Always used in conjunction with non-steroidal antiinflammatory agents.
E. Reserved for very brief therapy in otherwise healthy
patients who cannot tolerate other opioids
2423. Identify the statement describing the withdrawal when
chronic opioid use is discontinued?
A. Is of no clinical significance
B. Can be prevented by administering of a benzodiazepine
C. Can be prevented by administering of an amphetamine
D. Is best managed by slowly tapering the opioid dose by
no more than 10% every few days.
E. Can be alleviated by immediately starting on an opioid
agonist-antagonist
2424. The US work loss related to pain:
A. Half the workforce report having pain in the last two
weeks.
B. 10% of the work force was absent from work one or
more days per week
C. Abdominal pain was the most common complaint.
D. Half of the workforce lost productive time due to pain
E. An average of 8 hours of work per week is lost because
of pain
2425. A patient of yours is brought in for evaluation by his
wife, 3 days after cervical epidural steroid injection with
multiple complaints. On examination he is found to
be disoriented, confused and in a fugue-like state with
nystagmus on upward gaze, tachycardia and elevated
ASIPP
Section 12 • Controlled Substance Management
blood pressure. The most likely diagnosis is:
A. Cocaine intoxication
B. Cannabis intoxication
C. Barbiturate intoxication
D. Phencyclidine (PCP) intoxication
E. Epidural abscess
2426. What are the main types of legal/regulatory material at
the federal level governing record-keeping for the use of
controlled substances for the treatment of pain?
A. Acts and Laws, regulations and rules, and guidelines,
policy or position statements.
B. Laws and regulations.
C. Controlled Substances Act of 1970.
D. DEA Policy Statements.
E. Food and Drug Act.
2427. Applied to the skin in a transdermal patch (transdermal
therapeutic delivery system), this drug is used to prevent
or reduce the occurrence of nausea and vomiting that are
associated with motion sickness
A. Diphenhydramine
B. Chlorpromazine
C. Ondansetron
D. Dimenhydrinate
E. Scopolamine
2428. Use of which the following opioids by breast-feeding
mothers depresses
the behavior of the infant more
than the equianalgesic dose of morphine:
A. Fentanyl®
B. Meperidine®
C. Nalbuphin®
D. Buprenorphine®
E. Tramadol®
2429. The Controlled Substances Act of 1970 does not give
DEA the authority to do which of the following:
A. Scheduled drugs
B. Regulate medical practice
C. Administer the CSA and create policy related to the
CSA
D. Establish quotas for the manufacture of controlled
substances
E. Reschedule drugs depending on their potential for
abuse
2430. In order for a prescription to be valid under federal and
state law, it must be issued
A. With the proper date and the physician’s signature.
B. With the proper date, patient information, drug identification and instructions for use, physician signature,
and it must be issued for a legitimate medical purpose
within the usual course of professional practice.
C. To a patient who does not have a criminal history.
D. Only by a licensed physician and not by any mid-level
practitioner.
E. Within three days of seeing the patient.
Section 12 • Controlled Substance Management
477
2431. As a DEA registrant you have certain responsibilities,
D. The Controlled Drug Act.
including (1) proper registration and renewal; (2)
E. The Controlled Substances Act of 1970.
proper record-keeping; and (3) what newly explained
responsibility as stated in the Interim Policy Statement, 2436. You have a patient that you suspect may be altering
published by DEA in the Federal Register on November
your prescriptions. You want to fax a copy of a Schedule
16, 2004?
II prescription to the pharmacist the patient uses to fill
A. A responsibility to report to DEA about the misuse of
prescriptions. Is this legal and what must happen before
a DEA number.
the pharmacist dispenses the prescription?
B. A responsibility to seriously consider any sincerely
A. A.
No, it is not legal to fax a Schedule II preexpressed concerns made by family members about a
scription to a pharmacist.
patient’s potential abuse of controlled substances.
B. B.
Yes, but only if you have a HIPAA consent
C. A responsibility to see all patients every thirty days.
from the patient.
D. A responsibility to issue drugs for a legitimate medical
C. C.
Yes, and the patient must present the origipurpose.
nal prescription to the pharmacist.
E. A responsibility to review patient records every thirty
D. D.
No, because faxes apply only to emergency
days.
prescriptions for Schedule II drugs.
E. E.
No, because faxes apply only to hospice and
2432. How often must a practitioner renew his/her DEA
nursing home situations.
registration?
2437. Demerol (meperidine) should not be used for chronic
A. Every 2 years.
pain because:
B. Every 6 years.
A. It is addictive
C. Every 3 years.
B. It is ineffective
D. Every 4 years.
C. The metabolite causes seizures
E. Every 5 years.
D. The medication is expensive
E. All of the above
2433. Every practitioner who administers, prescribes, or
dispenses any controlled substance must be registered 2438. A former heroin addict is maintained on methadone, but
with the DEA and must maintain the DEA certificate of
succumbs to temptation and buys an opioid on the street.
registration at the registered location. If a practitioner
He takes it and rapidly goes into withdrawal. Which
has more than one office where controlled substances
opioid did he take?
are administered and/or dispensed, then the practitioner
A. Meperidine
must:
B. Heroin
A. Register only the principal office location.
C. Pentazocine
B. Register each office location where controlled substancD. Codeine
es are administered and/or dispensed.
E. Propoxyphene
C. Register only those office locations where controlled
substances are prescribed.
2439. You are a solo practitioner in a pain specialist capacity.
D. Do nothing.
You have a patient who wants to be treated in your office
E. Register every location that uses prescription pads.
for opioid addiction and pain. Do you need a separate
registration to provide Narcotic Treatment Services?
2434. If you move the location of your practice, you are
A. No, I can issue 72-hour emergency prescriptions as
required to do what regarding your DEA registration?
needed to detoxify a patient.
A. Send a request for modification of registration in writB. No, I can prescribe methadone to help them detoxify
ing to the nearest DEA field office, and obtain approval
because I prescribe methadone to treat their pain.
for the modified registration prior to the move.
C. No, I have the proper training on the use of Schedule
B. Wait until the next renewal date for your controlled
III-V drugs in the office based treatment of opioid
substances registration to notify DEA of the move.
addiction.
C. Tell only your state drug bureau about the move and
D. Yes, I must have a separate registration to provide narseek a new registration with them that you can use
cotic treatment services.
with DEA.
E. Yes, so I can prescribe drugs in Schedule II-V to detoxify
D. Send DEA a request for a new registration number after
a patient due to opioid addiction.
you move.
E. Wait until DEA asks you for updated information about 2440. Which one of the following effects is unlikely to occur
your registration and current address.
during treatment with amitriptyline?
A. Alpha adrenoceptor blockade
2435. The federal law governing the scheduling of drugs as
B. Elevation of the seizure threshold
“controlled substances” is called:
C. Mydriasis
A. The Food, Drug, and Cosmetic Act of 1962.
D. Sedation
B. The Federal Uniform Controlled Substances Act.
E. Urinary retention
C. The Code of Federal Regulations.
ASIPP
478
Section 12 • Controlled Substance Management
2441. The mechanism of action of most benzodiazepines is
by :
A. Activation of GABA receptors
B. Antagonism of glycine receptors in the spinal cord
C. Blockade of the action of glutamic acid
D. Increased GABA-mediated chloride ion conductance
E. Inhibition of GABA aminotransferase
2442. Concerning the proposed mechanisms of action
of antidepressant drugs, which one of the following
statements is accurate?
A. Bupropion (Wellbutrin®) can effective inhibitor of
NorEpinephrine and 5-HT transporters
B. Chronic treatment with an antidepressant often leads to
the up-regulation of adrenoceptor
C. Elevation in amine metabolites in cerebrospinal fluid is
characteristic of most depressed patient prior to drug
therapy
D. MAO inhibitors used as antidepressants selectively decrease the metabolism of norepinephrine
E. The acute effect of most tricyclics is to block the neuronal reuptake of both norepinephrine and serotonin in
the CNS
2443. A 28-year-old woman presents with symptoms of
major depression that are unrelated to a general medical
condition, bereavement, or substance abuse. She is
not currently taking any prescription or over-thecounter medications. Drug treatment is to be initiated
with a selective serotonin reuptake inhibitor. In your
information to the patient, you would NOT tell her that
A. Divided doses may help to reduce nausea and gastrointestinal distress
B. Muscle cramps and twitches sometimes occur
C. She must inform you if she anticipates using other
medications
D. Taking the drug in the evening will ensure a good
night’s sleep
E. The drug may require 2 weeks or more to become effective
2444. Which one of the following actions of opioid analgesics
is mediated via activation of kappa receptors?
A. Cerebral vascular dilation
B. Decreased uterine tone
C. Euphoria
D. Sedation
E. Psychologic dependence
2445. All of the statements are correct about chronic
pain syndromes, EXCEPT:
A. Pain that lasts longer than 6 months.
B. Varying levels of mental health problems are diagnosed
prior to, or concurrent with, onset of pain.
C. Impairment in vocational, social, emotional functioning.
D. Low rates of health care utilization.
E. Frequent issues of secondary gain and addiction to pain
medications.
ASIPP
2446. A deliberate (dealing) practitioner characteristics
include all of the following EXCEPT:
A. Practitioner becomes a mercenary
B. Sells drugs for money, sex, street drugs, etc.
C. Office becomes a pill factory—full of drug seekers
D. Prescribes for known addicts who will likely sell drugs
to others
E. Keeps close contact with DEA
2447. Is it legal for Internet pharmacies to approach a physician
to write prescriptions based on on-line consultations
with customers/consumer?
A. Yes, this is legal and on-line consultations qualify as a
proper physician-patient relationship.
B. Yes, but the physician must see the patient in person
and establish a valid physician-patient relationship
prior to issuing Internet prescriptions.
C. No, a physician cannot do this under existing law.
D. No, a physician cannot do this unless he/she obtains a
special Internet certification from the DEA.
E. Yes, but the physician must obtain a special Internet
certification from DEA.
2448. Correct statements describing the risks of malprescribing
include
A. Never given re-education options
B. Not reportable to databank
C. State Board Investigation or Sanction
D. Usually results in publicity to increase your practice
E. Not liable to civil lawsuits, only criminal liability
2449. You have a patient you have seen for several years. The
patient is stable and has been on the same controlled
substances, including a Schedule II drug, for one year.
The patient has been relatively compliant with your
treatment plan and fully compliant with medication
issues. Identify the federal legal/regulatory material that
prohibits you from issuing this patient multiple schedule
II medications with different fill dates or “do not fill
before” language on your prescriptions.
A. The Interim Policy Statement of November 16, 2004.
B. The Controlled Substances Act of 1970.
C. The Code of Federal Regulations pertaining to the issuance of prescriptions.
D. A, B, and C.
E. My state allows this so there is nothing in the federal
legal/regulatory material that prevents me from using
multiple schedule II prescriptions with different fill
dates with my patients.
2450. A drug that is used in the treatment of parkinsonism and
will also attenuate reversible extrapyramidal side effects
of neuroleptic is
A. Amantadine (Symmetrel®)
B. Levodopa (Dopar®)
C. Pergolide (Permax®)
D. Selegiline (Eldepryl®)
E. Trihexyphenidyl (Artane®)
Section 12 • Controlled Substance Management
2451. Meprobamate is the active metabolite of which skeletal
muscle relaxant?
A. Carisoprodol (soma®)
B. Cyclobenzaprine (Flexeril®)
C. Methocarbamol (Robaxin®)
D. Valdecoxib (Bextra®)
E. Baclofen®
2452. All of the following are true statements regarding
ketamine EXCEPT:
A. Ketamine is a dissociative anesthetic
B. Dissociative anesthesia induced by ketamine emphasizes that the anesthetized patient is “disconnected”from his or her environment
C. Ketamine is one of the most commonly abused drugs
D. Ketamine has been placed in Schedule I of the Federal
Controlled Substances Act.
E. Ketamine induces coma in a dose-dependent manner
479
compulsive disorders
B. Clonazepam has effectiveness in patients who suffer
from phobic anxiety states
C. Diazepam is used for chronic management of bipolar
affective disorder in patients who are unable to tolerate lithium
D. Intravenous buspirone is useful in status epilepticus
E. Symptoms of the alcohol withdrawal state may be alleviated by treatment with zaleplon
2456. Choose the correct statement about a duped
practitioner.
A. Never assumes the best about his patients and is gullible
B. Never leaves script pads lying around
C. Does not believe in hydrophilic medicine “fell” into the
toilet or the sink
D. Believes when patients only want specific medications
(i.e. OxyContin or Percocet)
E. Never co-dependent - always tells patients “No” when
they ask for narcotics
2453. You treat patients who suffer from conditions producing
chronic, non-malignant pain. You prescribe controlled
substances to your patients (1) for a legitimate medical 2457. Your friend’s daughter whom you have known for several
years makes an appointment with you. During the visit,
purpose, and (2) within the usual course of professional
she tells you that she is a heroin addict and requests
practice. The Federation of State Medical Boards and
a prescription for Hydrocodone. Your options in this
many state licensing boards require practitioners to keep
situation are as follows:
medical records that include which of the following items
A. Immediately call her father and give hydrocodone.
in connection with their use of controlled substances to
B. Immediately tell father and give her Methadone.
treat pain?
C. Start rapid detoxification in your office.
A. The name of the drug and the amount prescribed.
D. Provide her with a prescription for Methadone mainB. The medical history and physical examination, diagtenance
nostic, therapeutic and laboratory results, evaluations
E. Do not tell the father and do not give Hydrocodone.
and consultations, treatment objectives, discussion of
risks and benefits, informed consent and treatment
2458. Compared with midazolam, diazepam has which of the
agreements, treatments, medications (includ
following characteristics?
C. An inventory of all the drugs prescribed to each patient,
A. Greater solubility in water
a record of all communications with the patient, and
B. Shorter beta half-life
all office forms.
C. More potent ventilatory depressant effect
D. A carbon copy of the prescriptions issued, all contact
D. Lower risk for thrombophlebitis
information for the patient, and all clinical rationale
E. A pharmacologically active metabolite
for the drugs prescribed.
E. A list of each office visit you have with the patient and a
2459. True statement(s) with regards to urine drug testing
statement of all treatments rendered.
include:
A. Thin layer chromatography is a relatively new tech2454. A registered individual practitioner is not required to
nique, most sensitive, labor intense, and expensive.
keep records of controlled substances in Schedules II,
B. Gas chromatography is most sensitive, most reliable,
III, IV, and V which are prescribed in the lawful course of
inexpensive, an old and established technique.
professionalpractice, unless
C. Enzyme immunoassay is less sensitive than thin layer
A. He/she prescribes controlled substances in the course of
chromatography, more sensitive than gas chromamaintenance or detoxification treatment of a patient.
tography, and has ability to screen multiple drugs at
B. He/she prescribes controlled substances at more than
a time.
one practice location.
D. Rapid drug screens are similar to other enzyme imC. He/she uses the Internet for all patient contact.
munoassay tests but may be more expensive or less
D. He/she has been registered for less than three years.
expensive.
E. He/she uses electronic medical records.
E. Rapid drug screens are less sensitive than enzyme
immunoassay, gas chromatography, and thin layer
2455. Concerning the clinical uses of benzodiazepines and
chromatography and highly unreliable, but least exrelated drugs, which one of the following statements is
pensive.
accurate?
A. Alprazolam is effective in the management of obsessive-
ASIPP
480
2460. Which of the following opioids is vagolytic?
A. Morphine
B. Meperidine
C. Sufentanil
D. Nalbuphine
E. Alfentanil
2461. N-methyl-D-aspartate (NMDA) receptors are best
defined as:
A. NMDA receptors are calcium-permeable ion channels
that require only glutamate for activation.
B. NMDA receptors are calcium-permeable ion channels
that require only glycine for activation.
C. NMDA receptors are calcium-permeable ion channels
that require both glutamate and glycine for activation.
D. At the molecular level, NMDA receptors are composed
of a single subunit.
E. NMDA receptors have small intracellular C-termini
that interact with a single protein that regulates receptor phosphorylation.
2462. What are correct statements about drug interactions of
following statements?
A. Amitriptyline will increase morphine metabolism
B. Morphine will decrease desipramine levels
C. Erythromycin will decrease opioid levels
D. Tricyclic antidepressants will increase methadone
levels
E. Propoxyphene will increase propranolol levels
2463. Methadone in addition to being a µ-receptor agonist has
been proposed to also act as a:
A. COX-2 inhibitor
B. Sodium Channel blocker
C. NMDA receptor antagonist
D. Delta receptor agonist
E. Opiod Antagonist
2464. A patient presents to you with chronic low back pain. He
is being treated with OxyContin 40 mg twice a day and
hydrocodone 10 mg four times daily. You performed a
drug testing. The results of the drug testing were positive
for oxycodone, hydromorphone, and hydrocodone. The
results indicate the following:
A. The patient abusing controlled substances by taking
non-prescribed drugs.
B. He is non-compliant by not taking the prescribed
drugs.
C. He is selling drugs
D. The results of the drug test show a normal pattern
E. The drug test indicates the patient is taking his mother’s
hydromorphone
2465. A urine drug test in a patient on hydrocodone was
positive for hydrocodone and hydromorphone. Choose
the correct option.
A. The patient is taking Dilaudid
B. Refuse to write any more opioids
C. Ignore the results as a false positive
ASIPP
Section 12 • Controlled Substance Management
D. Counsel the patient regarding taking drugs that have
not been prescribed
E. Avoid the use of SSRIs in this patient
2466. The following statements are true with typical detection
times for urine testing of common drugs of abuse.
A. Methadone, 2 to 4 days
B. Chronic use of marijuana, 1 to 3 days
C. Morphine, 15 days
D. Cocaine, 15 days
E. Benzodiazepines, 15 days
2467. The Schedule I substance among the following drugs is?
A. Buprenorphine
B. Hydromorphone
C. Heroin
D. Diazepam
E. Morphine
2468. What are the correct statements about long-term use of
opioids in chronic non-cancer pain?
A. No reliable long term studies (> 8 months) that demonstrate efficacy and safety of long term opioid therapy
for chronic pain
B. Overall relief with opioids is high (75%)
C. Physicians must be liberal when prescribing long term
opioids
D. Outcomes are not patient-specific
E. No risk of adverse events, addiction, diversion, or noncompliance have been noted
2469. With regard to narcotic addiction treatment, which of
the following statements is most accurate?
A. A practitioner who dispenses Schedule II narcotic
drugs for maintenance or detoxification treatment
must obtain a separate registration every five years as a
narcotic treatment program.
B. Registration as a narcotic treatment program allows a
practitioner to administer, dispense, and prescribe,
Schedule II drugs approved by the Food and Drug
Administration for treatment of narcotic addiction.
C. The only Schedule II drugs approved by the Food and
Drug Administration for treatment of narcotic addiction are Methadone and levo-alpha-acetyl-methadol
(LAAM).
D. Registration as a narcotic treatment program is contingent on proper registration with the appropriate state
attorney general.
E. To obtain registration as a narcotic treatment program,
a practitioner must have been engaging in narcotic addiction treatment for at least five years.
2470. Which of the following statements concerning an opioid
treatment program (OTP) is false?
A. An OTP must apply to the Substance Abuse and Mental
Health Services Administration for certification.
B. To become certified by the Substance Abuse and Mental
Health Services Administration (SAMHSA), an OTP
must be accredited by a SAMHSA-approved accreditation body.
Section 12 • Controlled Substance Management
481
in controlled substance prescribing.
C. OTPs must comply with the Federal Law on ConfidenE. State DEA registration and a state medical license.
tiality of Substance Abuse Patient Records and with
the HIPAA privacy rule.
D. OTPs must notify the Substance Abuse and Mental 2475. Which one of the following statements best describes the
mechanism of action of benzodiazepines?
Health Services Administration within sixty days of
A. Benzodiazepines activate GABA B receptors in the
any replacement or change in the status of the prospinal cord
gram sponsor or medical director.
B. Their inhibition of GABA transaminase leads to inE. OTPs must be registered by the Drug Enforcement Adcreased levels of GABA
ministration before administering or dispensing any
C. Benzodiazepines block glutamate receptors in hieraropioid agonist treatment medications.
chical neuronal pathways in the brain
D. They increase the frequency of opening of chloride ion
2471. Regarding the use of opioids, which of the following
channels that are coupled to GABA A receptors
statements is true?
E. They are direct-acting GABA receptor agonists in the
A. Opioid medications have predictable side effects at
CNS
certain doses.
B. Blood level of the drug needed for perceived therapeutic
effect is consistent through the day.
C. If a pain signal is too weak to be perceived, it has no 2476. An 82-year-old woman, otherwise healthy for her age, has
difficulty sleeping. Triazolam (Halcion®) is prescribed
biophysiologic effect.
for her at one-half of the conventional adult dose. The
D. Complete pain relief is the goal of the use of opioid
most likely explanation for the increased sensitivity of
medication.
elderly patients to a single dose of triazolam and other
E. Treating pain aggressively early may decrease the risk of
sedative-hypnotic drugs is
sensitization.
A. Changes in plasma protein binding
B. Decreased renal function
2472. All of the statements are correct about Drug Abuse
C. Increased cerebral blood flow
Prevention and Control Act of 1970 EXCEPT:
D. Decreased hepatic metabolism of lipid-soluble drugs
A. It is Title 21, Chapter 13 of US Code
E. Changes in brain function that accompany the aging
B. Established current schedules, registrations, agencies,
process
enforcement and penalties.
C. There has been little change in laws since that time.
D. Enforcement since inception has changed significantly. 2477. A 40-year-old patient with liver dysfunction is scheduled
for an interventional surgical procedure. Lorazepam (
E. Described schedules of controlled drugs
Ativan®) can be used for sedation in this patient without
concern for excessive CNS depression because the drug A.
2473. A 29-year-old male uses secobarbital to satisfy his
A selective anxiolytic like buspirone
addiction to barbiturates. During the past week, he is
B. Actively secreted in the renal proximal tubule
imprisoned and is not able to obtain the drug. He is
C. Conjugated extrahepatically
brought to the prison medical ward because of the onset
D. Eliminated via the lungs
of severe anxiety, increased sensitivity to light, dizziness,
E. Reversible by administration of naloxone
and generalized tremors. On physical examination, he is
hyperreflexic. Which of the following agents should he be
2478. Addiction is defined as:
given to diminish his withdrawal symptoms?
A. Physical dependence and the need to increase the drug
A. Buspirone
to obtain the same effect.
B. Chloral hydrate
B. A patient who needs the drug and has good control over
C. Chlorpromazine
personal behavior.
D. Diazepam
C. Psychological dependence on the use of controlled subE. Trazodone
stances for their psychic effects and is characterized by
compulsive use.
2474. What are the basic requirements for prescribing
D. Slow but progressive deterioration of health in light of
controlled substances?
drug use.
A. DEA Registration, state DEA registration (where reE. Loss of effectiveness of the drug to control pain.
quired), state medical license allowing the provider
to prescribe controlled substances, and a legitimate
medical purpose within the usual course of profes- 2479. Alkalinization of the urine with sodium bicarbonate is
useful in the treatment of poisoning with
sional practice.
A. Aspirin (acetylsalicylic acid)
B. DEA Registration, state DEA registration (where reB. Amphetamine
quired), and a state medical license allowing the proC. Morphine
vider to prescribe controlled substances.
D. Phencyclidine
C. DEA Registration and a state medical license.
E. Cocaine
D. DEA Registration, a state medical license, and evidence
showing that you have had 10 or more hours training
ASIPP
482
Section 12 • Controlled Substance Management
2480. Which of the following statements regarding controlled
took “too many pain pills”. During this time he becomes
substances prescriptions is FALSE?
extremely lethargic, with slow respirations. No other
A. A controlled substances prescription must be dated as
history is available. Your immediate action, in addition to
of and signed on the day it is issued.
O2 administration is to administer:
B. A controlled substances prescription must contain the
A. Naloxone
full name and address of the patient.
B. Diphenoxylate
C. A physician who has given his or her staff at least eight
C. N-acetyl-L-cysteine
hours of training on the federal laws and regulations
D. Prochlorperazine
concerning controlled substances prescriptions has no
E. Flumazenil
liability for a controlled substance prescription completed by a staff member that does n
2486. The non-medical use of pain medication has been
D. A controlled substances prescription must contain the
followed since 1965. The largest growth of non-medical
name of the drug, the strength of the drug, the dosage
use of pain medication since this time has been between:
form of the drug, quantity of drugs prescribed, and
A. 1986 and 1990
directions for use.
B. 1995 and 2000
E. A prescription for a Schedule III, IV, or V drug given for
C. 2001 and 2002
the purpose of detoxification or maintenance treatD. 1965 and 1972
ment must include the unique identification number
E. 1968 and 1971
issued by the Administrator of DEA in addition to the
physician’s DEA number.
2487. An individual abruptly discontinuing long-term, high
dose use of an opioid drug will likely experience which
2481. The second most common opioid of choice for
one of the following conditions?
intrathecal infusion following morphine is:
A. Opioid-induced hyperalgesia
A. Fentanyl
B. Increased opioid mu receptor affinity
B. Sufentanil
C. Increased opioid tolerance
C. Hydromorphone
D. Loss of physical dependence and addiction
D. Morphine
E. Increased sympathetic nervous system activity
E. Mepiridine
2488. A 16-year old patient has terminal cancer and has failed
all treatment. Pain is worsening and he requires higher
doses of opioid analgesics for pain relief. He inquires as
to whether a research program may or may not help. One
of the side effects with the new treatment is worsening of
peripheral neuropathy. At this point, he refuses further
treatment. His parents want you to talk to him and enroll
him in the experimental protocol. Which of the following
is your next course of action?
A. Inform the patient that he can not refuse treatment
B. Begin treatment if the parents provide written consent
C. Respect the patient’s wishes and cancel plans for treatment
2483. What is the drug of choice to reverse the effect of
D. Avoid further escalation in opioid doses.
Diazepam® overdose
E. Discuss the issues with the patient
A. Naltrexone®
B. Physostigmine®
2489. A substance abuser who decides to abstain checks in
C. Pralidoxime®
to a county detoxification facility and undergoes a 5
D. Flumazenil®
day detoxification program. Assuming the abuser gets
E. Naloxone®
no further treatment or aftercare, his or her chance at
2482. A patient on methadone 90 mg daily, stable, with good
relief now presents with a kidney stone. For the present
problem :
A. Continue at 90 mg of methadone daily
B. Stop methadone
C. Continue 90 mg of methadone, but add higher than
normal doses of hydrocodone
D. Continue 90 mg of methadone and add lower doses of
hydrocodone
E. Continue 90 mg of methadone and add usual doses of
hydrocodone
remaining sober is about:
2484. Once an opioid treatment is selected, titration
A. < 5%
upwards should continue until:
B. 10%
A. A ceiling is reached.
C. 25%
B. Addiction occurs
D. 50%
C. Tolerance occurs
E. 90%
D. A balance between analgesia and side effects is reached.
E. Respiratory depression occurs
2490. Regarding the clinical use of antidepressant drugs,
which one of the following statements is false.
2485. A 25-year-old male receiving hydrocodone and diazepam
A. Patients should be advised not to abruptly discontinue
presents with disorientation. He states that he had
antidepressant medications.
nausea, vomiting, abdominal pain and diarrhea since he
B. In selecting an appropriate drug for treatment of de-
ASIPP
Section 12 • Controlled Substance Management
483
pression, the past history of patient response to spe- 2495. In severe tricyclic antidepressant overdose, it would
cific drugs is a valuable guide
NOT be of value to
C. In the treatment of major depressive disorders, sertraA. Administer lidocaine (to control cardiac arrhythmias)
line is usually more effective than fluoxetine
B. Institute hemodialysis (to hasten drug elimination)
D. MAO inhibitors are sometimes effective in depressions
C. Administer bicarbonate and potassium chloride (to
with attendant anxiety, phobic features, and hypocorrect acidosis and hypokalemia)
chondriasis
D. Provide intravenous diazepam (to control seizures)
E. Weight loss often occurs in patients taking SSRIs
E. Maintain the rhythm of the heart by electrical pacing
2491. In performing urine drug testing, a physician must know
all of the following EXCEPT:
2496. What are some of the signs of abuse of controlled
A. The characteristics of testing procedures, since many
substances by your office workers and co-workers?
drugs are not routinely detected by all UDTs.
A. No extra time is spent near a drug supply
B. Although no aberrant behavior is pathognomonic of
B. Extreme reliability in keep appointments and meeting
abuse or addiction, such behavior should never be
deadlines
ignored.
C. Never volunteer for overtime
C. Reliance on aberrant behavior to trigger a UDT will
D. Only at work when scheduled
miss more than 50% of those individuals using unpreE. Absenteeism, frequent disappearances or long unexscribed or illicit drugs.
plained absences, making improbable excuses and
D. Always prescribe “on-demand” for the patient until you
taking frequent or long trips to the bathroom or to the
are comfortable with the situation.
stockroom where drugs are kept
E. A history of drug abuse does not preclude treatment
with a controlled substance, when indicated, but does 2497. All of the following statements about Food and Drug Act
require a treatment plan with firmly defined boundarare correct, EXCEPT:
ies.
A. 1906 - Wiley Act was founded the F. D. A., and centered
on foods and meat packing
2492. All of the following are signs of controlled substance in
B. 1938 - Food, Drugs, and Cosmetics Act mandated prework place, EXCEPT:
market approval of drugs
A. Progressive deterioration in personal appearance and
C. Marijuana Tax Act 1938
hygiene
D. 1932 - Food, Drugs, and Cosmetics Act required proof
B. Uncharacteristic deterioration of handwriting and
safety
charting
E. 1906 - Wiley Act required concentration standards for
C. Wearing long sleeves when inappropriate
all medications
D. Personality change - mood swings, anxiety, depression,
lack of impulse control, suicidal thoughts or gestures
E. Increased personal and professional activities
Directions: Each question below contains four suggested responses of which one or more is correct. Select
2493. Drug interactions involving antidepressants do NOT
A if
1, 2 and 3 are correct
include
B if
1 and 3 are correct
A. Additive impairment of driving ability in patients takC if
2 and 4 are correct
ing trazodone when ethanol is ingested
D if
4 is correct
B. Behavioral excitation and hypertension in patients takE
if
All (1, 2, 3 and 4) are correct
ing MAO inhibitors with meperidine
C. Elevated plasma levels of lithium if fluoxetine is administered
D. Increased antihypertensive effects of methyldopa when
2498. If a patient who is on tramadol is given a CYP2D6
tricyclics are administered
inhibitor,
E. Prolongation of tricyclic drug half-life in patients with
1. The analgesia of tramadol will decrease
cimetidine
2. The analgesia of tramadol will increased
3. The excretion of tramadol will decrease
2494. Consequences of undertreatment of pain may include all
4. The excretion of tramadol will increase
of the following EXCEPT:
A. Possible jail time
B. Civil lawsuits
C. Loss or restriction of prescribing abilities
D. No effect of medical license
E. Exclusion from Medicare/Medicaid
2499. Medications that should be avoided with grapefruit
include:
1. Those with a low oral bioavailablity
2. Those metabolized by CYP3A4
3. Those with an intestinal transport by p-glycoprotein
4. Those metabolized by CYP2D6
ASIPP
484
2500. Incipient liver failure due to acetaminophen toxicity will
most affect the metabolism of:
1. Lidocaine
2. Methadone
3. Codeine
4. Meperidine
2501. Which of the following is a true statement:
1. Tramadol’s first metabolite has less activity than the
parent compound
2. Heroin is metabolized to morphine
3. Morphine’s metabolites increase in liver failure.
4. Hydrocodone is metabolized to hydromorphone.
2502. Significant drug-food interactions include:
1. Coffee and tea are more rapidly metabolized in the
presence of ciprofloxin.
2. Phenobarbital is useful to treat folate deficiency neuropathy.
3. Smokers have a greater perceived effect of propoxyphene than non- smokers.
4. NSAIDS cause greater renal damage in patients with
high dietary fat intake.
2503. As a pain physician, you are evaluating a new patient
who recently moved to your area. Consistent with your
usual office policy, all new patients get a urine drug test.
The patient’s urine is positive for morphine, however
his medication list shows no listing for any controlled
substances.Which of the following are possible sources
for this finding?
1. Morphine use
2. Heroin use
3. Poppy seeds
4. Codeine
2504. Drugs can be altered by:
1. Absorbsion
2. Distribution
3. Metabolism
4. Excretion
2505. A newly immigrated patient from Viet Nam with
tuberculosis, neuropathy, and acid reflux disease is being
prescribed methadone for pain following spine surgery
to stabilize a fracture. The metabolism of methadone has
been documented to be affected by:
1. Isoniazid
2. Carbamazepine
3. Cimetidine
4. Ethnicity
2506. Drug clearance from the plasma at a constant amount
per unit time and the time for the plasma concentration
of a drug to decrease by one-half, best represent which of
the following drug properties?
1. Pharmacodynamic profile
2. Bioequivalence
3. Drug potency
4. Metabolism and excretion
ASIPP
Section 12 • Controlled Substance Management
2507. What are the guidelines of the Federation of State
Medical Boards (FSMB), adapted by multiple State
Medical Boards?
1. Opioids are used for a legitimate medical purpose
2. Opioids are used for documented abnormalities on
MRI
3. Opioids are used in the course of professional practice
4. Opioids are used if another physician has previously
written prescriptions for controlled substances for the
patient
2508. The following statements are true regarding the
paleospinothalamic tract:
1. The paleospinothalamic tract is poor in opioid receptors.
2. The paleospinothalamic tract connects the thalamus
to the cortex.
3. The paleospinothalamic tract passes impulses from the
2nd order neurons to the C-fibers.
4. The paleospinothalamic tract connects the thalamus
and the reticular activating system.
2509. When choosing an opioid, factors to be considered
include:
1. Patient compliance
2. Dosing schedule of concurrent medications
3. Drug interactions
4. Opioid side effects
2510. If a patient has inadequate relief from an opioid, options
include:
1. Increase the dose
2. Increase the frequency
3. Add a breakthrough medication
4. Change medications
2511. The following are true statements regarding morphine:
1. Morphine is primarily renally metabolized
2. Morphine is primarily renally excreted
3. Morphine is metabolized by CYP2D6
4. Morphine is metabolized by glucuronidation
2512. The choice of opioid medications is influenced by:
1. Frequency of pain
2. Response to prior opioids
3. Daily activity
4. Cost and insurance plan
2513. Choose the correct statements about codeine and
morphine
1. Prescribed morphine cannot account for the presence
of codeine.
2. Codeine metabolizes to morphine.
3. Codeine alone is possible due to a small proportion of
patients who lack the cytochrome P450 2D6 enzyme
necessary to convert codeine to morphine.
4. Morphine metabolizes to codeine
Section 12 • Controlled Substance Management
2514. What are the pitfalls of opioid urine drug testing?
1. Tests for opiates are very responsive for morphine and
codeine.
2. Urine drug tests do not distinguish between morphine
and codeine.
3.
UDT’s show a low sensitivity for semisynthetic/
synthetic opioids such as oxycodone.
4. A negative response excludes oxycodone and methadone use.
485
2. Shorter hospital stays.
3. Less total analgesic use.
4. A greater potential for subsequent opiate dependence
2521. What are Federation of State Medical Boards
Guidelines for the Treatment of Pain?
1. Use of controlled substances, including opiates may be
essential in the treatment of pain
2. Effective pain management is a part of quality medical
practice
3. Patients with a history of substance abuse may require
monitoring, consultation, referral and extra documentation
4. MD’s should not fear disciplinary action for legitimate
medical purposes
2515. What are the correct statements of amphetamine in
urine drug testing?
1. Tests for amphetamine/methamphetamine are highly
cross-reactive.
2. Very predictive for amphetamine/methamphetamine use.
3. UDT will detect other sympathomimetic amines such
2522. Identify the components of the four cornerstones of
as ephedrine and pseudoephedrine
good clinical practice in chronic opioid prescribing.
4. Further testing is NOT required.
1. A specific diagnosis that is opioid responsive
2. Verify no evidence of drug abuse
2516. In order to qualify for a waiver under the Drug Abuse
3. Document improved function
Treatment Act of 2000 to treat opioid addiction with
4. Manage side effects appropriately
scheduled drugs, a physician must meet the following
conditions:
1. Hold a current state medical license and valid DEA 2523. True statements with regards to properties of NMDA
antagonists and therapeutic use and misuse are as
number
follows:
2. Hold an addiction certification from the American
1. Phencyclidine or PCP was developed as an intravenous
Society of Addiction Medicine
anesthetic.
3. Hold a subspecialty board certification in addiction
2.
The
unique anesthesia produced by phencyclidine was
psychiatry from the American Board of Medical Speassociated
with rapid emergence.
cialties
3.
MK-801
(dizocilpine)
was developed as an anticonvul4. Complete at least 8 hours of training in the treatment
sant and subsequently was used as a brain protective
and management of opioid-addicted patients
agent.
4.
Dextromethorphan
is a strong NMDA antagonist, even
2517. Dexmedetomidine and midazolam share the following
when taken in very small amounts.
pharmacologic properties:
1. Provide a continuum of sedation
2. Preserve respiratory function without potentiating 2524. Which of the following statements about alcohol
absorption are valid?
opioid-induced respiratory depression
1. Surgical removal of the pylorus allows more rapid
3. Clearance is decreased in hepatic disease
absorption of alcohol
4. Selective alpha-2 adrenergic agonist activity
2. Most alcohol is absorbed through the gastric mucosa
3. Secretion of gastric mucus induced by high concentra2518. Epidemiologic risks for work-related low back pain
tion of alcohol delays absorption
include:
4.
The longer the alcohol remains in the blood, the gre1. Prior WC claim
taer the effect.
2. Perceived over-education
3. Cumulative compressive back forces
4. Peak hand forces
2525. Mixed opioid agonist-antagonists (nalbuphine,
pentazocin) have limited use in cancer patients because:
1. Respiratory depression is a common side effect
2519. Physical dependence is a term defined as:
2. Mixed interaction at the opioid receptor can precipi1. The presence of withdrawal symptoms with abrupt
tate withdrawal symptoms.
discontinuation of drug.
3. Pruritus is a common side effect.
2. Is synonymous with addiction
4. Effectiveness is limited by a dose-related ceiling effect.
3. Reveals abrupt withdrawal symptoms when antagonist
is administered
4. Does not respond to a tapering dose, and requires 2526. In the management of alcohol withdrawal delirium, the
clinician may wish to use all of the following :
detoxification
1. Chlordiazepoxide
2. Magnesium sulfate
2520. True statements concerning patient controlled analgesia
3. Thiamine
include the following:
4. Intravenous glucose
1. Better patient satisfaction with pain control.
ASIPP
486
2527. Which of the following include subjective reports of
marijuana effects?
1. Increased sexual desire
2. Increased appetite
3. Enhanced tactile sensitivity
4. Increased motivation
Section 12 • Controlled Substance Management
2534. The CSA requires the following of practitioners who
dispense ONLY manufacturer’s samples of controlled
substances to patients.
1. An initial, and then biennial, inventories.
2. A secure locked box for storage of controlled substances.
3. As separate dispensing log, in addition to any records
kept in the patients’ charts.
4. Complete records of all controlled substances received,
dispensed, or otherwise disposed of.
2528. True statements about heroin are as follows:
1. The chemical name is diacetyl morphine
2. Heroin is more water soluble but less potent than
morphine
2535. What are the correct statements about UDT of cocaine?
3. Heroin is metabolized in humans by de-acetylation to
1. Tests for cocaine react principally with cocaine and its
6-mono-acetylmorhpine and morphine
primary metabolite, benzoylecgonine.
4. Heroin is classified as Schedule II drug and is widely
2. Tests for cocaine are non-specific in predicting cocaine
available for therapeutic purposes in the United
use.
States.
3. Tests for cocaine have low cross-reactivity with other
substances
2529. What is the prevalence of co-existing diseases in drug
4. Cold medicines may test false-positive for cocaine
dependence?
1. Alcohol dependence (63%)
2536. Mechanisms of drug interactions include:
2. Any psychiatric disorder (74%)
1. Drug-drug interactions
3. Antisocial personality disorder (44%)
2. Drug allergies
4. Major depression (25%)
3. Drug-food interactions
4. Drug doses
2530. Which of the following statements are true with regards
to the Controlled Substances Act of the Comprehensive 2537. “Joe the Doper” is one of your patients. You are
Drug Abuse Prevention and Control Act of 1970 ?
prescribing oxycodone for a legitimate medical purpose,
1. It is the legal foundation of the government’s fight
and his pain is well controlled. Joe had a routine urine
against the abuse of drugs and other substances.
drug test as part of a federal job interview and the screen
2. It is a consolidation of numerous laws regulating the
reported opiates as “none detected”Joe “found religion”
manufacture and distribution of narcotics, stimulants,
during his last incarceration in jail for cutting the heads
depressants, hallucinogens, anabolic steroids and
off parking meters and swears that he has been taking the
chemicals used in the illicit production of controlled
meds and not selling them on the street. Correct options
substances.
at this time include:
3. All the substances that are regulated under existing
1. Increase his dose of oxycodone and recheck his urine
federal law are placed into I of V schedules.
2. Make a note in the chart that you really believe him and
4. Schedule I is reserved for the least dangerous drugs that
continue the oxycodone
have the highest recognized medical use.
3. Change his pain medication to a fentanyl patch
4. Order GC/MS specifically for oxycodone
2531. A 34-year-old recently married man seeks help from
a methadone clinic. Chose all of the true statements 2538. The following MAY legally be prescribed by a physician
meeting criteria for acceptance into the program:
without a special registration or permission from DEA:
1. He has AIDS
1. Methadone
2. He has no legal charges pending
2. Buprenorphine
3. He has used heroin for 2 years
3. Injectible C-II morphine
4. He has been married to a heroin addict
4. Heroin
2532. Choose all side effects of clonidine (Catapres):
1. Drowsiness
2. Hypotension
3. Dizziness
4. Dry mouth
2539. When changing a patient’s controlled substance
medications, a physician may desire a patient to bring
in all unused supplies of discontinued controlled
substances. Which of the following is true?
1. The physician may refuse to prescribe additional controlled substances until the patient destroys (in a veri2533. If a patient is a “slow metabolizer”, possible responses to
fied manner) or surrenders all unused meds.
medications might include:
2. Because they are the legal property of the patient, the
1. Increased toxicity
physician can do nothing about the patient’s previ2. Decreased effect
ously prescribed and dispensed medications.
3. Increased effect
3. The physician may observe the patient destroy the
4. Decreased excretion
medications by flushing them down the toilet, unless
ASIPP
Section 12 • Controlled Substance Management
487
such disposal is prohibited by state law.
2546. Analgesic agents capable of producing tolerance,
4. The physician may take possession of unused medicadependence and withdrawal include:
tions, inventory them, and send them to the DEA.
1. Codeine
2. Propoxyphene (Darvon)
2540. Urine screening of patients should be able to detect each
3. Buprenorphine
of the following:
4. Pentazocine (Talwin)
1. Cocaine
2. Morphine
3. Alcohol
2547. What is the explanation of a routine urine drug test
4. Barbiturates
in a patient receiving codeine with acetaminophen 240
mg/day testing positive for codeine and negative for
2541. The CSA requires the following of practitioners who
morphine?
administer controlled substances via any modality to
1. The laboratory made a mistake
patients directly, before or during procedures in the
2. The morphine was metabolized faster than the codeine
office or surgical suite, from physician-owned stock or
and was therefore excreted earlier
supplies:
3. The morphine was “neutralized” by the specific carrier
1. An initial, and then biennial, inventories
agent in the Codeine with acetaminophen
2. A secure locked box for storage of controlled sub4. The patient is one of a small number who lack cystances
tochrome P450 2D6 and cannot convert codeine to
3. As separate dispensing log, in addition to any records
morphine
kept in the patients’ charts.
4. Complete records of all controlled substances received,
dispensed, or otherwise disposed of.
2548. Examples of pro-drugs include:
1. Gabapentin
2. Morphine
2542. An intervention on a chemically dependent individual
3. Baclofen
should include all of the following:
4. Codeine
1. A clear message
2. Caring for the patient
3. Planning
2549. When a state has different rules than the CSA:
4. Presence of persons important to the patient
1. Federal constitutional supremacy principles dictate
that the CSA overrules all state regulations relating to
2543. What are some of the communication issues faced by
controlled substances.
health care providers in terminal patients?
2. The issue is decided on a case-by-case basis, by the
1. Diagnosis and prognosis
courts.
2. Advanced directives and do-not-resuscitate(DNR)
3. State rules govern, as the states traditionally regulate
orders
medicine and pharmacy.
3. Spiritual needs
4. Whichever rule is more restrictive must be followed, as
4. Symptom Management
both have full legal force and effect.
2544. What are the identified problems of screening 2550. Choose the accurate statement(s) below regarding
tools of drug abuse?
the Drug Enforcement Administration’s proposed rule
1. Developed in psychiatric field
allowing issuance of multiple prescriptions for Schedule
2. Rely on subjective reporting of the patients
II controlled substances:
3. Not designed to detect prescription opioid abuse
1. A prescription for a Schedule II controlled substance
4. Designed to detect cocaine use
cannot be refilled.
2. The physician must write instructions on each prescription (other than the first prescription, if the
2545. What are true statements about painful crises in sicklephysician intends for that prescription to be filled
cell disease:
immediately) indicating the earliest date on which a
1. The pain is rarely severe.
pharmacy may fill the prescription
2. Abdominal crises are frequent.
3. The physician must conclude that providing the pa3. Joint crises are frequent, accompanied by swelling and
tient with multiple prescriptions in this manner does
discoloration of the affected joint.
not create an undue risk of diversion or abuse
4. Analgesics may frequently have to be administered in
4. Multiple prescriptions may be issued authorizing a
greater than standard doses due to the development
patient to receive a total of up to a 120-day supply of a
of tolerance.
Schedule II controlled substance.
ASIPP
488
Section 12 • Controlled Substance Management
2551. What are the correct statements about urine drug testing 2555. Which of the following are true regarding opioid(UDT)?
induced constipation in a patient with cancer pain?
1. A UDT would be positive if the patient took the drug
1. Impaired defecation reflex
(true positive) and negative if the drug was not taken
2. Increased colonic motility
(true negative).
3. Reduced colonic peristaltic activity
2. Sensitivity of a test is the ability to identify a particular
4. Bulk laxatives most helpful
drug.
3. False-positive or False-negative results can occur, so it 2556. What are the clinical recommendations in chronic
is imperative to interpret the UDT results carefully.
opioid therapy?
4. Specificity is the ability to detect a class of drugs.
1. Daily doses above 180 mg/day of morphine have not
been validated
2552. The following statements are accurate for addiction and
2. Dose escalation beyond the stabilization phase may
dependence.
predict a problem
1. Based on the Controlled Substances Act, the term
3. Opioid rotation may be helpful
“addict” means any individual who habitually uses
4. Drug formulation does not influence tolerance
any narcotic drug so as to endanger the public health
and safety.
2557. What precautions must a physician take in interpretation
2. Based on DSM-IV definition, addiction means malof urine drug testing?
adaptive pattern leading to distress or impairment.
1. Consult with laboratory regarding ANY unexpected
3. DSM-IV definition of substance dependence includes
results.
tolerance, withdrawal, and continued use despite
2. Never use results to strengthen physician-patient relaproblems.
tionship and support positive behavior change.
4. Federation of State Medical Board guidelines for the
3.
Schedule an appointment to discuss abnormal/
treatment of pain recommend use of controlled subunexpected results with the patient; discuss in a
stances in patients with history of substance with no
positive, supportive fashion to enhance readiness to
additional monitoring, referral, or documentation.
change/motivational enhancement therapy (MET)
opportunities.
2553. True statements regarding the five schedules of
4. It is not necessary to document results and interpretacontrolled substances, known as Schedules I, II, III, IV,
tion
and V include all the following:
1. The Schedule I substances have high potential for
abuse and the substance has no currently accepted 2558. If a patient is unable to tolerate oxycodone because of
medical use in the treatment in the United States.
nausea, the likely opioids to be tolerated would be:
2. The Schedule I substances may be changed to a lower
1. Fentanyl
schedule if the safety of the drug is demonstrated even
2. Propoxyphene
though there is a high potential for abuse and there is
3. Methadone
no accepted medical use for medical treatment.
4. Morphine
3. The Schedule II drugs have high potential for abuse
and may lead to severe psychological or physical de- 2559. What is the suggested protocol of Ballantyne and Mao
pendence.
published in New England Journal of Medicine?
4. Schedule V drugs or substances have a high potential
1. Ensure benefit will out weigh risk
for abuse and may lead to physical or psychological
2. Evaluate possible addiction and problems with poor
dependence.
functioning
3. Watch deterioration in function related to lack of mo2554. The following is an accurate statement with regards to
tivation to improve
function of Controlled Substances Act.
4. Once opioids are started no further monitoring is
1. It creates a closed system of distribution for those aurequired
thorized to handle controlled substances.
2. The cornerstone of this system is the licensure of all
those authorized by the State Medical Licensure Board 2560. Pain physicians must consider the following in chronic
to handle controlled substances.
long-term opioid therapy:
3. Only the individuals and practices which dispense di1. Prolonged, high dose therapy may have adverse conrectly to the patients from their clinics are required to
sequences
maintain a DEA license.
2. The opioid formulation does not reduce development
4. It is required to maintain complete inventory of conof tolerance
trolled substances, only if the drugs are administered
3. Abuse potential of long acting and short acting formuby physician, but not if dispensed to the patient.
lations are the same
4. Long term opioids produce adverse physiologic changes (immune, hormonal, pain, etc)
ASIPP
Section 12 • Controlled Substance Management
2561. What are the requirements of continued controlled
substance prescribing?
1. Reduction in pain
2. Improvement in functional status
3. Lack of evidence of drug abuse
4. Lack of unmanageable side effects
2562. Which of the following are true about lorazepam:
1. Has a serum half-life of approximately 12 hours
2. Exhibits linear kinetics
3. Is almost completely converted to benzoylecgonine
4. Is frequently used an anti-anxiety agent
489
to the analgesic effects of the maintenance dose of
methadone.
2. During opioid maintenance treatment, a cross-tolerance develops to all opioid agonist drugs.
3. The usual maintenance dose of opioid maintenance
does not provide any analgesia, and adequate analgesia
will require higher doses of opioid agonists given more
frequently than in the non-tolerant patient.
4. The usual maintenance dose provides significant analgesia, thus, no opioid agonists are required to provide
analgesia for the acute pain.
2568. Which of the following statements about alcohol
withdrawal delirium are correct except ?
1. May be precipitated by surgery
2. Withdrawal seizures are most common 24 hours after
withdrawal
3. Delirium tremens has peak incidence four days after
withdrawal
2564. Which of the following statements about alcoholics are
4. Does not occur while still drinking
correct?
1. Suicidal behavior is common after personal loss
2569. When a patient has been taking heavy doses of
2. High incidence of alcohol abuse is patients who combarbiturates for an extended period, early symptoms of
mit suicide
withdrawal are likely to include:
3. Alcohol tends to worsen depression
1. Weakness
4. Alcoholics who threaten suicide usually do not kill
2. Insomnia
themselves
3. Anxiety
4. Tremulousness
2565. Which of the following is a red flag that prosecutors look
for when deciding whether to prosecute a case involving 2570. In treatment opioid overdose, which of the following is
inappropriate prescribing?
effective?
1. A physician issued prescriptions knowing that the pa1. Methadone
tient was delivering the drugs to others.
2. L-alpha-acetylmethadol (LAAM)
2. The physician prescribed controlled drugs at intervals
3. Buprenorphine
inconsistent with legitimate medical treatment.
4. Naloxone
3. The physician involved used street slang rather than
medical terminology for the drugs prescribed.
2571. Physicians should not fear regulatory action from
4. There was no logical relationship between the drugs
the Board for ordering, prescribing, dispensing or
prescribed and treatment of the condition allegedly
administering controlled substances, including opioid
existing
analgesicsChoose correct statements described in
model policy for the use of controlled substances for the
2566. Substance dependence is best characterized by which of
treatment of pain by Federation of State Medical Boards:
the following statements?
1. For a legitimate medical purpose
1. Substance use is discontinued once there is insight
2. For documented abnormalities/pathology on M.R.I.
about the physical or psychological harm that is likely
Scan
to have been caused or exacerbated by the substance
3. In the course of professional practice
use
4. If another physician has previously written prescrip2. Important social, occupational or recreational activitions for controlled substances for the patient
ties are decreased because of the use of substances.
3. Symptoms may be due to another general medical 2572. Choose the accurate statement(s) below regarding
condition.
the purposes for which a prescription for controlled
4. Tolerance and withdrawal are associated with it.
substance can be issued:
1. A prescription for a controlled substance must be
2567. A patient presents to you with injury of ankle strain.
issued for a legitimate medical purpose in the usual
The ankle is swollen and extremely painful. However,
course of a physician’s practice.
there was no fracture. The patient is also on opioid
2. A prescription for a controlled substance may be issued
maintenance treatment with methadone of 120 mg daily.
to a physician so that he or she can dispense the drugs
True statements with regards to his pain management
to patients as medically necessary.
including the following:
3. Prescriptions for Schedule III, IV, and V drugs are per1. Opioid maintenance patients develop full tolerance
mitted for purposes of detoxification or maintenance
2563. Strategies to reduce aberrant drug behaviors include:
1. Random urine drug screens
2. Narcotic contracts
3. No early refills
4. Opioid rotation
ASIPP
490
Section 12 • Controlled Substance Management
treatment if the drug is specifically approved by the 2579. Analgesia of six hours or longer in duration can be
obtained with all the following drugs:
Food and Drug Administration (FDA) for those uses
1. Levo Dromoran
and the prescribing physician meets the leg
2. MS Contin
4. Prescriptions for Schedule II drugs for detoxification
3. Methadone
or maintenance treatment are permitted.
4. Oxycodone
2573. The transdermal route of fentanyl administration
has been used in cancer patients because it offers the 2580. Choose the correct statements about cocaine testing.
1. A patient’s urine may test positive for the cocaine
following advantages:
metabolite benzoylecgonine after a procedure with
1. Convenience of dosing.
cocaine as a topical anesthetic for up to 3 to 4 weeks.
2. Rapid absorption through the skin allows quick titra2. Cocaine, a topical anesthetic, is clinically used in certion.
tain trauma, dental, ophthalmoscopic, and otolaryn3. Highly potent opioid for analgesic efficacy.
gologic procedures.
4. Low cost.
3. There is structural similarity between other topical anesthetics that end in “caine” (eg, Novocaine, lidocaine)
2574. Methadone blood levels are:
and cocaine or benzoylecgonine.
1. Increased by cimetidine
4. A positive UDT result for the cocaine metabolite, in
2. Decreased by butabutal
the absence of a medical explanation, should be inter3. Increased by Ciprofloxin
preted as due to deliberate use.
4. Decreased by grapefruit juice
2581. What are the reasons for drug testing in patients in your
practice?
1. To assess if the patient is taking the medications prescribed
2. To assess if the patient is taking substances/drugs NOT
prescribed
3. To assess if the patient is taking licit and illicit drugs
2576. In accordance with the Federal Controlled Substances
4. To assess if the prescribed drugs caused diabetes
Act of 1970, which of the following applies towards
regulating the use of narcotic drugs for opioid
detoxification or maintenance?
1. Practitioner is separately registered with the DEA as a 2582. Drug testing may be performed by any of the following
1. Hair samples
narcotic treatment program
2. Saliva testing
2. Practitioner may dispense or prescribe a controlled
3. Serum drug testing
drug in schedules III, IV, or V to a narcotic dependent
4. Urine drug screening
individual for addiction treatment
3. Practitioner in solo practice will not treat more than
30 patients at any one time with scheduled drugs for
2583. Controlled substance abuse in work place may be
detoxification or maintenance
identified by the following signs?
4. A narcotic drug can not be dispensed for a period in
1. Work performance alternating between periods of high
excess of 180 days for the purposes of detoxification
and low productivity and mistakes made due to inattention, poor judgment and bad decisions.
2577. What are the true statements about early history of
2. Confusion, memory loss, and difficulty concentrating
Opium?
or recalling details and instructions. Ordinary tasks
1. Arabia - (600-900 A. D.) used medicinally. When the
require greater effort and consume more time
Koran forbade alcohol, Opium and Hashish became
3. Interpersonal relations with colleagues, staff and pathe primary social drugs.
tients suffer.
2. Galen - 100 A. D. - ‘great cure-all’
4. Promptly admits errors or accepts blame for errors or
3. Greece - mixed with wine 100 B. C.
oversights
4. Eber’s Papyrus, 1500 B. C. - pain relief
2575. Medical Records should include which of the following:
1. Treatment objectives
2. Instructions and agreements
3. Periodic reviews
4. Financial contracts
2578. What are correct statements of Food and Drug Act 2584. True statements about methadone include the
following:
Amendments?
1. It is useful as an analgesic
1. Durham-Humphrey 1951 - make OTCs require pre2. It has greater oral efficacy than morphine
scriptions.
3. It produces a milder but more protracted withdrawal
2. Boggs Act (1951)
syndrome than that associated with morphine
3. Narcotic Control Act (1956)
4. Adverse reactions may include constipation, respira4. Drug Abuse Control Acts of 1956 and 1958 labeled
tory depression, and light headedness
‘potential drugs of abuse’ and gave power to DEA precursors to regulate.
ASIPP
Section 12 • Controlled Substance Management
2585. The following statements are true:
1. All opiates are opioids
2. All opioids are opiates
3. All opioids are narcotics
4. All narcotics are opioids
2586. During cocaine withdrawal, which of the following
symptoms can be anticipated?
1. Cardiac arrhythmias
2. Desire for sleep, often with insomnia
3. Delirium
4. Depression
2587. After chronic amphetamine use, abrupt withdrawal is
likely to cause which of the following symptoms?
1. Seizures
2. Delirium
3. Formication
4. Sleep disturbance
491
2592. What are the correct statements of Harrison Narcotics
Tax Act of 1914?
1. Required physicians to register and keep records of
prescribed medications.
2. Created Bureau of Narcotics of Treasury Department
(and Federally run Heroin Clinics for addicts).
3. 3,000 physician arrests during 1920s.
4. By 1930 “addict pattern” was male, minority, criminal.
But much smaller (?20,000).
2593. What are the steps to avoiding trouble with misuse of
controlled substances?
1. Know your state laws and regulations
2. Know your Medical Licensure Board’s guidelines on
prescribing controlled substances
3. DEA statement about your state on its website
4. Follow the rules of advocacy groups and organization
supporting unrestricted use of controlled substances
2588. Regarding the metabolism of opioids:
1. Some opioids are metabolized by glucuronidation
2. Some opioids are metabolized by the P450 enzyme
system
3. M6G is an analgesic metabolite of morphine
4. M3G is an analgesic metabolite of morphine
2594. Which of the following are true regarding nociceptive
pain?
1. Alpha 2 antagonists are useful for management.
2. Automatic firing of damaged nerves is a component.
3. Deafferentation can produce a firing of first order
neurons.
4. Prostaglandin inhibition may be useful for management.
2589. Patient complains of low back pain, headaches, and
depression, She is taking Lortab (hydrocodone) 10/500
six per day, Fioricet (butalbital) six per day, and Paxil
(paroxetine) 20mg per day. She complains of inadequate
pain relief. Appropriate medication management would
include:
1. Counsel patient on the amount of acetaminophen, and
change her to lower acetaminophen products
2. Start methadone 10mg six per day
3. Change her antidepressant to a super-selective serotonin reuptake inhibitor
4. Change the butalbital to a triptan
2595. Which of the following is true about state medical
regulation:
1. Investigation of physicians is generally the result of
complaints received
2. State Boards may obtain copies of patient records without
patient consent, upon written request, under HIPAA.
3. State regulations generally prohibit any medical act
which is an unreasonable danger to the health, safety
or welfare of patient or public.
4. A physician has a right to hire a lawyer at any stage of a
medical board investigation or proceeding.
2590. Effects on the respiratory system by opioids include:
1. Equipotent doses of opioids result in equal amounts of
respiratory depression
2. Depression of cough is a different mechanism than
respiratory depression
3. There is a direct respiratory depression effect on the
medulla
4. Respiratory rate decreases decrease first and then CO2
and hypoxia response decreases.
2591. What are the correct statements of urine drug testing?
1. Thin-layer chromatography (TLC) is a relatively old
technique, testing the migration of a drug on a plate or
film, which is compared to a known control
2. Gas chromatography (CGMS) is most sensitive and
specific test, most reliable, and labor intensive/costly
3. Enzyme immunoassay is easy to perform/highly sensitive, more sensitive than TLC, and less expensive than
GC/MS
4. Rapid drug screens are not similar to other enzyme immunoassay testsand may be more expensive
2596. A state could, if it chose to do so, do which of the
following:
1. Make ordinary negligence in treatment a basis for
professional discipline, even without any requirement
of intent.
2. Require random periodic audits of physician’s practices, including patient health care records, to ensure
quality of care.
3. Generally outlaw any practice which is not accepted by
the majority of physicians in the relevant specialty.
4. Authorize state medical boards to impose very short
jail terms (less than 30 days).
2597. The ultimate decisions regarding the specific medical
treatment to be rendered to a patient in a specific
situation are made by:
1. The physician
2. The courts
3. The state legislature
4. The patient
ASIPP
492
Section 12 • Controlled Substance Management
2598. What is the level of care - necessary to achieve and
maintain abstinence from opioids?
1. Medically managed inpatient treatment
2. Intensive outpatient program
3. Residential treatment program
4. Medically managed phone consultations
2599. What are the perceived barriers to non-opioid
management?
1. The opioid model words well from a business standpoint
2. Easy to assemble a multidisciplinary team
3. “Rebound pain” phenomenon after detoxification
4. Multidisciplinary model works well from a business
standpoint
“legitimate medical reasons”.
2604. What are the correct statements about controlled
substance abuse?
1. Almost half a ton of prescription narcotics reached six
counties in Eastern Kentucky from 1998-2001, equating to .75 pound for every adult in those counties.
2. On a per capita basis, Eastern Kentucky drugstores,
hospitals, and legal outlets receive more prescription
painkillers than anywhere else in the United States.
3. Nationally, emergency room visits for hydrocodone
overdoses increased 500 percent from 1990-2000
4. OxyContin sells on the street for about $10/pill; Lortab
sell for $2/pill and Lorcet for $1/pill
2600. All of the following statements are correct about
detoxification EXCEPT:
1. In one year post completion outcome data all patients
still had some pain - most had much less
2. In one year post completion outcome data 70% had
achieve continuous sobriety
3. In one year post completion outcome data pain was
not ever worse without narcotics than it was while
taking narcotics
4. In one year post completion outcome data only 20%
had continued sobriety
2605. The Drug Abuse Treatment Act of 2000 allows for which
of the following for detoxification treatment?
1. The use of schedule III drugs to detoxify chemically
dependent patients in an office setting
2. The use of opioids in a substance abuser for legitimate
medical reasons to provide analgesia
3. The use of opioids for detoxification in a nonabuser,
who is opioid dependent from legitimate pain therapy
and desires to discontinue opioid use
4. Practitioners may administer, dispense, or prescribe a
controlled drug from any schedule to a narcotic dependent individual for addiction treatment
2601. Choose the correct statement about state board rules
physicians do not know:
1. Cannot Rx Schedule II or III for family members
2. Can provide samples of unscheduled drugs for family,
but MUST document in a medical record
3. Cannot Rx to anyone (including friends) if you have
not documented their H&P and have a current chart
on file.
4. Can Rx for yourself
2606. Which of the following statements about U.S. enlisted
men who became addicted to opioids in Vietnam are
correct?
1. Nearly 90% did not become addicted again within
three years of return to the United States
2. Relapse more common in older white soldiers
3. Higher relapse rate in sons of alcoholic parents
4. About 75% of soldiers who used heroin five or more
times became drug dependent
2602. Controlled substance is considered a national epidemic
in U.S. What are correct statements showing grim
national statistics?
1. Opioid abuse increased 85% from 1994-2000
2. Oxycodone abuse increased 166% since 1994
3. Hydrocodone abuse increased 116% since 1994
4. Methadone abuse increased 140% since 1994
2607. Factitious disorders consist of the following:
1. Physical or psychiatric symptoms that are intentionally
produced to assume a sick role
2. External incentives are present
3. Most severe form of Münchhausen Syndrome
4. Intentional symptoms with obvious goal
2603. Choose the correct statements describing opioids in
patients with substance abuse?
1. Federal Guidelines allow for use of opioids for analgesia in persons with substance abuse disorder for
“legitimate medical reasons”.
2. No clear documentation of the pain problem is needed
to demonstrate the physician without proper credentials is not attempting to detoxify an opiate abuser.
3. State regulations in certain states do no allow for prescription of opioids in patients with substance abuse
and consider prescribing opioids in known substance
abusers malprescribing.
4. Federal Guidelines do not allow for use of opioids for
analgesia in persons with substance abuse disorder for
ASIPP
2608. Which of the following statements best describes a
characteristic of the antimigraine agent ergotamine?
1. It promotes vasodilatation
2. It is useful in reducing premature contractions of the
uterus
3. It acts as a serotonin (5HT)-receptor antagonist
4. It acts as an alpha-adrenoceptor agonist
2609. Which of the following adverse effects if associated with
the use of Neuroleptic agents ?
1. Acute dystonia
2. Gynecomastia
3. Sedation
4. Loss of libido
493
Section 12 • Controlled Substance Management
2610. Which of the following statements about
biotransformation are true ?
1. Biotransformation often produces metabolites with
less affinity for receptors than the parent drug
2. Biotransformation often produces metabolites with a
higher renal clearance than the parent drug
3. Biotransformation often entails multiple enzyme-catalyzed reactions
4. Biotransformation reactions often occur in the liver
2611. What are common side effects leading to discontinuation
of opioids?
1. Constipation
2. Nausea
3. Somnolence
4. Hyperactivity
2612. Which of the following sedative medications have
analgesic properties:
1. Midazolam®
2. Ketamine®
3. Propofol®
4. Dexmedetomidine®
2613. What are the risks of malprescribing?
1. Legal charges, probably jail time
2. Conviction rate is currently almost 30%
3. Felony conviction will likely prevent or at least severely
limit future practice
4. Duped and Dated are highly viable defenses
2614. Mixed opioid agonist-antagonists (nalbuphine,
pentazocine) have limited use in cancer patients because:
1. Respiratory depression is a common side effect
2. Interaction at the opioid receptor can precipitate withdrawal symptoms.
3. Pruritus is a common side effect.
4. Effectiveness is limited by a dose-related ceiling effect.
2615. Evidence based medicine was developed in response to
public need. What are the accurate statements?
1. Patients and payers call for more accountability
2. Changing patient-physician relationship with financial
focus is positive
3. Concern about increasing costs
4. Clinicians (and patients) are good at decision making
2616. Which of the following may cause constipation in the
cancer patient?
1. Chronic opioid use for pain.
2. Iron supplementation for anemia.
3. Antacids containing Ca and Al.
4. Drugs with anticholinergic effects.
2617. A COX 2 selective agent may be preferred in the
following:
1. History of GI bleed or complicated ulcer
2. Anticoagulant use
3. Oral corticosteroid use
4. Age < 60
2618. The scope of the problem of prescription opioids among
physicians is as follows:
1. Up to 20% of prescribed opioids are diverted.
2. 567 physician arrests and sanctions in 2002.
3. No more federal diversion program for malprescribing
physicians.
4. State boards may not take any actions.
2619. Goals of pharmacotherapy in opioid addiction include
the following:
1. Prevention or reduction of withdrawal symptoms
2. Prevention or reduction of drug craving
3. Restoration to or toward normalcy of any physiologic
function disrupted by chronic drug use.
4. To provide addictive drugs to prevent relapse
2620. What are substance induced disorders?
1. Delirium-intoxication or withdrawal
2. Dementia
3. Amnestic disorders
4. Psychotic disorders
2621. Prescriptions for pain relief are receiving special
attention by the Drug Enforcement Agency and the Office
of Inspector General due to . . .
1. The significant increase in the types of pain prescriptions available today.
2. The significant increase in the number of pain medications prescribed today.
3. Evidence of doctor shopping by persons who obtain
pain prescriptions from doctors either for their own
abusive use or for illegal resale to others.
4. A lessening of illegal drug trafficking.
2622. If urine tampering is suspected, the following should be
included in urine drug testing (UDT)
1. Temperature
2. Creatinine
3. PH
4. Color
2623. What are the true statements about marijuana urine
drug testing (UDT)?
1. UDTs provide reasonable reliability
2. Marinol tests positive
3. Protonix may test false-positive
4. Marijuana may be positive 2 years after use.
2624. The “Whizzinator” is which of the following:
1. An electromechanical device for stirring alcoholic
drinks
2. A urologic testing device to measure the force of urine
flow
3. A suction device for maintaining an erection
4. A commercially available device to thwart urine drug
testing
ASIPP
494
Section 12 • Controlled Substance Management
2625. The pain management physician is challenged to find 2631. A prominent pain physician in North Carolina is treating
non-traditional approaches for pain management.
Ms. Lavonia Gotrocks from Indiana. Traveling to see this
Effective treatments to be considered in a multimodality
physician greatly interferes with her social calendar, and
approach include
she does not wish to make the trips more than every
1. Myofascial relief and musculoskeletal treatment stratethree months. She is currently taking sustained release
gies.
oxycodone and hydromorphone.The physician writes her
2. Psychological drug therapy.
prescriptions on September 1, and then writes additional
3. Chiropractic care.
prescriptions for the same medications and indicates that
4. Naturopathic medicine.
they should be filled on October1, and November1. Her
next appointment in the office is December 1.Which of
2626. The action of Tramadol is considered:
the following responses would include the following
1. A centrally acting synthetic opioid analgesic.
1. This is correctly called “alternate dating” and not cov2. Associated with mu-opioid receptor activity.
ered by the “No Refill” rule
3. Inhibition of reuptake of norepinephrine and sero2. Is a recurring tactic among physicians who seek to
tonin.
avoid detection when dispensing controlled substanc4. Tramadol induced analgesia is totally reversed by
es for unlawful (nonmedical) purposes
naloxone.
3. Is legal if done for no more than 3 months total (3
prescriptions)
2627. The Federation of State Medical Boards (FSMB)
4. Is tantamount to writing a prescription authorizing
encourage practitioners to include the following in the
refills of a schedule II controlled substance
medical record :
1. Medical history
2. Pain history
2632. What are D.E.A considered “certain recurring
3. Working diagnosis
concomitance of condemned behavior” in physician
4. History of allergies to opioids
conviction?
1. The physician involved used street slang rather than
2628. Identify the “federal five” drugs or drug classes that must
medical terminology for the drugs prescribed
be tested for in federal employees and federally regulated
2. The physician warned the patient to fill prescriptions at
industries?
different drug stores
1. Marijuana
3. There was no logical relationship between the drugs
2. Cocaine
prescribed and treatment of the condition allegedly
3. Amphetamine/Methamphetamine
existing
4. Methadone
4. The physician issued prescriptions to a patient known
to be delivering the drugs to others
2629. Urine Drug Testing (UDT) may be useful in which of the
following situations?
2633. An opiate overdose patient may present with all of the
1. To determine if a patient is taking the controlled subfollowing:
stances prescribed
1. Increase in respiratory rate
2. To determine the patient’s state of hydration in an ef2. Small pupils
fort to regulate the dosage of medication
3. Hypertension
3. To determine if the patient is taking medications and
4. Coma
substances which are not prescribed by the physician
administering the test
4. To determine the half-life of the drugs prescribed
2634. Based on systematic reviews, what are the correct
statements about opioids?
2630. If a physician is aware that a patient is a drug addict or
1. Morphine and oxycodone are not equal
has resold prescription narcotics, which of the following
2. Morphine 20 mg/day and oxycodone 30 mg/day are
is correct according to the “Interim Policy”
equally ineffective .
1. It is merely recommended that the physician engage in
3. Improved functional status was conclusive
additional monitoring of the patient’s use of narcotics
4. Opioids did not improve depression
2. The physician, as a D.E.A. registrant, has a responsibility to exercise a much greater degree of oversight to
prevent diversion
2635. What are the characteristics of a drug dependent (addict)
3. Should prescribe controlled substances for intervals of
practitioner?
1-2 weeks only with frequent urine drug screens
1. Starts by taking controlled drug samples
4. May not dispense controlled substances with the
2. Never asks staff to pick up medications in their names
knowledge they will be used to support addiction or
3. Calls in scripts in names of family members or fictibe resold
tious patients and picks them up himself
4. Never uses another doctor’s DEA number
ASIPP
Section 12 • Controlled Substance Management
495
2636. What are the true statements of drug abuse in the United 2642. DSM-IV definition of substance dependence includes
some of the following items as part of at least 3 in 12
States?
months.
1. In lifetime, 46% of persons aged 12 and older used
1. Larger amounts/longer periods
drugs
2. Patient spends large amount of time using/obtaining
2. In past month, 8.3% of persons aged 12 and older used
controlled substances
drugs
3. Activities given up: social/work/recreation
3. In past year, 14.9% of persons aged 12 and older used
4. Continued use despite problems
drugs
4. Prescription opioids are not abused
2643. What are the clinical implications of non-responsiveness
to opioids in chronic pain? Choose the correct
2637. Which of the following statements about
statements.
biotransformation reactions are true?
1. Difficult to distinguish pharmacologic tolerance from
1. They may introduce an active center for further conopioid-induced abnormal pain sensitivity
jugations
2. Treating increasing pain with increasing doses may be
2. They almost always yield water-soluble metabolites
futile
3. They are often important in activating pro-drugs
3. High dose therapy may have adverse consequences
4. They are all inducible upon repeated drug administra4. Push the dose to highest level or combine 2 opioids
tion
2638. Which one of the following results may occur from 2644. What are the advantages of prolonged, high-dose opioid
therapy?
repeated administration of a drug?
1. No evidence to support high dose therapy (>200 to 300
1. Increased metabolism of the drug
mg/day or more)
2. Increased metabolism of other drugs
2. Opioid doses should not be limited in the name of
3. Induction of cytochrome P-450 or glucuronyl transimproving efficacy and safety
ferase
3. Anecdotal evidence that pain relief not better
4. Increased metabolism of endogenous compounds
4. There is solid evidence that 3600 mg of oxycodone per
day is effective in neuropathic pain
2639. What is the definition of addiction as per the Controlled
Substances Act?
1. The term “addict” means any individual who habitu- 2645. What were the physician opioids of choice in 2002?
1. Hydrocodone 40%
ally uses any narcotic drug so as to endanger the public
2. Oxycodone 25%
morals, health, safety, or welfare
3. Ultram 70%
2. “Addict” is a patient who is taking as per prescription
4. Dilaudid 25%
very high opioids in cancer pain
3. Any individual who is so far addicted to the use of
narcotic drugs as to have lost the power of self-control
2646. Symptoms of opioid toxicity include which of the
with reference to his addiction.
following?
4. “Addict” is a patient taking controlled prescription
1. Leg muscle twitching
drugs
2. Pulmonary edema
3. Seizures
2640. Definitions of addiction include the following:
4. Hypothermia
1. A primary, chronic neurobiologic disease with genetic,
psychosocial and environmental factors effecting its
course and presentation
2. Characterized by either impaired control of drug use 2647. What were the physician opioids of choice in the 1990s?
1. Oxycodone 20%
or other symptoms
2. Hydrocodone 70%
3. Addiction involves loss of control, craving, compulsive
3. Dilaudid 20%
use, and continued use despite consequences
4. Fentanyl class 10%
4. DSM-IV definition defines addiction to involve impaired control of drug use
2648. A hypertensive crisis is most likely to result from the
2641. Morphine is used therapeutically
action of drugs from which one of the following drug
1. To suppress the withdrawal syndrome associated with
classes:
the chronic use of alcohol
1. Tricyclic antidepressants
2. To induce miosis
2. Barbiturates
3. To treat severe constipation
3. Opioids
4. To relieve pain associated with a heart attack
4. Monoamine oxidase (MAO) inhibitors
ASIPP
496
2649. Which of the following effects if produced by tricyclic
antidepressant drugs?
1. Increase in the antihypertensive effect of guanethidine
2. Hypertensive crisis
3. Increased absorption of an oral dose of levodopa
4. Precipitation of narrow-angle glaucoma
2650. Which of the following effects is associated with
benzodiazepines?
1. Paradoxical excitement
2. Ataxia
3. Sedation
4. Amnesia
2651. Which of the following effects is produced by
morphine?
1. Relief of dyspnea accompanying pulmonary edema
2. Decreased sensitivity of the respiratory center to carbon dioxide
3. Miosis that can be blocked by atropine
4. Vasodilation of cerebral blood vessels
2652. Which of the following drugs will increase blood levels
of oxycodone?
1. Fluoxetine (Prozac®)
2. Sertraline (Zoloft®)
3. Paroxetine (Paxil®)
4. Carbamazepine (Tegretol®)
Section 12 • Controlled Substance Management
2. Only aware of a few treatments or medications
3. Prescribes for friends or family without a patient
record
4. Well aware of controlled drug categories
2658. The most significant increase in prescriptive medication
for illicit use is:
1. THC (tetrahydrocannabinol)
2. Cocaine
3. Benzodiazepines
4. Pain relievers
2659. What are the pitfalls of prescription practices?
1. 4 D’s - Deficient, Duped, Deliberate, Dependent Practitioner
2. Never say “NO” - Family, Friends, Patients
3. Ignore complaints
4. Focus on positive aspects of regulations and reimbursement
2660. Which of the following statements concerning
barbiturate is true?
1. Barbiturates can increase bleeding time when administered to patient taking anticoagulants
2. Patients tolerant to the therapeutic actions of barbiturates are also tolerant to the analgesic effect of
morphine
3. Barbiturates are used to prevent withdrawal symptoms
2653. DSM-IV definition of substance abuse includes at least
associated with heroin dependence
one of the following in 12 months:
4. Barbiturates are contraindicated in patients with acute
1. Maladaptive pattern leading to distress or impairment
intermittent porphyria
2. Recurrent failure to fill role
3. Recurrent physically hazardous behavior
2661. What are the risks of malprescribing related to practice
4. Recurrent legal problems
management?
1. Loss of “Provider Status”
2654. Which of the following have been identified as algogenic
2. Insurers frequently report to Boards
substances?
3. Plans may remove providers for “overprescribing”
1. Serotonin
4. Insurers are unable to report any type of national data2. Leukotrienes
bank for malprescribing
3. Acetylcholine
4. Histamine
2662. Symptoms of withdrawal from opioids include the
following symptoms and signs:
2655. Opioids recommended for lactating patients include
1. Sweating
1. Morphine
2. Restlessness
2. Hydromorphone
3. Irritability
3. Hydrocodone
4. Hot/cold flashes
4. Meperidine
2663. What were the physician drugs of abuse in 2001?
2656. Identify accurate statements of open label studies of
1. Opioids 30%
opioids
2. Alcohol 20% to 30%
1. Less then 50% of patients were continued on opioids
3. Benzodiazepines 20%
at 2yeras
4. Marijuana 2%
2. Most of patients experiences at least one adverse event
3. No firm conclusions were made about tolerance and 2664. A 16-year-old boy is brought for emergency evaluation
addiction
after taking some of his mother’s medication in order to
4. Results were made applicable to general public
get “high.” He is flushed and his pupils are dilated and
only poorly reactive. He complains of dry mouth. He
2657. What are the correct statements about a deficient (dated
is restless, confused at times, and may be having visual
practitioner)?
hallucinations. Which of the following medications is he
1. Too busy to keep up with CME
likely to have taken?
ASIPP
497
Section 12 • Controlled Substance Management
1.
2.
3.
4.
Phenelzine
Disulfiram
Alprazolam
Benztropine
3. Increased blood pressure
4. Diarrhea
2671. In the management of detoxified substance-abusing
patients in a therapeutic community, poor prognosis is
more likely with:
2665. Chose the correct statements about use of opium in
1. Coexisting severe psychopathology
Europe and America:
2. Dropout before three months
1. Paracelsus 1500 called laudanum (opium, cloves, and
3. Continued alcohol use
alcohol) the “stone of immortality”.
4. Adjunctive use of antidepressants
2. Thomas Sydenham brought to England about 1700.
3. 1831 Sertuener given Nobel Prize for isolating indi2672. Withdrawal convulsions are likely to occur in patients
vidual opiates.
who have used chronically which of the following drugs?
4. 1853 - hypodermic syringe invented. The American
1. Secobarbital
Civil War and Prussian Wars of 1860-1870 led to wide2. Desipramine
spread morphine injection.
3. Lorazepam
4. Phencyclidine
2666. What are the historical aspects of opiate abuse?
1. “Soldier’s Disease” - morphine addiction - reported
during American Civil War and Prussian Wars of 2673. A 42 year old physician who suffers from ankylosing
spondylosis is referred for pain management. After 2
1860-1870.
months of treatment with long-acting opioids he feels
2. 10% of U. S. population used laudanum nonmediciso much better that he plans to return to work as an
nally.
emergency room physician. Which of the following
3. Widespread laudanum and opiate abuse in U. S. in late
would be appropriate actions:
1800s. Sears carried syringe kits in catalogs. Estimated
1. Recommend he have neuropsychologic testing to doc500,000 - 1,000,000 addicted in U. S.
ument good mental function while taking opioids.
4. Heroin introduced in 1898 as “non-addicting” mor2. Write a statement that he is capable of returning to the
phine.
practice of emergency medicine at this time.
3. Insist the physician inform his employer and the hos2667. True statements regarding the Controlled Substances
pital of the medications he takes as a requirement to
Act in determining the control or removal from schedules
continue treatment.
of substances include the following:
4.
Inform
him that legally he cannot return to practicing
1. Drugs actual or relative potential for abuse.
medicine while taking opioids.
2. Scientific evidence of its pharmacological effect, if
known.
2674. A significant minority of persons with substance abuse
3. Any risks to the public health
disorder have and atypical course and will eventually
4. Guarantee by the manufacturer that it will be provided
either stop using or be able to return to controlled use.
at affordable price to public.
Which of the following factors are associated with being
able to stop or control use?
2668. When a person taking a medication or abusing a drug
1. Stable premorbid personality
develops tolerance, which of the following statements are
2. Developing medical complications of substance abuse
valid?
3. Age greater than 40
1. The same dosage of the drug has reduced effect
4. Arrest and incarceration
2. Tolerance develops uniformly to all effects of the drug
3. Physical dependence tends to develop in parallel with
2675. What are the characteristics of simple chronic pain?
tolerance
1. Pain lasting longer than 6 months.
4. Withdrawal symptoms are less likely after tolerance
2. Tend to have no more distress or psychopathology than
has developed
what is expected in the general population.
3. Tend to continue working.
2669. Which of the following statements are correct about
4. Tend to maintain meaningful relationships
heroin abuse?
1. The peak incidence is age 25 to 35
2676. Choose the statements reflecting prescription drug use
2. It affects men three times as often as women
for non-medical purposes.
3. The majority of heroin abusers are involved in main1. Overall, it is believed that 10% of prescription drugs
tenance programs
are used for non-medial purposes.
4. Most heroin abusers eventually stop on their own
2. Controlled prescription drugs for non-medical purposes have been reported to be used over 6 million
2670. Symptoms of heroin withdrawal include all of the
people over the age of 12 years in the United States.
following :
3. Prescription drug abuse and illicit drug abuse may be
1. Twitching movements in the legs
significantly higher in chronic pain patients than in
2. Dilated pupils
ASIPP
498
Section 12 • Controlled Substance Management
normal population.
4. Marijuana use is only second to cocaine.
3. Prescription opiate abuse increases in late 1990s (?or
increased awareness?)
4. Heroin resurgence begins late 1880’s.
2677. An otherwise healthy 45 year old patient taking 100 mg
of morphine per day for pain associated with rheumatoid 2683. Drugs able to diminish the dose of opioids analgesics
arthritis has decided to stop his pain medication 3 days
required to relieve pain include the following:
prior to seeing you in the office. Which of the following
1. Amphetamines
symptoms indicate severe withdrawal that probably
2. Tricyclic antidepressants
requires treatment:
3. Non-steroidal anti-inflammatory agents
1. Fever
4. Acetaminophen (Tylenol)
2. Headache
3. Hypertension
2684. Identify reasons for a standard order for Meperidine 75
4. Rhinorrhea
mg every four hours to be inappropriate?
1. Effective analgesia lasts only 2.5 to 3 hours.
2678. A patient with esophageal cancer has been taking
2. If a person is also receiving a monoamine oxidase inhydrocodone 10mg 2 TID by mouth with good relief.
hibitor, severe toxicity can occur.
However, he is admitted to the hospital with esophagitis
3. That dose is equivalent only to 5 to 7.5 mg of morphine.
from radiation and is not able to tolerate any medications
4. In the presence of impaired renal function toxicity may
by mouth. Options for pain management include:
occur.
1. 1 mg/hr morphine IV continuously
2. 50 mg meperidine IM every 6 hours
2685. What are true statements about non-opiate drugs of
3. 0.5mg hydromorphone IV every 4 hours
abuse?
4. 25 mcg/hr Fentanyl transdermally
1. Coca products began widespread use in 1880s in patent
medicines and ‘soft drinks’. Endorsed by the Surgeon
2679. Those that drink alcohol on a regular basis are
General in 1886.
recognized to have increased risk when mixed with
2. Amphetamines 1920s, used by military, physicians,
controlled substances, particularly sedatives, opioids and
widespread public use in 30s.
benzodiazepines. Which is true of those that consume
3. Barbiturates and amphetamines began widespread use
alcohol, and placed at risk in this population?
in 1940s, originally as OTC medications and in patent
1. 120 million Americans use alcohol regularly.
medications.
2. 20% of the American population consider themselves
4. Hallucinogens popularized in 1960s. Declined by late
alcohol users.
1970s.
3. 16.1 million are heavy, or regular drinkers.
4. 1% of Americans have participated in a binge drinking 2686. Intravenous heroin use causes or is associated with all of
episode one month prior to the survey.
the following:
1. Affective disorder
2. Liver disease
2680. In obtaining a history during the evaluation of a new
3. Endocarditis
patient, which of the following would be considered risk
4. Gall bladder disease
factors for possible substance abuse?
1. History of substance abuse in a brother and father.
2687. Upon abrupt discontinuation of L-alpha-acetyl2. History of psychiatric problems in the mother
methadol (LAAM) in a tolerant patient, one may observe:
3. History of bipolar illness in the patient
1. Decreased respiratory response to increased carbon
4. Age over 40 years old
dioxide.
2. Withdrawal symptoms, which may not appear for up
to 3 days.
2681. According to the ASAM, APS, AAPM consensus statement
3. Vomiting, diarrhea, hypertension, skeletal muscular
on the use of opioids for the treatment of chronic pain,
twitching as severe signs.
which of the following characterize addiction?
4. Few symptoms until 72 hours have passed.
1. Impaired control over drug use
2. Continued use despite harm
2688. Highly tolerant opioid users, maintained on their drug
3. Craving
in a research setting, will:
4. Compulsive drug use
1. Continue to feel a “rush” when their drug of choice is
administered intravenously.
2682. Choose the correct statement of U.S. opiate abuse
2. Continue to use their drug of choice for reasons other
patterns:
than fear of experiencing withdrawal.
1. Change from morphine to heroin in 50s-60s. Dramatic
3. Continue to experience pleasurable effects from food,
increase in numbers 60s-70s.
sex, tobacco and other non-opiate drugs.
2. Reduction in opiate abuse during 80s and early 90s
4. Will not use any other drug for medical or recreational
(?cocaine effect?)
purposes
ASIPP
Section 12 • Controlled Substance Management
499
2689. The accurate statements about prescription drug abuse
4. Heavy chronic use can lower serum testosterone levels
for non-medical purposes include the following:
in men
1. Prescription drug abuse, particularly of opioid pain
killers, has increased at an alarming rate over the last 2695. Chronic alcoholism is associated with:
ten years.
1. Retrobulbar optic neuropathy
2. Non-medical use of narcotic pain relievers, tranquil2. Caudate calcification
izers, stimulants, and sedatives ranks second (behind
3. Cerebellar anterior lobe degeneration
marijuana) as a category of illicit drug abuse among
4. Acoustic neuroma
adults and youth.
3. In 2003, 6.3 million Americans were current abusers 2696. 30 mg of Morphine Sulphate orally is equivalent to:
of prescription drugs, with 4.7 million using pain
1. 10mg MSO4 IV
relievers.
2. 20mg of oral oxycodone
4. E.R. visits for Benzodiazepine abuse in 2002 were
3. 1.5mg hydromorphone IV
100,000 and for 0pioid abuse in 2002 were 119,000
4. 20mg methadone
2690. Which of the following are principles of medical ethics?
1. Justice
2. Beneficence
3. Autonomy
4. Non-negligence
2697. Delirium is an acute confusional state that results from
diffuse organic brain dysfunction. In the cancer patient,
the causes include:
1. Opioid toxicity
2. Dehydration
3. Hypoxia
2691. A 29-year old patient whom you had been treating
4. Brain metastases
for postlaminectomy syndrome with only time release
morphine, 120 mg per day, had a urine drug screen 2698. As per the Controlled Substances Act, a denial,
positive for cocaine and benzodiazepines. When this
revocation, or suspension of registration may be carried
was discussed with the patient, he admitted getting
out based on the following grounds.
medications from a number of physicians and to
1. The physician has failed to inform change of his office
injecting cocaine intravenously, but refused evaluation
address.
by an addictionologist and was dismissed from your
2. Has been convicted of a felony.
practice. 3 days later an emergency room physician calls
3. The physician has reached an agreement with the State
because the patient has been admitted following his first
Medical Board of Licensure.
ever grand mal seizure. Which of the following are likely
4. Has had his state license or registration suspended,
causes of the seizure?
revoked, or denied.
1. Cocaine overdose
2. Opiate withdrawal
2699. No Schedule II prescription drug may be dispensed
3. Benzodiazepine withdrawal
without a written prescription, unless:
4. Cocaine withdrawal
1. A physician calls in a refill.
2. It is dispensed directly by the physician to the ultimate
2692. Which of the following is NOT true regarding penalties
user.
for an unlawful intentional or knowing distribution or
3. The drug is dispensed to another physician.
dispensation of controlled substances?
4. There is an emergency situation which is defined by
1. As little as a one-year sentence may be imposed.
regulation; in that case, an oral prescription may be
2. A life sentence may be imposed.
allowed.
3. Supervised release will be imposed and will last from
1-5 years.
2700. For purposes of ordering a Schedule II emergency
4. The court has the power to suspend a sentence or grant
oral prescription, an emergency situation exists if the
probation or parole, no matter what the facts of the
prescribing physician determines . . .:
case are.
1. That immediate administration of the controlled
substance is necessary for proper treatment of the
2693. Sequelae of an acute cocaine overdose include
intended ultimate user.
1. Myocardial ischemia and high output cardiac failure
2. That no appropriate alternative treatment is available,
2. Seizure activity
including administration of a drug which is not a con3. Tremulousness and hyperthermia
trolled substance under schedule II.
4. Blockade of inhibition of epinephrine
3. That it is not reasonably possible for the prescribing
physician to provide a written prescription to be pre2694. The characteristics of marijuana include:
sented to the person dispensing the substance, prior to
1. It may lower intraocular pressure
the dispensing.
2. A sign of acute intoxication is reddening of conjunc4. That the patient complains of extreme and unremittiva
ting pain.
3. It has antiemetic properties
ASIPP
500
Section 12 • Controlled Substance Management
2701. Identify the suggested requirements to be included in 2706. Findings of illicit drug use based on insurance in
an agreement, if the patient is at high risk for medication
interventional pain management setting was:
abuse or has a history of substance abuse:
1. Third party - 17%
1. Reasons for which drug therapy may be discontinued
2. Medicare with/without third party - 10%
(e.g., violation of agreement)
3. Medicare/Medicaid - 24%
2. Requirement for medical interview with members of
4. Medicaid - 39%
immediate family
3. Urine/serum medication levels screening when re- 2707. All of the following are symptoms of alcohol withdrawal
quested
:
4. Periodic reports from a local law enforcement agency
1. Coarse tremor of hands or tongue
2. Generalized tonic-clonic seizures
2702. A fax will serve as the original prescription for a Schedule
3. Tachycardia, sweating, dilated pupils
II narcotic substance . . .:
4. Abducent nerve paresis or paralysis
1. That is to be compounded for direct administration to
the patient by parenteral, IV, intramuscular injection, 2708. An opioid treatment program must provide which of the
subcutaneous or intraspinal infusion.
following?
2. For a resident of a long-term care facility.
1. Drug abuse testing services
3. For a patient enrolled in a hospice program licensed
2. Vocational services
by the state or certified and/or paid for by Medicare,
3. Educational services
if the prescription notes that the patient is a hospice
4. Medical services
patient.
4. For any terminally ill patient.
2709. Somatoform disorders include the following:
1. Physical symptoms suggestive but not fully explained
2703. Which of the following is true with respect to a partial
by a general medical disorder
filling of a Schedule II prescription?
2. Includes: somatization disorder
1. If the remaining portion of the prescription is not or
3. Includes: conversion disorder
cannot be filled within 72 hours of the partial filling,
4. Includes: major depression
the pharmacist must notify the prescribing physician.
2. A partial filling is allowed for a terminally ill patient, 2710. All states that have guidelines, laws, and/or regulations
and the prescribing physician has the sole responsigoverning the use of controlled substances to treat pain
bility to make sure the controlled substance is for a
set, as a minimum standard, that the physician should ask
terminally ill patient.
the patient whether he/she has
3. A partial filling is allowed for a terminally ill patient,
1. Ever used prescribed controlled drugs before.
and the prescribing physician and the pharmacist both
2. Ever been to a pain doctor before.
have the responsibility to make sure the controlled
3. A history of chemical/substance abuse, including alcosubstance is for a terminally ill patient.
hol, illicit, and licit drugs.
4. A partial filling is allowed for a terminally ill patient,
4. Had any tests related to his/her pain condition.
and the pharmacist has the sole responsibility to make
sure the controlled substance is for a terminally ill 2711. What is the impact of psychological factors in treatment
patient.
of pain with a comorbid substance use disorder?
1. Impedes diagnosis and complicates interventions
2704. Partial filling of a prescription for a controlled substance
2. Pain can mask addiction - switch to legal drugs
listed in Schedule III, IV, or V is permissible, if:
3. Promotes regression and may induce hyperalgesia and
1. Each partial filling is recorded in the same manner as
extreme tolerance
a filling.
4. Positive impact as psychotherapeutic drugs provide
2. The total quantity dispensed in all partial fillings does
excellent analgesia
not exceed the total quantity prescribed.
3. No dispensing occurs after six months after the date on 2712. What are the Methadone interactions?
which the prescription was issued.
1. Cimetidine will increase methadone levels
4. The prescribing practitioner authorizes the partial fill2. Butalbital will decrease methadone levels
ing in writing.
3. Ciprofloxin will increase methadone levels
4. Phenytoin will decrease methadone levels
2705. Cocaine abuse is likely to produce symptoms similar to
which of the following?
1. Major depression
2. Obsessive compulsive disorder
3. Generalized anxiety disorder
4. Paranoid schizophrenia
ASIPP
Section 12 • Controlled Substance Management
501
2713. A former drug addict is admitted to the psychiatric ward 2716. Drug testing may be performed by utilizing any of the
following technique(s):
with depression secondary to chronic back pain. He has
1. Urine drug screening
been taking tramadol 6 tablets per day. He was started on
2. Hair samples
paroxetine and you were consulted for pain management.
3. Saliva testing
He complains that the tramadol is not working, and he is
4. Specific drug analysis of blood
noted to become more agitated. Your assessment is:
1. He is drug seeking
2717. After starting a hospital patient on a morphine PCA, you
2. He is withdrawing from illicit medications
get a call from the nurse that the patient’s face is itching.
3. His depression is making his pain worse
You tell the nurse that:
4. He is on the inappropriate antidepressant
1. This is an allergic reaction, and to stop the medicine
2. This is a histamine release and will likely go away
2714. True statements regarding the Controlled Substances
3. The chart should be marked “allergic to all opioids”
Act (CSA) include which of the following?
4. Changing the PCA to hydromorphone will likely stop
1. Methadone is a DEA Schedule II controlled substance
the itching
that is indicated for the relief of severe pain and detoxification or maintenance of narcotic addiction.
2. A prescription may not be issued for the dispensing 2718. “Opioid rotation”
1. May work because of variable affinity for the µ recepof methadone for detoxification treatment or maintetor
nance treatment
2. May work because of differing opioid metabolic path3. To use Schedule II opiates for detoxification from opiways
ate addiction, a special registration is required.
3. May work because of avoidance of metabolite antago4. Drug Abuse Treatment Act of 2000 allows physicians
nists
to use Schedule III agents to detoxify chemical depen4. May work because of differing drug interactions
dent patients in an office setting, provided the physician qualifies for and obtains a waiver issued through
2719. Opioids exert their action by:
DEA.
1. Inhibiting the release of substance P
2. Activating G proteins
2715. What are the true statements describing history of
3. Inhibiting adenylate cyclase
spread of opium?
4. Activating dopaminergic neurons
1. Arabic traders spread use and cultivation to China by
900 A. D.
2. 1644 China outlawed tobacco, opium smoking became 2720. What are the requirements according to the FSMB Policy
Guidelines, to prescribe controlled substances within the
endemic
course of professional practice?
3. 1700 British East India company smuggled opium
1. A physician-patient relationship
from India to China for tea (China refused legitimate
2. Diagnosis and documentation of unrelieved pain
trade).
3. Compliance with applicable state or federal law
4. 1839 - Because of “rampant addiction” Chinese em4. Urine drug testing (UDT) at least every six months
peror destroyed 45,000 pounds of British opium in
Canton, beginning the Opium war.
ASIPP
Answers
2310. Answer: A
Explanation:
75% of illicit drug users were employed full or part time,
with American Indian and Alaskan natives at about 12%
of the population of abuse of illicit drug users. Blacks and
whites were essentially equal, with Asians lowest.
Source: Hans C. Hansen, MD
2311. Answer: C
Source: Renee R. Lamm, MD, Sep 2005
2312. Answer: C
Explanation:
Cytochrome P-450 catalyzes demethylation, not methylation or conjugation, reactions. The enzyme is located in
the endoplasmic reticulum, not in mitochondria. Drugs
can induce cytochrome P-450 activity.
2313. Answer: B
Reference: Miscellaneous accepted standards of care; the
Federation of State Medical Boards’ Model Policy for the
Use of Controlled Substances for the Treatment of Pain
(May 2004); and www.deadiversion.usdoj.gov; various
Intractable Pain Treatment Acts from states like
California, Tennessee, and Texas.
Explanation:
A. This is not the best answer because a provider has a
responsibility to minimize the potential for abuse and
diversion of controlled substances (DEA Interim Policy
Statement, Nov. 16, 2004). If the patient is not known to
you and you do not have reliable paperwork from which to
make an informed judgment about the patient’s history of
prior pain treatments, including the use of medications
reported, then you should be very cautious about
prescribing medication. Several states have policy
statements cautioning physicians to “control the drug
supply.”
B. This is the best answer. The fact pattern shows that you
have agreed to see this patient and establish a physicianpatient relationship. For this reason, you must balance
your ethical duty to prevent the patient from experiencing
acute withdrawal symptoms with your legal/regulatory
obligation to minimize the potential for abuse and diversion of controlled substances. Do what you can to verify
the patient’s self-report. Use an appropriate form of urine
or serum screen, especially if the patient has a history of
substance abuse (him/herself or through a first-degree
family relative). Control the initial supply of controlled
substances to this patient.
C. This is not the best answer. You should attempt to
verify the patient’s self-report of recent pain treatments
and medications and perform an appropriate urine screen.
However, you should also control the drug supply and this
answer omits that statement and fails to acknowledge the
ethical duty to prevent or minimize the patient’s acute
withdrawal from controlled substances.
D. This is not the best answer. If you simply accept the
patient’s self-report and continue to treat him/her without
verifying the patient’s past pain treatments and use of
controlled substances, you are likely violating one of your
state’s regulations/rules or guidelines/policies/position
statements on the use of controlled substances for the
treatment of pain. More importantly, you are likely
ignoring accepted standards of care and may place yourself
in a position of prescribing controlled substances outside
the usual course of professional practice.
E. This may appear to be the best answer and it might be
if you had not agreed to see the patient on a referral basis.
In other words, if someone simply drops into your practice on a Friday afternoon claiming they want you to treat
their pain and that their prior physician recommended
you, then you are right to be very cautious about taking
this patient without having the appropriate referral material. If you elect to send a patient back to his/her referring
provider (or if they show up at your office on a self-referral) make sure you contact the referring provider and/or
caution the patient to go to the emergency room if they
believe they are
undergoing acute withdrawal symptoms. No answer is easy
in this situation and your best bet is to document your
medical record carefully and ensure you document your
clinical rational for saying “no,” including any aspect of the
patient’s (1) medical history, (2) behaviors, (3) physical
findings, and (4) lab screen or test results.
Source: Jennifer Bolen, JD, Sep 2005
2314. Answer: D
Explanation:
Reference: The Federation of State Medical Boards’ Model
Policy for the Use of Controlled Substances for the
Treatment of Pain, (May 2004); Bolen, J Pain Medicine
News (Informed Consent).
Explanation:
A. This is not the best answer. A narcotic contract
(typically called a Controlled Substances Treatment
Agreement) usually contains boundaries for use with high
risk patients. Most often, boundary language includes the
use of urine screens, one physician and one pharmacy for
obtaining controlled substances, a specific term for
periodic review, and a discussion of the consequences
should the patient fail to abide by the agreement.
B. This is not the best answer. A History and Physical
Evaluation form is not used to explain the risks and
benefits of using controlled substances to treat pain.
Rather, the H&P form is designed to gather information
about the patient’s medical history and treatment past, so
the provider can decide on a course of treatment. Once the
provider and the patient agree upon a treatment plan, the
provider should engage in informed consent with the
patient.
C. This is not the best answer. A pain scale and periodic
evaluation form are used to follow the patient after
treatment begins.
D. This is the best answer. An Informed Consent form is
different from a Narcotic Contract or Treatment
Agreement, because it helps the practitioner establish the
proper interaction between him/her and the patient
concerning the risks, benefits, treatment alternatives, and
special issues regarding the use of controlled substances to
treat pain. When a practitioner uses an Informed Consent,
he/she is minimizing legal exposure for negligence
associated with the use of controlled substances to treat
pain. Of course, it is up to the practitioner to follow
accepted current clinical care standards, which include a
proper informed consent process.
E. This is not the best answer. Rarely, if ever, does a letter
from the patient’s health care benefit plan contain language relating to informed consent.
Source: Jennifer Bolen, JD, Sep 2005
2315. Answer: C
Explanation:
Reference: 21 C.F.R. § 1307.21 (Disposal of Controlled
Substances).
returned more medication than what you wrote down in
the medical record. The federal law states in § 1307.21(a)
“[a]ny person in possession of any controlled substance
and desiring or required to dispose of such substance may
request assistance from the Special Agent in Charge of the
Administration in the area in which the person is located
for authority and instructions
to dispose of such substance. The request should be made
as follows: (1) If the person is a registrant, he/she shall list
the controlled substance or substances which he/she
desires to
dispose of on DEA Form 41, and submit three copies of
that form to the Special Agent in Charge in his/her area.
Remember, your state guidelines, laws, and regulations
may be stricter and prohibit certain actions or require
more from you in this area.
B. This is not the correct answer. Except in very limited
circumstances which are outside the scope of this question
pattern, the law prohibits the reintroduction of controlled
substances in this manner.
C. This is the best answer. Check with your state licensing
board and/or state bureau of narcotics to determine
whether you can dispose of these controlled substances at
your office and, if the board says it is appropriate to do so,
have the patient inventory the controlled substances
returned, write down the amount on a return form, sign
the return form, use a witness to sign the return form, and
then have the patient witnessed as he/she flushes the
returned substances down the toilet. Alternatively, but only
if your state allows this option, you may follow the
guidance of 21 C.F.R. § 1307.21, when disposing of
controlled substances.
D. This is not the best answer. While flushing returned
drugs down the toilet may seem like an easy option, some
states actually prohibit this action. See the answers to A
and C above for complete guidance.
E. This is not the best answer. While 21 C.F.R. § 1307.21
permits a registrant to contact the Special Agent in Charge
of the nearest DEA office, your state may require you to do
something else. Thus, answer C is the best answer and you
should check with your state licensing board and/or state
bureau of narcotics for guidance on how to dispose of
controlled substances returned by patients. In all cases, you
should document the medical record accurately and
completely.
Explanation:
Source: Jennifer Bolen, JD, Sep 2005
A. This is not the best answer. You must understand the
federal law in this area and then check to see if you state
requires more of you when it comes to disposing of controlled substances. If you wait until after the patient leaves
your office to record the disposal or destruction of the
returned controlled substances, you may be in violation of
state licensing board regulations/rules or guidelines, policy,
or position statements, or the controlled substances
act. Further, the patient may try to argue that he/she
2316. Answer: D
2317. Answer: E
Source: Renee R. Lamm, MD, Sep 2005
2318. Answer: B
Explanation:
Reference: Hardman, pp 632-633.
504
Section 12 • Controlled Substance Management
Hepatic necrosis can occur with overdosage of
Cholinomimetic agents may be used to overcome
acetaminophen. The hepatic toxicity is the result of the
symptoms that persist.
biotransformation of acetaminophen to NSource: Stern - 2004
acetylbenzoquinoneimine, which reacts with hepatic
proteins and glutathione. This metabolite depletes
glutathione, stores and produces necrosis. The
2323. Answer: D
administration of N-acetyl-L-cysteine restores hepatic
Explanation:
concentrations of glutathione and reduces the potential
Fluoxetine is a highly selective serotonin reuptake
hepatotoxicity. Sulindac is biotransformed to sulindac
inhibitor (SSRI) acting on the 5-HT transporter. It forms
sulfide, the active form of the drug. Both sulindac and its
an active metabolite that is effective for several days.
metabolites are excreted in the urine and in the feces.
Selective serotonin reuptake inhibitors are inhibitors of
Indomethacin undergoes a demethylation reaction and an
cytochrome P450 isoenzymes, which is the basis of
N-deacylation reaction. The parent compound and its
potential drug interactions
metabolites are mainly excerted in the urine. Procainamide
is converted to an active metabolite by an acetylation
2324. Answer: B
reaction. The product that is formed is NExplanation:
acetylprocainamide (NAPA). In addition, procainamide is
Reference: Katzung, p 482.
hydrolyzed by amidases. An N-acetylation reaction occurs
Haloperidol, a butyrophenone is by far the most likely
also in the biotransformation of isoniazid. In the liver, the
antipsychotic to produce extrapyramidal toxicities.
enzyme N-acetyltransferase converts isoniazid to
Other agents, such as piperazine (an aromatic
acetylisoniazid.
phenothiazine), thiothixene ( a thioxanthene), and
Source: Stern - 2004
pimozide ( a diphenylbutyropiperidine) are comparatively
less likely to produce extrapyramidal toxicity than
haloperidol.
2319. Answer: B
The antagonism of dopamine in the nigrostriatal system
Explanation:
might explain the Parkinson-like effects.
Reference: Katzung, pp 493-494.
Both haloperidol and pimozide act mainly on D2
A decrease in thyroid function occurs in most patients on
receptors, whereas thioridazine and piperazine act on
lithium. This effect is usually reversible or not
alpha-adrenergic receptors, and have a less potent but
progressive, but a few patients develop symptoms of
definite effect on D2 receptors.
hypothyroidism.
Source: Stern - 2004
A serum thyroid-stimulating hormone (TSH)
concentration is recommended every 6 to 12 months.
2325. Answer: D
“Browning” of vision, clinically described as pigmentary
Explanation:
retinopathy, occurs with thioridazine. This is due to retinal
The W.H.O. ladder emphasizes Step One: Non-Opioid,
deposition of the drug.
Step Two: Opioid for Mild to Moderate Pain, and Step
Although neurologic adverse effects (e.g., tremor,
Three: Opioids for Moderate to Severe Pain. The ladder is
choreoathetosis, motor hyperactivity, ataxia, dysarthria,
just that. It increases potency and adjunctive medication
and aphasia) can occur with lithium, it does not cause the
to treat pain, and is highly effective, well-validated,
neuroleptic malignant syndrome associated with antisuggesting 90% of cancer patients receiving relief. 75% of
psychotic agents.
terminally ill patients also report relief.
Pseudodepression sometimes occurs in patients on antiSource: Hans C. Hansen, MD
psychotics. This may be related to drug-induced akinesia.
Source: Stern - 2004
2320. Answer: D
Source: Hansen HC, Board Review 2004
2321. Answer: D
Source: Hansen HC, Board Review 2004
2322. Answer: A
Explanation:
Reference: Katzung, pp 471, 473, 482.
The phenothiazines as a class are the most potent
anticholinergics of the neuroleptics.
Tolerance to their anticholinergic effects occur in most
patients.
ASIPP
2326. Answer: A
Explanation:
Pseudotolerance is an increase in dosage that is not due to
tolerance, but to other factors such as disease progression,
new disease, increased physical activity, lack of compliance,
change of medication, or drug interaction. It may be a
sign of addiction, more importantly defined as a drug that
has lost its effectiveness at a fixed dosage. Pseudotolerance
should be contrasted to pseudoaddiction. In contrast, of
those that believe “pseudoaddiction” is actually a real
entity, pseudoaddiction is drug seeking behavior that
appears similar to addiction, but is due to unrelieved pain.
Haddox, et al. suggested this term in the literature, and it
seems to have stuck. Actually pseudoaddiction is an
unsophisticated term that is not scientifically relevant.
505
Section 12 • Controlled Substance Management
The concept of pseudoaddiction is better defined by real
terminology that would be considered a component of
tolerance. Plausible scientific explanations of tolerance
offered in the literature suggests that mechanisms
involving cAMP and protein kinase pathways, with up
regulated cellular pathways are important mediators of
tolerance. Receptor modulation is also considered a
potential mechanism, involving sodium and potassium
channels, and, in some instances, endocytosis of opioid
receptors.
Source: Hans C. Hansen, MD
2327. Answer: D
Explanation:
There is no specific ceiling dose for pure opioid agonists,
though we are becoming aware of the potential for
hyperalgia from at least morphine metabolites.
Combination drugs are limited by the APAP or NSAID
content. Use long acting for baseline pain, and short acting
opioids for “breakthrough” pain. Agonist-antagonists have
very little role in chronic pain management in general,
and specifically will trigger withdrawal when used as
breakthrough meds for other agonists.
Source: Andrea M. Trescot, MD
2328. Answer: B
Explanation:
Receptor affinity for the drug will determine the number
of drug-receptor complexes formed. Efficacy is the ability
of the drug to activate the receptor after binding has
occurred. Therapeutic index (TI) is related to safety of the
drug. Half-life and secretion are properties of elimination
and do not influence formation of drug-receptor
complexes.
Source: Laxmaiah Manchikanti, MD
2329. Answer: E
Explanation:
Ref: Murphy. Chapter 16. Opioids. In: Clinical
Anesthesia, 2nd Edition. Barash, Cullen, Stolling;
Lippincott, 1992, pg 431
Source: Day MR, Board Review 2003
they cause minimal central nervous system depression.
Both drug classes facilitate the action of aminobutyric acid
(GABA), although by different mechanisms of action.
They are both used to prevent seizures, and both
can result in physical dependence with long-term use.
2333. Answer: D
Source: Cole EB, Board Review 2003
2334. Answer: B
Source: Day MR, Board Review 2005
2335. Answer: C
Explanation:
Source: Manchikanti L - Pain Physician 2005; 8:257-262.
2336. Answer: C
Source: Chou et al. - J Pain Manage Symptom Manage Vol.
25, No. 5 Nov. 2003, 1026-1048.
2337. Answer: A
Explanation:
The most commonly used illicit drug remains marijuana,
with a lifetime use at roughly 20%. It is the most
commonly used illicit drug on a regular basis, 6.2%, 14.6
million Americans. In contrast 1% were cocaine users,
and ectasy is on the decline, from 3.2 million to 2.1 million
users.
Source: Hans C. Hansen, MD
2338. Answer: A
Source: Chou et al. - J Pain Manage Symptom Manage Vol.
25, No. 5 Nov. 2003, 1026-1048.
2339. Answer: E
Explanation:
A major metabolite of meperidine is to normeperidine.
Accumulation of normeperidine with repeated doses may
cause seizures.
Meperidine should only be used for short term acute pain
management, if at all.
2330. Answer: C
Source: Hans C. Hansen, MD
Pentazocine, morphine, and ketamine are not associated
with toxic metabolites at normal repeated dosing.
2331. Answer: C
Explanation:
Severe tremors, along with confusion, drowsiness,
vomiting, ataxia and dizziness, are an early sign of lithium
toxicity. Retention of lithium may be enhanced by a lowsodium diet because sodium competes with lithium for
reuptake in the kidney. The onset of lithium action may
take a week or more; the drug is used to normalize mood
in patients with mania or bipolar disorder.
Reference: Melzack and Wall 2003, Page 381
Source: Art Jordan, MD, Sep 2005
2340. Answer: D
Explanation:
A noncompetitive antagonist decreases the magnitude of
the response to an agonist but does not alter the agonist’s
potency (i.e., the ED50 remains unchanged). A
competitive antagonist interacts at the agonist binding site.
2332. Answer: D
Explanation:
Benzodiazepines are much safer than barbiturates because
2341. Answer: B
Source: Roger Cicala, MD, Sep 2005
ASIPP
506
Section 12 • Controlled Substance Management
2342. Answer: C
Explanation:
They are contraindicated in patients with a history of
hallucinations, delirium, or paranoid disorders. They are
relatively contraindicated with a history of substance
abuse or hypertension. Their use may exacerbate the
above.
Examples include methylphenidate, destroamphetamine,
and pemoline.
Reference: Melzack and Wall 2003, page 390
Source: Art Jordan, MD, Sep 2005
2343. Answer: A
Explanation:
Tolerance is the concept of the need to increase dosage of
drug to produce the same level of analgesic capacity that
previously existed. Tolerance may occur at a constant
dose, and tolerance should not be considered within the
defined purview of addiction.
Source: Hans C. Hansen, MD
Schedule III. It has an average serum half-life of 3.8 hours.
It is widely used as an antitussive.
Disposition of Toxic Drugs and Chemicals in man. Fifth
Edition. Randall C. Baselt 2000
Drug Enforcement Administration website
Source: Art Jordan, MD, Sep 2005
2348. Answer: D
Explanation:
Benzodiazepines decrease REM and Stage and Stage 4
sleep, resulting in increased fatigue and sleep deprivation.
Some studies have shown a cognitive dysfunction rate as
high as 70% in patients on long term benzodiazepines.
Withdrawal from benzodiazepines may be worse than
opioids. There is no evidence that benzodiazepines have a
primary analgesic effect.
Source: Art Jordan, MD, Sep 2005
2349. Answer: A
Explanation:
While drug interactions based on p;harmacolinetics do
occure with sedative-hypnotics, the most common drug
interaction is additive CNS depression. Additive effects
can be predicted with concomitant use of alchololic
beverages, anticonvulsants, opioid analgesics and
phenothiazines. Less obvious but equally important is
enhanced CNS depression with many antihistamines,
antiypertensives, and antidpressants of th etricyclic class.
Source: Katzung & Trevor’s Pharmacology, Examination
and Board Review, 6th Ed., McGraw Hill, New York, 1998
2344. Answer: B
Explanation:
According to the Henderson-Hasselbalch equation, the pK
equals the pH when the log of the ratio is ionized
(dissociated) and protonated (undissociated) forms is 0
(i.e., their concentrations are equal and have a ratio of 1).
When the pH of a solution (blood) is 7 and the pK of the
acid is 6, at equilibrium, the log of the ratio of
concentrations of ionized form to protonated form is 1
(i.e., there is 10 times more ionized than protonated acid
2350. Answer: A
in the blood). When the pH of a solution (stomach) is 3
Explanation:
and the pK of the acid is 6, the log of the ratio of the
A. The majority of recovering persons can be
concentrations of the ionized to protonated forms is –3
successfully treated with chronic opioid therapy, but they
(i.e., the concentration of the ionized form is 1/1000 that
have some increased risk of addiction.
of the protonated form, meaning that there is 1000 times
B. The majority of recovering persons can be
more protonated than ionized acid). Drug will accumulate
successfully treated with chronic opioid therapy, but they
in the compartment in which it is more highly charged
have some increased risk of addiction.
(ion trapping) – in this case, the blood. Acidification of
C. Even though opioids were not the patient’s drug of
the urine will increase the protonation of an acid and
choice, cross addiction can and does occur.
increase reabsorption, thereby slowing renal excretion.
D. Short acting agents may be more likely to trigger
addictive disease than long acting agents. They certainly
are no safer.
2345. Answer: E
E. There is no such legal ramification, although a
Source: Erin Brisbay McMahon, JD, Sep 2005
physician may face licensure issues for prescribing to a
known active addict.
2346. Answer: C
Source: Roger Cicala, MD, Sep 2005
Explanation:
Pain Medicine: A Comprehensie Review, Second Edition;
2351. Answer: D
P. Prithvi Raj: Mosby, Page 390
Explanation:
Source: Art Jordan, MD, Sep 2005
Long term treatment has the highest success rate for
continued abstinence, about 65%.
2347. Answer: C
Source: Roger Cicala, MD, Sep 2005
Explanation:
D.A.W.N. Drug Abuse Warning Network Reports
2352. Answer: C
Hydrocodone is a Schedule II controlled substance only
Explanation:
when prescribed alone. With acetaminophen, it is a
ASIPP
507
Section 12 • Controlled Substance Management
The biologic half-life (t1/2) will be affected by dose in a
drug when the elimination process is saturated. In that
case, the drug will accumulate on repeated dosings, and
elimination will be independent of plasma concentration.
The amount (not fraction) of drug eliminated per unit
time will be constant, ad zero-order elimination will be
observed.
2353. Answer: C
Explanation:
Most studies indicate the prevalence rate of substance
abuse is 6% to 7%.
Source: Roger Cicala, MD, Sep 2005
2354. Answer: B
Explanation:
Recurrent substance related legal problems are a clear sign
of abuse, but do not in themselves indicate there is
dependence upon the substance.
Source: Roger Cicala, MD, Sep 2005
2355. Answer: B
Source: Andrea M. Trescot, MD
2356. Answer: D
2357. Answer: E
Explanation:
Alcohol accounts for 60% of all cases of substance abuse.
Source: Roger Cicala, MD, Sep 2005
2358. Answer: C
Explanation:
A. Tolerance involves reduced effectiveness, not
complete loss of effect
B. Withdrawal can occur without tolerance, nor does
withdrawal always occur when tolerant patients stop the
substance in question.
C. Tolerance indicates less effectiveness of a given dose.
Increasing the dose can restore the effect, up to a point.
D. Many opiate side effects decrease over time, but not
necessarily in parallel with developing tolerance.
E. A different medication may be more effective, but this
does not of itself demonstrate tolerance.
Source: Roger Cicala, MD, Sep 2005
2359. Answer: C
Explanation:
Pinpoint pupils are a sign of opiate intoxication, dilated
pupils would be more likely in withdrawal. All of the other
symptoms are associated with opiate withdrawal.
Source: Roger Cicala, MD, Sep 2005
2360. Answer: B
Explanation:
It may be surprising, but adults who have completed four
years of college are more likely to experience an illicit
drug, 51.1% of the American population. 38% of those
who did not complete high school have used an illicit
drug. The use is higher in metropolitan than nonmetropolitan areas.
Source: Hans C. Hansen, MD
2361. Answer: E
Explanation:
Different substances may activate different primary
receptors in the brain, but dopaminergic neurons in the
mesolimbic system are activated either directly, or
secondarily with all substances of abuse.
Source: Roger Cicala, MD, Sep 2005
2362. Answer: A
Explanation:
While a callback requirement might be a good idea, it is
not a condition to an oral authorization for a refill of a
Schedule III or IV prescription.
Reference: 21 CFR 1306.22
Source: Erin Brisbay McMahon, JD, Sep 2005
2363. Answer: B
Explanation:
A. Using a substance for purpose other than described
in the PDR has nothing to do with substance abuse.
B. Use of a substance in hazardous situations is one of
the criteria for substance abuse.
C. Development of tolerance to the substance can occur
in any individual who takes the substance, even properly
as prescribed. It MAY indicate substance dependence but is
not a criteria for indicating substance abuse.
D. Attempts to cut down substance use are on of the
criteria for substance dependence, not substance abuse.
E. An episode of withdrawal substance can occur in any
individual who takes the substance in sufficient quantities,
even properly as prescribed. It MAY indicate substance
dependence but is not a criteria for indicating substance
abuse.
Source: Roger Cicala, MD, Sep 2005
2364. Answer: C
Explanation:
Explanation: The signs and symptoms are those of
withdrawal in a patient physically dependent on an opioid
agonist. Such signs and symptoms usually start within 610 hours after the last dose; their intensity depends on the
degree oh physical dependence that has developed. Peak
effects usually occur at 36-48 hours. Mydriasis is a
prominent feature of the abstinence syndrome; other
symptoms include rhinorrhea, lacrimation, piloerection,
muscle jerks, and yawning.
Source: Katzung & Trevor’s Pharmacology, Examination
and Board Review, 6th Ed., McGraw Hill, New York, 1998
2365. Answer: B
Explanation:
Fluvoxamine inhibits liver drug-metabolizing enzymes.
Dosages of alprazolam, theophylline, and warfarin must be
ASIPP
508
Section 12 • Controlled Substance Management
reduced if any of these drugs are given concomitantly with
fluvoxamine. Nefazodone may also decrease the
metabolism of benzodiazepines, and venlafaxine may
inhibit haloperidol metabolism.
Source: Katzung & Trevor’s Pharmacology, Examination
and Board Review, 6th Ed., McGraw Hill, New York, 1998
2366. Answer: E
Explanation:
Concomitant administration of meperidine and MAO
inhibitors has resulted in life-threatening hyperpyrexic
reactions that may culminate in seizures or coma. Such
reactions have even occurred when phenelzine was
administered 14 days after a patient had been treated with
meperidine! Note that concomitant use of SSRIs and
meperidine has resulted in the serotonin syndrome,
another life-threatening drug interaction.
Source: Katzung & Trevor’s Pharmacology, Examination
and Board Review, 6th Ed., McGraw Hill, New York, 1998
2367. Answer: E
Explanation:
The fentanyl transdermal patch releases the drug over 72
hours. The blood levels achieved will often provide
analgesia for postoperative pain but at the same time will
increase arterial PCO2 due to depression of the brain stem
respiratory center. This effect has contributed to severe
respiratory depression with occasional fatalities.
Source: Katzung & Trevor’s Pharmacology, Examination
and Board Review, 6th Ed., McGraw Hill, New York, 1998
2368. Answer: C
Explanation:
The elderly patient may be especially sensitive to
antidepressant drugs that cause sedation, atropine-like side
effects, or postural hypotension. Paroxetine (or another
SSRI) is the best choice for this patient because it is the
least likely of the drugs listed to exert such actions.
Source: Katzung & Trevor’s Pharmacology, Examination
and Board Review, 6th Ed., McGraw Hill, New York, 1998
2369. Answer: C
Explanation:
Opioid calculators are potentially dangerous to use
secondary to metabolic polymorphism. Long acting
opioids have a long half-life, reaching steady state in
approximately 5 days, so increasing doses too frequently
can lead to overdose. Bone pain probably responds better
to NSAIDs. Hyperalgia may be due to M3G.
Source: Andrea M. Trescot, MD
2370. Answer: D
Explanation:
Areas throughout the mesolimbic system, including all of
the above, the stria terminalis, and portions of the frontal
lobes are all activated during intake of a substance of
abuse.
Source: Roger Cicala, MD, Sep 2005
ASIPP
2371. Answer: D
Source: Raj P, Pain medicine - A comprehensive Review Second Edition
2372. Answer: A
Explanation:
Phencyclidine (“angel dust,” “crystal,” “hog”) toxicity
induces organize mental disorders, intoxication, delirium,
delusional mood, and flashback disorders with physical
problems related to high blood pressure, muscle rigidity,
ataxia, coma, nystagmus (particularly vertical), and dilated
pupils. Treatment is with IV diazepam as the drug of first
choice. Propranolol can be used for an adrenergic crisis,
and haloperidol is effective for psychotic and disruptive
behavior. Elimination of the drugs is enhanced by
ammonium chloride in the acute stage and later by
ascorbicacid. Environmental stimuli should be kept to a
minimum. The urine is positive for PCP up to seven days,
but there can be false negatives.
Source: Psychiatry specialty Board Review By William M.
Easson, MD and Nicholas L. Rock, MD
2373. Answer: B
Explanation:
Reference: Katzung, p 538.
A. Flashbacks can occur with use of LSD and mescaline
but have not been associated with the use of cocaine.
B. Use of crack cocaine has led to seizures and cardiac
arrhythmias.
C. Some of cocaine’s effects (sympathomimetic) are due to
blockade of norepinephrine reuptake into presynaptic
terminals; it does not block receptors. D. Crack is the
free-base (nonsalt) form of the alkaloid cocaine. It is called
crack because, when heated, it makes a crackling sound.
E.Heating crack enables a person to smoke it; the drug is
readily absorbed through the lungs and produces an
intense euphoric effect in seconds.
Source: Stern - 2004
2374. Answer: E
Explanation:
If a patient increases the medication despite the knowledge
that he will be discharged, this may be addiction. If he
increases the medication because it is no longer effective,
that may be tolerance.
Source: Trescot AM, Board Review 2004
2375. Answer: C
Explanation:
Reference: Hardman, pp 89-90.
Strychnine acts as a competitive antagonist of glycine, the
predominant postsynaptic inhibitory transmitter in the
brain and spinal cord. The fatal adult dose is 50 to 100mg.
Persons poisoned by strychnine suffer convulsions that
progress to full tetanic convulsions. Because the
diaphragm and thoracic muscles are fully contracted, the
patient cannot breathe. Hypoxia eventually causes
medullary parasysis and death. Control of the convulsions
509
Section 12 • Controlled Substance Management
and respiratory support are the immediate objectives of
the therapy. Diazepam may be preferred to a barbiturate in
controlling the convulsions because it offers less
concomitant respiratory depression. Poisoning caused by
the other drugs listed in the question is not associated with
convulsions but with depression of the CNS.
Source: Stern - 2004
2376. Answer: C
Explanation:
Reference: Hardman, pp 574-575
A. Phencyclidine has no opioid activity
B. Its mechanism of action is amphetamine-like with
opioid activity.
C. Phencyclidine is a hallucinogenic compound with no
opioid activity. Its mechanism of action is amphetaminelike.
D. A withdrawal syndrome has not been described for this
drug in human subjects.
E. In overdose, the treatment of choice for the psychotic
activity is the antipsychotic drug haloperidol.
Source: Stern - 2004
C. Cocaine is biotransformed by plasma esterases to
inactive products.
D. Cocaine has local anesthetic properties; it can block the
initiation or conduction of a nerve impulse.
E. Cocaine also blocks the reuptake of norepinephrine.
This action produces CNS stimulant effects including
euphoria, excitement, and restlessness.
Source: Stern - 2004
2380. Answer: E
Explanation:
Only E is correct. A is expressly prohibited; B is not
discussed in the Act at all, C is incorrect because “narcotic”
is defined to include not only the opioids and opiates, but
also cocaine. D is incorrect because medical cocaine is a
C-II substance, and is often used as a topical anesthetic in
sinus surgery.
Source: Arthur Thexton
2381. Answer: C
Explanation:
Sedation and pruritus (due to direct histamine release)
abate over time. Although tolerance to pain relief can
occur, with long acting narcotics (especially methadone) it
is less likely. Constipation, however, should be expected to
be a problem for the entire length of treatment.
Source: Trescot AM, Board Review 2004
2377. Answer: E
2378. Answer: D
Explanation:
You could easily argue that alcohol far exceeds marijuana
in use, based on national data. 120 million Americans, or
50% of the population consume alcohol. The definition
would easily move to alcohol as the number one used illicit
drug, if alcohol was uniformly considered “a drug”. It
does have physical dependence, psychic and toxic effects,
but for purposes of terminology and the use of controlled
substances, marijuana will be considered the number one
drug of abuse. Ironically, it is now not a controlled
substance, as the Supreme Court has defined no legitimate
medical need for marijuana. Marijuana is properly termed
a drug of abuse, Schedule I. There are 7 thousand new
users per day.
Source: Hans C. Hansen, MD
2379. Answer: D
Explanation:
Reference: Hardman, pp 338, 570.
A. Peripherally, cocaine produces sympathomimetic
effects including tachycardia and vasoconstriction.
B. Cocaine is an ester of benzoic acid and is closely related
to the structure of atropine. Death from acute overdose
can be from respiratory depression or cardiac failure.
2382. Answer: E
Explanation:
29 CFR § 1910.1030(d)(3).
Source: Erin Brisbay McMahon, JD
2383. Answer: B
Explanation:
65 Fed. Reg. 59434
Source: Erin Brisbay McMahon, JD
2384. Answer: B
Explanation:
Buprenorphine has low oral bioavailability, but is well
absorbed sublingually. Naloxone has poor sublingual
bioavailability, but is formulated along with
buprenorphine to prevent misuse when administered IV.
Buprenorphine is a partial agonist at the mu-opiate
receptor and an antagonist at the kappa receptor.
While buprenorphine has high opioid mu receptor affinity,
it has a low intrinsic activity.
Malinoff HL, Barkin RL, Wilson G. Sublingual
buprenorphine is effective in the treatment of chronic pain
syndrome. American Journal of Therapeutics 2005; 12(5):
379-384
http://buprenorphine.sahsa.gove/about.html
Source: James D. Colson, MS, MD
ASIPP
510
Section 12 • Controlled Substance Management
2385. Answer: B
Explanation:
Butorphanol exhibits opioid kappa receptor agonist and
mu receptor antagonist activity. Respiratory depression
with butorphanol is similar to that produced by equivalent
morphine doses. Histamine release is prominent with
morphine and not butorphanolStoelting RK.
Pharmacology and Physiology in Anesthesia Practice. 3rd
ed. Lippincott-Raven, Philadelphia, 1999.
Evers AS, Maze M. Anesthetic Pharmacology: Physiologic
Principles and Clinical Practice. Churchill Livingstone,
Philadelphia, 2004.
Stoelting RK. Pharmacology and Physiology in Anesthesia
Practice. 3rd ed. Lippincott-Raven, Philadelphia, 1999
Source: James D. Colson, MS, MD
2386. Answer: E
Explanation:
Schedule III substances are comprised of drugs possessing
analgesic, anabolic steroid, sedative, and/or hypnotic
properties.
Controlled Substances Act-U.S. Drug Enforcement
Administration (http://www.usdoj.gov/dea/agency/
csa.htm)
Source: James D. Colson, MS, MD
2387. Answer: D
Explanation:
Drug properties, such as receptor affinity, intrinsic activity
or the propensity to develop tolerance or tachyphylaxis
are more inherent to the drug itself and not specifically to
its duration of action or particular formulation. Opioids
as a class do not have a ceiling effect for analgesia. Longacting opioids tend to have a delayed onset and protracted
course of withdrawal following abrupt discontinuation.
Stoelting RK. Pharmacology and Physiology in Anesthesia
Practice. 3rd ed. Lippincott-Raven, Philadelphia, 1999.
Savage SR. Critical clinical issues in pain and addiction.
Pain Management Rounds 2005; 2(9):
Source: James D. Colson, MS, MD
2388. Answer: D
Explanation:
A. Schedule II drugs have a high potential for abuse and a
high rate of psychological or physical dependence.
Narcotic analgesics are schedule II drugs. Non-narcotic
drugs such as amphetamines, methylphenidate, and
pentobarbital are also schedule II.
B. Schedule III drugs have a potential for abuse less than
the drugs or other substances in schedules I and II. Abuse
of the drug or other substance may lead to moderate or
low physical dependence or high psychological
dependence.
ASIPP
C. Schedule V drugs have the lowest potential for abuse of
the drugs under the jurisdiction of the Controlled
Substances Act. Antitussives and antidiarrheal
preparations that contain narcotics in limited quantities
are schedule V drugs.
D. Schedule I is reserved for the most dangerous drugs
without recognized medical value.
E. Schedule IV drugs are thought to have less potential for
abuse than schedule I, II, and III drugs. Examples of
schedule IV drugs are benzodiazepines, phenobarbital,
meprobamate, chloral hydrate, and dextropropoxyphene
(Darvon).
2389. Answer: A
Explanation:
Within 7 days after the oral authorization to the
pharmacist, the physician must deliver to the pharmacist a
written prescription for the emergency quantity
prescribed, and the written prescription must have written
on its face “Authorization for Emergency Dispensing” and
the date of the oral authorization.
Reference: 21 CFR 290.10, 1306.11(d).
Source: Erin Brisbay McMahon, JD, Sep 2005
2390. Answer: E
Explanation:
A. At high doses, benzodiazepines may cause anterograde
but not retrograde, amnesia.
B. Diazepam use can cause a decrease in psychomotor
function.
C. Diazepam has no more effectiveness than placebo in
the treatment of major depression
D. Benzodiazepines do not increase activity of liver drugmetabolizing enzymes or of enzymes involved in
porphyrin synthesis.
E. With abrupt discontinuance following chronic use,
anxiety and agitation may occur, sometimes with
hyperreflexia and, rarely, seizures.
Source: Katzung & Trevor’s Pharmacology, Examination
and Board Review, 6th Ed., McGraw Hill, New York, 1998
2391. Answer: E
Explanation:
A-D. Benzodiazepines and barbiturates are ontraindicated
in breathing-related sleep disorders because they will
further compromise ventilation. In the obstructive sleep
apnea syndrome (pickwickian syndrome), obesity is a
major risk factor. The best prescription you can give this
patient is to lose weight.
E. Patient probably suffers with sleep apnea syndrome and
should be referred to sleep disorder clinic or sleep study.
Source: Katzung & Trevor’s Pharmacology, Examination
and Board Review, 6th Ed., McGraw Hill, New York, 1998
511
Section 12 • Controlled Substance Management
2392. Answer: C
Explanation:
A. The dose-response curve for benzodiazepines is flatter
than that for barbiturates.
B. Flumazenil is an antagonist at Benzodiazepine
receptors and is used to reverse CNS depressant effects of
benzodiazepines.
C. As a weak acid (pKa +=7), phenobarbital will exist
mainly in the ionized (nonprotonated) form in the urine
at alkaline pH and will not be reabsorbed in the renal
tubule.
D. Induction of liver drug-metabolizing enzymes occurs
with barbiturates and may lead to decreases in half-life of
other drugs.
E. Withdrawal symptoms from use of the shorter-acting
barbiturate secobarbital are more severe than with
phenobarbital.
Source: Katzung & Trevor’s Pharmacology, Examination
and Board Review, 6th Ed., McGraw Hill, New York, 1998
2393. Answer: A
Explanation:
It would be tempting to increase the tramadol, since he is
not on the maximum of 400mg/day. However, that would
increase his risk of seizures even more. The simplest
treatment would be to change his antidepressant to one
that was not a CYP2D6 inhibitor, which would then allow
the tramadol to be more active, while at the same time
decreasing the risk of seizures because of excretion of the
drug. Codeine, hydrocodone, and oxycodone are also
metabolized to active forms by CYP2D6, and therefore
would also be less effective in the face of CYP2D6
inhibition. Agonist/antagonist opioids are not usually a
good choice for chronic pain management.
Source: Andrea M. Trescot, MD
2394. Answer: C
Explanation:
Because tramadol is not only activated by CYP2D6, but
also metabolized for excretion by CYP2D6, the patient is at
risk for accumulation of the drug, leading to seizures.
Source: Andrea M. Trescot, MD
2395. Answer: C
Explanation:
Decreased GI motility is the deliberate effect of poorly
absorbed opioids such as loperamide (Lomotil). The rest
are usually undesirable effects.
Source: Andrea M. Trescot, MD
2396. Answer: A
Explanation:
Only 8% of new low back pain patients made a return
appointment after 12 months. 25% still had pain at 12
months, and 50% still had difficulties with ADLs at 12
months. 79% still had pain at 3 months, and 32% made
follow up at 3 months.
Croft PR, Macfarlane GJ, Papageorgiou AC, et al. Outcome
of low back pain in general practice: a prospective study.
BMJ (1998);316(7141):1356-9
Source: Andrea M. Trescot, MD
2397. Answer: B
Source: Stimmel, B
2398. Answer: C
Source: Stimmel, B
2399. Answer: B
Explanation:
Answers (a) and (c)-(e) are noted in the DEA’s Interim
Policy Statement as behaviors that lead to criminal
convictions.
Reference: 69 Fed. Reg. 67170.
Source: Erin Brisbay McMahon, JD, Sep 2005
2400. Answer: D
Explanation:
Answer (d) is wrong because, to be convicted of being the
manager or organizer of a continuing criminal enterprise,
a person must violate a provision of the Controlled
Substances Act, the punishment for which is a felony.
Reference: 21 U.S.C. § 848.
Source: Erin Brisbay McMahon, JD, Sep 2005
2401. Answer: C
Explanation:
Answer (c) should be limited to thirty patients.
Reference: 21 USC 823(g); 21 CFR 1306.07(d).
Source: Erin Brisbay McMahon, JD, Sep 2005
2402. Answer: E
Explanation:
Answer (e) is not one of the credentials that will render a
physician a qualified physician to dispense and prescribe
Schedule III, IV, or V controlled substances specifically
approved by the Food and Drug Administration for
narcotic addiction treatment to a narcotic dependent
person.
Reference: 21 USC 823(g)(2)(B)
Source: Erin Brisbay McMahon, JD, Sep 2005
2403. Answer: B
Explanation:
Schedule III and IV prescriptions may not be refilled more
than 5 times after the date of the original prescription
unless renewed by the prescribing practitioner.
Reference: 21 USC 829 (b) - (c).
Source: Erin Brisbay McMahon, JD, Sep 2005
ASIPP
512
Section 12 • Controlled Substance Management
2404. Answer: D
2405. Answer: A
Source: Hans C. Hansen, MD
Source: Arthur Thexton
2410. Answer: A
Source: Stimmel, B
2411. Answer: D
2406. Answer: D
Source: Stimmel, B
Explanation:
The seizures from normeperidine cannot be reversed by
2412. Answer: E
naloxone. Although it has been used for many years,
Source: Stimmel, B
recognition of its poor analgesia, metabolite accumulation,
and abuse potential has lead to gradual condemnation by 2413. Answer: C
the pain community. It is metabolized by glucuronidation,
Source: Stimmel, B
is not safe in the elderly and does not become more
effective over time.
2414. Answer: A
Source: Andrea M. Trescot, MD
Source: Stimmel, B
2407. Answer: D
Source: Stimmel, B
2415. Answer: E
Source: Murray McAllister, PsyD, LP - Spring 2004
2408. Answer: A
Explanation:
A. Constipation is the most frequent side effect of opioid
therapy.
2416. Answer: A
Source: Stimmel, B
2417. Answer: A
Tolerance does not develop to this side effect.
Therefore, as the dose of opioid increases, so does
the
potential for constipation.
Frank bowel obstruction, biliary spasm, and ileus have
occurred with opioid use.
2418. Answer: D
Source: Stimmel, B
2419. Answer: A
Source: Stimmel, B
2420. Answer: A
It is crucial to place patients on an active bowel regimen
Source: Stimmel, B
that includes laxatives, stool softeners,
adequate fluids and exercise, and cathartics as needed to 2421. Answer: E
prevent the severe constipation that
Source: Stimmel, B
can occur with opioid use.
2422. Answer: E
B. Surgical complications are unlikely.
Source: Stimmel, B
C. Constipation is not a symptom of drug abuse.
2423. Answer: D
Source: Stimmel, B
D. Transdermal fentanyl may be an option if morphine
titration fails. Constipation is similar.
E. Methadone maintenance is not indicated
Source: Manchikanti L, Board Review 2005
2409. Answer: D
Explanation:
A is not covered at all in the CSA, but is a matter of state
law. B is incorrect because the CSA does not permit this;
the HIPAA rule does this. C is incorrect, in that a special
registration is required to order C-I substances for any
purpose. And, E is incorrect because methadone may be
prescribed for pain relief or for any other medically
appropriate purpose, EXCEPT the treatment of addiction,
without any special registration.
ASIPP
2424. Answer: A
Explanation:
Only 1% of the work force was absent from work,
headaches and low back pain were the most common
complaints, 12% of the workforce lost productive time due
to pain, and an average of 4.6% hours per week are lost
because of pain.
Stewart WF, Ricci JA, et al. Lost Productive Time and Cost
Due to Common Pain Conditions in the US Workforce.
JAMA (2003)290:18, p. 2446.
Source: Andrea M. Trescot, MD
2425. Answer: D
Source: Stimmel, B
513
Section 12 • Controlled Substance Management
2426. Answer: A
Explanation:
Reference: Federal and state materials nationwide; Bolen, J
Pain Medicine News; Bolen, J Journal of Opioid
Management (forthcoming publication 2005).
Explanation:
A. This is the best answer. At the federal level there are
three main types of legal/regulatory materials governing
the parameters of and record-keeping for the use of
controlled substances to treat pain: (1) laws, like the
Controlled Substances Act of 1970, (2) regulations
governing the issuance of prescriptions, as found in the
Code of Federal Regulations, and (3) policy statements
and rules, as used by DEA to explain the federal
interpretation or position on the laws and regulations.
B. This is not the best answer. Laws and regulations do
exist at the federal level. However, the DEA uses an
additional level of legal/regulatory material called policy
statements and rules to explain the laws. It is vital for DEA
Registrants to read and understand these policy statements
and rules because they give the registrant a better
understanding of how the DEA applies and interprets the
federal legal/regulatory materials governing recordkeeping and the use of controlled substances to treat pain.
C. This is not the best answer. Although the Controlled
Substances Act of 1970 is a law, it is just part of the body of
federal materials governing the use of controlled
substances in the United States.
D. This is not the best answer. The DEA policy statements,
such as the Interim Policy Statement of November 16,
2004, are only a part of the body of federal materials
governing the use of controlled substances in the United
States.
E. This is not the best answer. The Food and Drug Act is a
law that, in general, pertains to the development, approval,
and marketing of drugs in the United States. While it
certainly has much to do with the drugs that ultimately get
scheduled under the controlled substances act, it is only
part of the body of federal materials governing the use of
controlled substances in the United States.
Source: Jennifer Bolen, JD, Sep 2005
2427. Answer: E
Explanation:
Reference: Hardman, p 930.
All the drugs listed in the question are used as antiemetics.
A. Diphenhydramine and dimenhydrinate are used orally
for the active and prophylactic treatment of motion
sickness.
B. Chlorpromazine is a general antiemetic, used orally,
rectally, or by injection for the control of nausea and
vomiting that is caused by conditions that are not
necessarily defined.
C. Ondansetron is indicated in the oral or intravenous
route for the prevention of nausea and vomiting caused by
cancer chemotherapy.
D. Dimenhydrinate is used for prophylaxis and treatment
of motion sickness.
E. Scopolamine is a transdermal preparation used in the
prevention of motion sickness. The drug is incorporated
into a bandage-like adhesive unit that is placed behind the
ear. The scopolamine delivered in this manner is well
absorbed and maintainsan effect for up to 72h.
Other drugs that are prepared for transdermal
delivery include clonidine (an antihypertensive agent),
estradiol (an estrogen), fentanyl ( an opioid analgesic),
nicotine (a smoking deterrent), nitroglycerin (an
antianginal drug), and testosterone ( an androgen).
Source: Stern - 2004
2428. Answer: B
Source: Raj, Pain Review 2nd Edition
2429. Answer: B
Explanation:
Reference: The Controlled Substances Act of 1970, codified
at 21 U.S.C. § 801 and sections that follow; see also 1994,
Joranson DE, Gilson AM. Chapter 8 - Controlled
substances, medical practice and the law. In: Schwartz HI.
Psychiatric Practice Under Fire: The Influence of
Government, the Media and Special Interests on Somatic
Therapies. Washington, DC: American Psychiatric Press,
Inc., 1994:173-194.
Explanation:
A. This is not the correct answer. The Controlled
Substances Act of 1970 does give DEA the authority to
schedule drugs according to whether the drug has been
approved by the Food and Drug Administration for
medical use and according to the drug’s potential for
abuse.
B. This is the correct answer. The states, not the federal
government, have the authority to regulate medical
practice. State authority derives from both federal and
state constitutions. States create medical practice acts to
regulate the practice of medicine and protect the public.
The CSA does not give DEA the authority to regulate
medical decisions and it does not permit DEA to change
or limit indications for which a drug may be prescribed.
Likewise, the CSA does not give DEA the authority to
regulate the quantity or chronicity of prescribing relative
to controlled substances.
ASIPP
514
Section 12 • Controlled Substance Management
C. This is not the correct answer because the Controlled
Substances Act of 1970 does give DEA authority to
administer the CSA and create policy related to the various
provisions of the CSA.
D. This is not the correct answer because the Controlled
Substances Act of 1970 does give DEA authority to
establish manufacturing quotas that drug manufacturers
must follow when producing controlled substances. There
are many facets to DEA’s authority to establish
manufacturing quotes and the important point is that
DEA must ensure that there are sufficient controlled drugs
available to meet legitimate health care demands.
E. This is not the correct answer because the Controlled
Substances Act of 1970 does give DEA authority to
reschedule drugs into higher or lower schedules,
depending on whether a drug remains approved for
medical use and depending on developments and trends
regarding the trends for the abuse of these drugs.
Source: Jennifer Bolen, JD, Sep 2005
D. This is not the correct answer because most states allow
physicians to use mid-level practitioners in the treatment
of patients. These working relationships allow for
practitioners to see more patients on a given day. While
some state laws may require the physician to sign all
prescriptions, in many cases mid-level practitioners also
have DEA registrations and thus may prescribe controlled
substances for the treatment of pain. Anyone who signs a
controlled substances prescription must be licensed
properly by state authorities and registered with the DEA.
E. This is not the correct answer. Federal and state laws do
not require physicians to issue controlled substances
prescriptions within three days of seeing the patient.
Source: Jennifer Bolen, JD, Sep 2005
2431. Answer: B
Explanation:
Reference: DEA Interim Policy Statement, Fed. Reg. Vol.
69, No. 220, pp. 67170-67172 (Nov. 16, 2004).
Explanation:
2430. Answer: B
Explanation:
Reference: Code of Federal Regulations, 21 C.F.R. §
1306.04 (Prescriptions).
Explanation:
A. This is not the best answer. Prescriptions must meet
both substantive and technical requirements to be valid. As
a technical matter, a prescription must be dated and
signed on the date it is actually issued to the patient or
guardian. The prescription must contain not only the date
and the properly licensed and registered provider’s
signature, but also information about the patient and the
drug prescribed. However, date and signature alone are
not enough to make a prescription valid and the
prescription must meet the substantive requirements of
(1) legitimate medical purpose, and (2) usual course of
professional practice to be wholly valid.
B. This is the best answer. To be valid, the federal and state
laws and regulations require a prescription be issued for a
legitimate medical purpose within the usual course of
professional practice. State licensing boards often add to
this requirement by requiring physicians and other health
care practitioners to document the medical record to show
compliance with applicable laws and regulations
governing controlled substance prescribing.
A. This is not the correct answer. While a physician must
report to DEA if their DEA number has been
compromised, the Interim Policy Statement does not
discuss this duty.
B. This is the best answer. The Interim Policy Statement
states: “[g]iven the addictive and sometimes deadly nature
of prescription narcotic abuse, the tremendous volume of
such drug abuse in the United States, and the propensity
of many drug addicts to attempt to deceive physicians in
order to obtain controlled substances for the purpose of
abuse, a physician should seriously consider any sincerely
expressed concerns about drug abuse conveyed by family
members and friends.”
C. This is not the best answer because the Interim Policy
Statement did not assign a time frame within which to see
patients and does not say that you must see your patients
every thirty days.
D. This is not the correct answer. To be valid, a physician
must issue a prescription for controlled substances for (1)
a legitimate medical purpose, and (2) within the usual
course of professional practice. This has been the law of
the Controlled Substances Act of 1970 and was not a new
directive from the Interim Policy Statement.
E. This is not the correct answer. The Interim Policy
Statement did not assign a time frame within which a
physician must review patient records.
Source: Jennifer Bolen, JD, Sep 2005
C. This is not the correct answer because federal and state
laws do not prohibit a physician from prescribing
controlled substance to a person with a criminal history.
2432. Answer: C
The only caveat to this statement concerns the patient’s
Explanation:
history of substance abuse, as it may call for restrictive
Reference: 21 U.S.C. § 823; 21 C.F.R. § 1301.01 through
monitoring on the patient’s use of controlled substances or
1301.55.
other restrictions to ensure that the controlled substances
prescribed get used as they are intended – for pain relief.
ASIPP
515
Section 12 • Controlled Substance Management
Explanation:
DEA requires practitioners to renew their DEA
registration number every 3 years. This may be done online through www.deadiversion.usdoj.gov. Moreover,
DEA requires practitioners to request modifications of
their DEA registration numbers when they move to a new
medical practice or open another office requiring
additional registration.
Source: Jennifer Bolen, JD, Sep 2005
2433. Answer: B
Explanation:
Reference: 21 U.S.C. § 823; 21 C.F.R. § 1301.12;
www.deadiversion.usdoj.gov.
Explanation:
A. This is not the correct answer. If a practitioner
administers and/or dispenses controlled substances at
more than one location, he/she must register and post a
DEA registration number at each principal place of
business or professional practice where controlled
substances are stored, administered, or dispense by a
person. Thus, under this hypothetical registering only one
principal office location is insufficient.
B. This is the best answer. A separate registration is
required for each principal place of business or
professional practice where controlled substances are
stored, administered or dispensed by a person. If a
practitioner will only be prescribing from an additional
location located within the same state, then an additional
registration is not necessary.
C. This is not the best answer. A practitioner must register
each principal place of business or professional practice
where controlled substances are stored, administered, or
dispensed by a person.
D. This is not the correct answer. Do nothing is not the
answer here as a registration is required at all locations
where practitioners administer and/or dispense controlled
substances. This is because the DEA requires registration
of locations that keep controlled substances on the
premises. The practice is different if the only thing kept on
the premises is a prescription pad.
E. This is not the correct answer. Registering a principal
place of business where one administers and/or dispenses
controlled substances is different from registering every
place where the practitioner travels and issues
prescriptions for controlled substances. Practitioners
must register their principal place of business. If a
practitioner has several offices, he/she must register
separately those offices where he/she administers and/or
dispenses controlled substances. However, he/she is not
required to register every location where he/she uses
prescription pads for controlled substances prescriptions.
Source: Jennifer Bolen, JD, Sep 2005
2434. Answer: A
Explanation:
Reference: 21 U.S.C. § 823; 21 C.F.R. § 1301.11 (separate
registrations) and § 1301.51 (modification of
registration); www.deadiversion.usdoj.gov.
Explanation:
A. This answer is the best answer. If you move your
principal registered location, you must send DEA a request
for modification of registration in writing and obtain
approval for the modified registration prior to the move.
This request for modification should be addressed to the
DEA field office nearest to your currently registered
location. You may obtain an Address Change Request from
DEA’s internet site at www.deadiversion.usdoj.gov. You
must manually sign and fax or mail it to the local DEA
office. The form must include (1) a copy of your current
state medical/professional license for the new address
along with a copy of the practitioner’s corrected state
controlled substance registration, if applicable.
B. This is not the correct answer. You may not wait until
the next renewal date for your controlled substances
registration to notify DEA of the move. If you do so, you
may lose your registration number and you will be
considered in violation of the law.
C. This is not the best answer. You must first get
permission from DEA to modify your registration, and
thus simply telling your state drug bureau is not sufficient.
You should learn what your state requires from you if you
move principal locations from which you administer,
dispense, and prescribe controlled substances
D. This is not the correct answer. You may not wait until
after you move to send DEA a request for a new
registration number. Because you are administering and/
or dispensing controlled substances from your currently
registered location, you must first obtain DEA’s approval
to use the new location to store controlled substances.
E. This is not the correct answer. You may not wait until
DEA asks you for updated information about your
registration and current address. DEA puts the burden on
you, the registrant, to stay current with the paperwork
surrounding the registration of individuals and locations.
Source: Jennifer Bolen, JD, Sep 2005
2435. Answer: E
Explanation:
Reference: The Controlled Substances Act of 1970, codified
in 21 U.S.C. § 801 and sections that follow.
Explanation:
A. The Food, Drug, and Cosmetic Act of 1962, establishes
the law on drug development, approval for medical use,
and marketing in the United States. However, this Act does
ASIPP
516
Section 12 • Controlled Substance Management
not contain the law governing the scheduling of drugs as
controlled substances.
B. There is no such federal act. Instead, many states have
adopted a Uniform Controlled Substances Act (sometimes
referred to as a Uniform Controlled Drugs Act).
C. The Code of Federal Regulations explains various
components of the Controlled Substances Act of 1970, but
the CFR does not contain the law relating to the
scheduling of drugs in the United States.
D. There is no such federal act.
E. The Controlled Substances Act of 1970 contains the law
in the United States governing the scheduling of drugs as
“controlled substances.” The CSA places controlled
substances into five schedules. Schedule I contains drugs
with no accepted medical use. Schedule I drugs are
available only for scientific research. Schedules II-V
contain drugs that have been approved for medical and
schedules them according to potential for abuse, with
drugs having the highest potential for abuse assigned to
Schedule II.
Source: Jennifer Bolen, JD, Sep 2005
2436. Answer: C
Explanation:
Reference: 21 C.F.R. § 1306.11(a) (Requirement of a
Prescription), which states “[a] prescription for a
Schedule II controlled substance may be transmitted by
the practitioner or the practitioner’s agent to a pharmacy
via facsimile equipment, provided that the original
written, signed prescription is presented to the pharmacist
for review prior to the actual dispensing of the
controlled substance, except as noted in paragraph (e), (f),
or (g) of this section. The original prescription shall be
maintained in accordance with § 1304.04(h) of the CSA.”
prior to dispensation of the prescription. This measure
would prove helpful in determining whether the patient in
question has attempted to alter your prescriptions. Do not
tell the patient that you are faxing a copy of the
prescription to the pharmacist.
D. This is not the correct answer because faxes are not
limited to emergency prescriptions.
E. This is not the correct answer because faxes are not
limited to hospice and nursing home situations. However,
it is important to note that the requirement of the original
prescription does not apply to hospice and nursing home
situations.
Source: Jennifer Bolen, JD, Sep 2005
2437. Answer: C
Explanation:
All opioids can potentially be abused. Meperidine may be
useful for acute pain, and it is cheap. The metabolite
normeperidine can cause seizures and can accumulate
with chronic dosing, especially in renal failure
Source: Trescot AM, Board Review 2004
2438. Answer: C
Explanation:
Reference: Hardman, p 546.
Pentazocine is a mixed agonist-antagonist of opioid
receptors. When a partial agonist, such as pentazocine,
displaces a full agonist, such as methadone, the receptor is
less activated; this leads to withdrawal syndrome in an
opioid-dependent person.
Source: Stern -2004
2439. Answer: D
Explanation:
Reference: 21 U.S.C. § 823; 21 C.F.R. 1306.07; and
www.deadiversion.usdoj.gov.
Explanation:
Explanation:
A. This is not the best answer because it states only one of
the two requirements concerning the faxing of
prescriptions. While it is legal to fax a Schedule II
prescription to a pharmacist, the pharmacist may not
dispense the prescription to the patient without the
original prescription.
B. This is not the correct answer because HIPAA consent
has nothing to do with this law regarding the faxing of
Schedule II controlled substances prescriptions to
pharmacists. If you want to discuss the patient’s
prescription with the pharmacist in connection with your
treatment of the patient, you may do so and HIPAA does
not require the patient’s consent for such conversations.
C. This is the best answer because it is legal to fax a
Schedule II prescription to a pharmacist and the patient
must present the original prescription to the pharmacist
ASIPP
A. This is not the correct answer because the 72-hour
exception to the federal law requirement of a separate
registration for detoxification or maintenance treatment
only allows a practitioner to administer or dispense (but
not prescribe) (1) one day’s worth of emergency
medication to the patient at one time, (2) for not more
than a total of 72-hours, and (3) the practitioner may not
extend or renew the 72-hour period. This 72-hour
exception is known as the “three day rule” and it is found
in 21 C.F.R. § 1306.07(b). Thus, if a practitioner is not
separately registered as a narcotic treatment program,
he/she may administer BUT NOT prescribe narcotic drugs
to a patient for the purpose of relieving acute withdrawal
symptoms while arranging for the patient’s referral for
treatment. Congress intended § 1306.07(b) to give
practitioners flexibility in emergency situations when
confronted with a patient undergoing withdrawal. Thus,
517
Section 12 • Controlled Substance Management
Congress established this exception to “augment, not to
circumvent” the separate registration requirement set
forth in the CSA.
B. This is not the correct answer because no one is legally
permitted to prescribe methadone to detoxify or maintain
a patient for addiction.
C. This is not the correct answer because having only the
training on the use of Schedule III-V controlled substances
to perform the office-based treatment of opioid addiction
is not enough. Practitioners must obtain an “X”
certification from DEA to use Schedule III-V controlled
substances to treat patients in their offices for opioid
addiction. Also, you may administer or dispense, but not
prescribe, methadone according to the “three day rule”
described above in answer A.
D. This is the best answer. If a practitioner wants to use
Schedule II narcotic drugs for maintenance and/or
detoxification, federal and state law require the
practitioner to obtain separate registration from the DEA
as a narcotic treatment program pursuant to the Narcotic
Addict Treatment Act of 1974. Significantly, this
registration allows a practitioner to administer or
dispense, but not prescribe, scheduled narcotic drugs that
are approved by the United States Food and Drug
Administration (FDA) for the treatment of narcotic
addiction. Until October 2000, methadone and levo-alphaacetyl-methadol (LAAM) were the only scheduled
narcotics approved by FDA for use in maintenance and
detoxification treatment. In October 2000, pursuant to the
Drug Abuse Treatment Act of 2000 (DATA 2000),
Congress amended the CSA (21 U.S.C. § 823(g)) to allow
properly registered and trained practitioners to dispense
or prescribe Schedule III-V controlled substances
specifically approved by the FDA for the office-based
treatment of opioid addiction.
E. This is not the correct answer. While a separate
registration is required to render narcotic treatment
services, no DEA registration to do so allows any
practitioner to prescribe Schedule II controlled substances
to detoxify or maintain an individual with an addiction. As
stated above in answer D, those properly registered and
trained may prescribe those drugs in Schedules III-V
approved by FDA for the purpose of detoxification from
opioids.
Source: Jennifer Bolen, JD, Sep 2005
2440. Answer: B
Explanation:
Tricyclics modify peripheral sympathetic effects in two
ways; through blockade of norepinephrine reuptake at
neuroeffector junctions and through alpha adrenoceptor
blockade. Sedation and atropine-like side effects are
common with tricyclics, especially amitriptyline. In
contrast to sedative-hypnotics, tricyclics lower the
threshold to seizures.
Source: Katzung & Trevor’s Pharmacology, Examination
and Board Review, 6th Ed., McGraw Hill, New York, 1998
2441. Answer: D
2442. Answer: E
Explanation:
The mechanism of action of bupropion is unknown, but
the drug does not inhibit amine transporters. Levels of
norepinephrine and serotonin metabolites in the
cerebrospinal fluid of depressed patients prior to drug
treatment are not higher than normal. Some studies have
reported decreased levels of these metabolites. Downregulation of adrenoceptor appears to be a common
feature of all mode3s of chronic drug treatment of
depression, including the use of drugs that have no direct
actions o catecholamine receptors. MAO inhibitors used
in depression are nonselective.
Source: Katzung & Trevor’s Pharmacology, Examination
and Board Review, 6th Ed., McGraw Hill, New York, 1998
2443. Answer: D
Explanation:
The SSRIs have CNS-stimulating effects. They may cause
agitation, anxiety, “the jitters”, and insomnia. The evening
is not the best time to take such drugs. Anorexia and
nausea, akathisia, dyskinesias, and dystonic reactions may
occur. Because of the possibility of drug interactions, the
physician needs to be informed of changes in drug
regimens when maintaining a patient on antidepressants.
Source: Katzung & Trevor’s Pharmacology, Examination
and Board Review, 6th Ed., McGraw Hill, New York, 1998
2444. Answer: D
Explanation:
Kappa receptor activation does not appear to be
responsible for dependence, euphoria, or effects on
smooth muscle. Increases in cerebral blood flow and
(possibly) increased intracranial pressure result from the
respiratory depressant actions of opioid analgesics. The
latter effects are due to increased arterial PCO2, which
results from mu receptor inhibition of the medullary
respiratory center. However, the activation of kappa
receptors contributes to analgesia at the spinal level and is
probably responsible forsedative actions of the opioids.
Source: Katzung & Trevor’s Pharmacology, Examination
and Board Review, 6th Ed., McGraw Hill, New York, 1998
2445. Answer: D
2446. Answer: E
Explanation:
Deliberate (Dealing)
Practitioner becomes a mercenary
Sells drugs for money, sex, street drugs, etc.
Office becomes a pill factory—full of drug seekers
ASIPP
518
Section 12 • Controlled Substance Management
Prescribes for known addicts who will likely sell drugs to
others
Source: Roger Cicala, MD, Sep 2005
State Board Investigation or Sanction
Often given re-education options
May result in databank report
2447. Answer: B
Explanation:
Reference: 21 C.F.R. § 1306.04 (prescriptions) and DEA
Guidance Document on Dispensing and Purchasing
Controlled Substances over the Internet, Fed. Reg. Vol. 66,
No. 82 (April 27, 2001);
http://www.deadiversion.usdoj.gov/fed_regs/notices/2001
/fr0427.htm.
Explanation:
A. This is not the correct answer. A physician may use the
Internet to provide information and to communicate with
the patient, but the Internet communications cannot be
the sole basis for authorizing the prescriptions. If a valid
physician-patient relationship exists, a physician may use
the Internet to communicate with patients. For example, a
physician may use the Internet to receive requests for
treatment. However, all requests for treatment should be
logical based upon the physician’s knowledge of the
patient’s medical history and the presenting complaint.
DEA states that, assuming a valid physician-patient
relationship, it is permissible for a physician to use the
Internet to receive requests for refills of prescriptions
from patients. Practitioners should check to see whether
their licensing state places additional requirements on
those who engage in telemedicine.
B. This is the best answer. A physician may use the
Internet to communicate with patients, but the Internet
communications may not form the sole basis for the
physician-patient relationship. The physician must
establish a valid physician-patient relationship with each
patient in accordance with federal and state laws governing
telemedicine. Most states have internet prescribing
policies requiring physicians to obtain a thorough medical
history and conduct an appropriate physical examination
before prescribing any medication for the first time.
C. This is not the correct answer. See explanations above.
D. This is not the correct answer. There is no such Internet
certification from the DEA allowing physicians to
prescribe controlled substances to patients over the
Internet.
E. This is not the correct answer. There is no such Internet
certification from the DEA.
Source: Jennifer Bolen, JD, Sep 2005
2448. Answer: C
Explanation:
Risks of Malprescribing
ASIPP
Usually results in adverse publicity
Attention attracts civil lawsuits
Source: Laxmaiah Manchikanti, MD
2449. Answer: D
Explanation:
Reference: DEA Interim Policy Statement, Fed. Reg. Vol.
69, No. 220, pp. 67170-7172 (Nov. 16, 2004).
Explanation:
A. This is not the best answer. Although the Interim Policy
Statement discusses the prohibition against the use of
“multiple schedule II prescriptions with different fill
dates,” the Interim Policy Statement refers to the
Controlled Substances Act of 1970’s prohibition against
refills of Schedule II controlled substances. Likewise, the
federal law on prescriptions for scheduled drugs is found
in the CFR. For the answer to be correct, you would have
to look to all three levels of federal materials on the subject
matter.
B. This is not the best answer. Although the Controlled
Substances Act of 1970 prohibits refills of Schedule II
prescriptions, it is the Interim Policy Statement that
actually discusses the prohibition against the use of
“multiple schedule II prescriptions with different fill
dates.” Likewise, the federal law on prescriptions for
scheduled drugs is found in the CFR. For the answer to be
correct, you would have to look to all three levels of federal
materials on the subject matter.
C. This is not the best answer. Although the Code of
Federal Regulations contains references to prescriptions
and the prohibition against refills on Schedule II
controlled substances, it is the Interim Policy Statement
that actually discusses the prohibition against the use of
“multiple schedule II prescriptions with different fill
dates.” Likewise, the federal law on prescriptions for
scheduled drugs is found in the CSA. For the answer to be
correct, you would have to look to all three levels of federal
materials on the subject matter.
D. This is the best answer. DEA discussed the prohibition
against the use of multiple schedule II prescriptions with
different fill dates in the Interim Policy Statement.
However, DEA makes reference to the CSA and the federal
law also involves the Code of Federal Regulations. Thus,
all three levels of federal materials apply.
E. This is not the correct answer. Federal law takes priority
519
Section 12 • Controlled Substance Management
over state law in this area. States may create laws and rules
that are stricter than the federal rule prohibiting the use of
multiple schedule II prescriptions with different fill dates,
but states may not permit their providers to use
prescriptions in this format - with “Do not fill before”
dates and language.
Source: Jennifer Bolen, JD, Sep 2005
2450. Answer: E
2451. Answer: A
Source: Jackson KC. Board Review 2003
2452. Answer: D
2453. Answer: B
Explanation:
Reference: The Federation of State Medical Boards’ May
2004 Model Policy for the Use of Controlled Substances
for the Treatment of Pain, www.fsmb.org.
Explanation:
A. This is not the best answer because it only involves
some of the records state licensing boards require
practitioners to keep when they prescribe controlled
substances for the treatment of pain.
B. This is the best answer. The Federation of State Medical
Boards’ Model Policy for the Use of Controlled Substances
for the Treatment of Pain sets forth the following
categories of medical records that practitioners should
keep when they treat pain. Many states have adopted these
categories in prescribing guidelines and, quite frequently,
in prescribing regulations or rules.
1. the medical history and physical examination,
2. diagnostic, therapeutic and laboratory results,
3. evaluations and consultations,
4. treatment objectives,
5. discussion of risks and benefits,
6. informed consent,
7. treatments,
8. medications (including date, type, dosage and
quantity prescribed),
9. instructions and agreements and
10. periodic reviews.
C. This is not the best answer because it only involves
some of the records state licensing boards require
practitioners to keep when they prescribe controlled
substances for the treatment of pain.
some of the records state licensing boards require
practitioners to keep when they prescribe controlled
substances for the treatment of pain.
Source: Jennifer Bolen, JD, Sep 2005
2454. Answer: A
Explanation:
A. This is the best answer. Unless a practitioner prescribes
controlled substances in the course of detoxification or
maintenance of opioid addiction, he/she is NOT
REQUIRED to keep the records described by the
Controlled Substances Act of 1970. However, most states
have Uniform Controlled Substances Act, a medical
practice act, regulations or rules, and/or guidelines or
policy statements (sometimes called position statements)
containing recordkeeping requirements for controlled
substances.
B. This is not the correct answer. The fact that a provider
prescribes at more than one location does not change
his/her obligation to follow federal and state controlled
substances recordkeeping mandates.
C. This is not the correct answer. A provider who uses the
Internet to interact with patients must do so pursuant to a
valid physician patient relationship. Likewise, providers
must follow federal and state controlled substances
recordkeeping mandates.
D. This is not the correct answer. It does not matter how
long a provider has been registered with the DEA. As long
as a practitioner holds a DEA registration, he/she is
responsible for following federal and state controlled
substances recordkeeping mandates.
E. This is not the correct answer. Using electronic medical
records does not excuse a provider from following federal
and state controlled substances recordkeeping mandates.
Source: Jennifer Bolen, JD, Sep 2005
2455. Answer: B
Explanation:
Benzodiazepines have no significant therapeutic benefit in
the management of obsessive-compulsive disorders.
Drugs effective for this condition increase the activity of
serotonergic systems in the brain. Clonazepam has been
used commonly as an anticonvulsant and also has efficacy
in anxiety states, including agoraphobia. Clonazepam (not
diazepam) has also been used as a back-up drug in bipolar
affective disorder.
Source: Katzung & Trevor’s Pharmacology, Examination
and Board Review, 6th Ed., McGraw Hill, New York, 1998
D. This is not the best answer because it only involves
some of the records state licensing boards require
practitioners to keep when they prescribe controlled
substances for the treatment of pain.
E. This is not the best answer because it only involves
ASIPP
520
Section 12 • Controlled Substance Management
2456. Answer: D
Explanation:
Duped
Always assumes the best about his patients and is gullible
Leaves script pads lying around
Hydrophilic medicine—fell into the toilet or the sink
Patients only want specific medications (i.e. OxyContin or
Percocet)
Co-dependent—cannot tell patients “No” when they ask
for narcotics
Source: Roger Cicala, MD, Sep 2005
2457. Answer: E
2458. Answer: E
Source: American Board of Anesthesilogy, In-trainnig
examination
2459. Answer: D
Explanation:
Urine Drug Testing
* Thin-layer chromatography (TLC)
- Relatively old technique, testing the migration of a
drug on a plate or film, which is compared to a known
control
* Gas chromatography: liquid and mass spectometry
(CGMS)
- Most sensitive and specific tests
- Most reliable
- Labor intensive/costly
- Several days to know results
- Used to confirm results of other tests
* Enzyme immunoassay
- Easy to perform/highly sensitive
- More sensitive than TLC
- Less expensive than GC/MS
- Common tests
EMIT (enzyme multiplied immunoassay test)
FPIA (fluorescent polarization immunoassay)
RIA (radioimmunoassay)
- Screen only one drug at a time
- Rapid drug screens
- Similar to other enzyme immunoassay tests
- May be more expensive
2460. Answer: B
Source: American Board of Anesthesilogy, In-trainnig
examination
2461. Answer: C
Explanation:
ASIPP
A. NMDA receptors require glutamate and glycine for
activation.
B. NMDA receptors require glycine, but also glutamate for
activiation.
C. NMDA receptors are calcium-permeable ion channels
that require both glutamate and glycine for activation.
The amino acid-binding sites on the receptor are
contributed by two different subunits, NR1 and NR2.
Antagonism of either of these two sites is sufficient to
completely block the ion flocks that normally follows
receptor activation.
D. At the molecular level, NMDA receptors are composed
of multiple subunits that co-assemble to form functional
channels.
NR1 subunits exist as a family of 8 splice variants
generated by alternative splice of 1N-terminal
cassette and to intracellular C-terminal cassettes.
The presence of 1 or more of the NR1 and NR2
subunits in a single receptor complex confers
unique biophysical and pharmacologic properties to
the NMDA receptor.
E. NMDA receptors have large intracellular C-termini
that interact with a variety of important proteins that
regulate receptor phosphorylation and clustering to
important signaling complexes.
2462. Answer: E
Explanation:
Propoxyphene will increase carbazepine, doxepin,
metoprolol and propranolol levels, and decrease the
excretion of benzodiazepines, leading to accumulation and
overdose. Amitriptyline will decrease morphine
breakdown, leading to increased blood levels. Morphine
will decrease the breakdown of desipramine, leading to
increased doses. Erythromycin will increase opioid effects,
and methadone will increase TCA levels.
References: Bergendal L, Friberg A, Schaffrath AM, et al,
The clinical relevance of the interaction between
carbamazepine and dextropropoxyphene in elderly
patients in Gothenburg,
Abernethy DR, Greenblatt DJ, Morse DS, et al, Interaction
of propoxyphene with diazepam, alprazolam and
lorazepam. Br J Clin Pharmacol 1985;19:51-7
Source: Andrea M. Trescot, MD
2463. Answer: C
2464. Answer: D
Source: Manchikanti L, Board Review 2005
521
Section 12 • Controlled Substance Management
2465. Answer: E
Explanation:
The hydromorphone is most likely from the 2D6
metabolism of hydrocodone, not from abuse. Discharging
the patient, refusing to write more opioids, or ignoring the
results would be inappropriate. SSRIs, especially
fluoxetine and paroxetine, will inhibit 2D6 and prevent the
metabolism of hydrocodone to hydromorphone, which
will decrease his analgesia.
Source: Andrea M. Trescot, MD
2466. Answer: A
Explanation:
Drug Test
Detection
Amphetamine or methamphetamine
Barbiturates (short-acting)
Barbiturates (long-acting)
Benzodiazepines
Cocaine (benzoylecgonine-cocaine metabolite)
Heroin or morphine
Marijuana (occasional use)
Marijuana (chronic use)
Methadone
Phencyclidine (occasional use)
Phencyclidine (chronic use)
2 to 4 days
2 to 4 days
Up to30 days
Up to 30 days
1 to 3 days
1 to 3 days
1 to 3 days
Up to 30 days
2 to 4 days
2 to 7 days
Up to 30 days
Source: Manchikanti L, Board Review 2005
2467. Answer: C
Explanation:
The Controlled Substances Act has divided drugs under its
jurisdiction into five schedules. Schedule I drugs have a
high potential for abuse and no accepted medical use in
the United States. Examples of schedule I drugs include
heroin, marijuana, LSD, etc.
A. Buprenorphine is Schedule III drug
B. Hydromorphone is Schedule II
C. Heroin is Schedule II
D. Diazepam is Schedule IV
E. Morphine is Schedule II
Source: Manchikanti L, Board Review 2005
2468. Answer: A
Explanation:
* No reliable long term studies (> 8 months) that
demonstrate efficacy and safety of long term opioid
therapy for chronic pain
* Overall relief with opioids is modest (35%)
* Physicians must be cautious when prescribing long term
opioids
* Outcomes are patient-specific
* Be aware of risk of adverse events
- Addiction; diversion; noncompliance
- Concomitant psychiatric illness
- Accidental overdose; etc
* What are your true practice costs?
Source: Mark V. Boswell, MD, KSIPP 2005
2469. Answer: C
Explanation:
A) Registration is required every year.
B) Schedule II drugs cannot be prescribed for narcotic
addiction.
C) This is correct.
D) Registration as an NTP is contingent on proper
registration with the State Methadone Authority and the
Department of Health and Human Services.
E) This is incorrect; a practitioner must obtain SAMHSA
certification.
Reference: 21 U.S.C. § 823(g).
Source: Erin Brisbay McMahon, JD, Sep 2005
2470. Answer: D
Explanation:
Answer (d) is wrong; the time limit is 3 weeks.
Reference: 42 CFR 8.11
Source: Erin Brisbay McMahon, JD, Sep 2005
2471. Answer: E
Explanation:
Opioids are unpredictable, and the blood levels needed for
analgesia vary with level of activity and time of day. Even
when a pain signal is below threshold, there are potential
physiologic effects. Early aggressive treatment is the goal
of preemptive analgesia.
Source: Andrea M. Trescot, MD
2472. Answer: E
Explanation:
Drug Abuse Prevention and Control Act of 1970
* Title 21, Chapter 13 of US Code
* Established current schedules, registrations, agencies,
enforcement and penalties.
* Little change in laws since that time, although
enforcement varies and case law has modified
interpretation.
Source: Roger Cicala, MD, Sep 2005
2473. Answer: D
Explanation:
Reference: Hardman, p 564.
ASIPP
522
Section 12 • Controlled Substance Management
A. The anxiolytic effects of buspirone take several days to
develop, obviating its use for acute severe anxiety.
B. Chloral hydrate a sedative, is used in the short-term
treatment of insomnia .
C. Chlorpromazine is used to treat psychotic disorders
and symptoms such as hallucinations, delusions, and
hostility
D. A long-acting benzodiazepine, such as diazepam, is
effective in blocking the secobarbital withdrawal
symptoms.
E. Trazodone is an anti-depressant
Source: Stern - 2004
2474. Answer: A
Explanation:
Reference: 21 U.S.C. § 829; 21 C.F.R. § 1306.01 through §
1306.26; www.deadiversion.usdoj.gov.
Explanation:
A. This is the best answer. Only practitioners acting in the
usual course of their professional practice may prescribe
controlled substances. Practitioners must be registered
with DEA and licensed to prescribe controlled substances
by the State(s) in which they operate. In addition, if the
state requires a separate state DEA registration number,
practitioners must obtain this registration prior to
applying for a federal DEA registration. Finally, a
prescription must be issued in the usual course of
professional practice and for a legitimate medical purpose
(or authorized research).
B. This is not the correct answer because it omits the
requirement of (1) legitimate medical purpose within (2)
the usual course of professional practice.
2475. Answer: D
Explanation:
Benzodiazepines are thought to exert most of their CNS
effects by increasing the inhibitory effects of GABA.
Benzodiazepines interact with specific receptors (BZ
receptors) that are components of the GABA A receptorchloride ion channel macromolecular complex to increase
the frequency of chloride ion channel opening.
Benzodiazepines are not GABA receptor agonists because
they do not interact directly with this component of the
complex.
Source: Katzung & Trevor’s Pharmacology, Examination
and Board Review, 6th Ed., McGraw Hill, New York, 1998
2476. Answer: E
Explanation:
Decreased blood flow to vital organs, including the liver
and kidney, occurs during the aging process. These
changes may contribute to cumulative effects of sedativehypnotic drugs. However, this does not explain the
enhanced sensitivity of the elderly patient to a single dose
of central depressant, which appears to be due to changes
in brain function that accompany aging.
Source: Katzung & Trevor’s Pharmacology, Examination
and Board Review, 6th Ed., McGraw Hill, New York, 1998
2477. Answer: C
Explanation:
The elimination of most benzodiazepines involves their
metabolism by liver enzymes, including cytochrome P450
isozymes. In a patient with liver dysfunction, lorazepam,
which is metabolized extrahepatically, is less likely to
cause excessive CNS depression. Benzodiazepines are not
eliminated via the kidneys or lungs. Flumazenil is used to
reverse excessive CNS depression caused by
benzodiazepines.
Source: Katzung & Trevor’s Pharmacology, Examination
and Board Review, 6th Ed., McGraw Hill, New York, 1998
2478. Answer: C
Explanation:
Addiction is a chaotic disturbance in physical and
psychological control factors that involve impulse control,
and often evolves to a patient utilizing the drug to their
D. This is not the correct answer because the federal law
detriment, inducing physical harm, and personal disregard
does not contain a training requirement currently. Note,
of danger. There is a significant loss of personal control,
however, this may change in the near future due to the
and the patient seeks the drug, sometimes at all costs.
growing abuse and diversion of prescription controlled
Addiction does not necessarily mean a non-functional
drugs in the United States. Several states require some
individual. We see some levels of addiction, even with our
level of training in the use of controlled substances to treat
very straightforward pain control patients, i.e. tobacco use.
pain.
Functional alcoholics perform in some segments of
society, and it is not uncommon to be introduced to a
E. This is not the correct answer because it omits two
patient with an iatrogenic addiction to a controlled
elements: (1) the state DEA registration number (where
substance such as benzodiazepine (Xanax?), opioid,
required), and (2) legitimate medical purpose within the
Oxycontin?. It is also not unusual to hear that some of
usual course of professional practice.
these patients are very highly respected members of
Source: Jennifer Bolen, JD, Sep 2005
society, kind and caring, and have had personality changes
that are not appreciated by the individual. This is where
professional and family intervention is necessary.
C. This is not the correct answer because it omits the
requirement of a state DEA registration number, which
many states require.
ASIPP
523
Section 12 • Controlled Substance Management
Source: Hans C. Hansen, MD
2479. Answer: A
Explanation:
Reference: Hardman, pp 16-20.
A. Sodium Bicarbonate is excreted principally in the urine
and alkalinizes it. Increasing urinary pH interferes with
the passive renal tubular reabsorption of organic acids
(such as aspirin and Phenobarbital) by increasing the ionic
form of the drug in the tubular filtrate. This would
increase their excretion.
intolerable side effects.
Source: Trescot AM, Board Review 2004
2485. Answer: A
2486. Answer: B
Explanation:
The non-medical use of pain medication experienced its
largest growth between 1995 and 2000, and has somewhat
slowed in the past couple of years.
Source: Hans C. Hansen, MD
B-E. Excretion of organic bases (such as amphetamine,
cocaine, phencyclidine, and morphine) would be enhanced 2487. Answer: E
by acidifying the urine.
Explanation:
Source: Stern - 2004
A withdrawal syndrome with symptoms consistent with
increased sympathetic activity will occur following the
2480. Answer: C
abrupt discontinuation of long-term opioid use.
Explanation:
Hyperalgesia is associated with continued use of high dose
Answer (c) is wrong because, under 21 CFR 1306.05, the
opiates. Mu receptor affinity is an inherent pharmacologic
physician is responsible if the prescription does not
property of the opioid and would not be expected to
conform to applicable laws and regulations.
change with its discontinued use. Tolerance will be
expected to diminish with discontinuation of use, while
Reference: 21 CFR 1306.05.
physical dependence is a physiologic state in which abrupt
Source: Erin Brisbay McMahon, JD, Sep 2005
cessation of opioid results in a withdrawal syndrome.
2481. Answer: C
Source: Nader and Candido – Pain Practice. June 2001
2482. Answer: E
Source: Laxmaiah Manchikanti, MD
2483. Answer: D
Explanation:
A. Naltrexone is an antagonist therapy for heroin
addiction
B. Physostigmine is used to treat glaucoma
C. Pralidoxime is used together with another medicine
called atropine to treat poisoning caused by organic
phosphorus pesticides
D. Flumazenil is a competitive antagonist of
benzodiazepines at the GABA receptor. Repeated
administration is necessary because of its short half-life
relative to that of most benzodiazepines.
E. Naloxone is an opioid antagonist.
2484. Answer: D
Explanation:
There is no ceiling for opioids (other than the limitations
of agonist/antagonists or APAP). The goal is to prevent
addiction. Tolerance is less likely with long acting opioids.
Respiratory depression is unlikely with stable doses of
opioids. The goal is a balance between pain relief and
Stoelting RK. Pharmacology and Physiology in Anesthesia
Practice. 3rd ed. Lippincott-Raven, Philadelphia, 1999.
Savage SR. Critical clinical issues in pain and addiction.
Pain Management Rounds 2005; 2(9):
Source: James D. Colson, MS, MD
2488. Answer: E
Explanation:
The next course of action is to explore the issues with the
patient.
2489. Answer: A
Explanation:
The rate of continued abstinence after simple
detoxification is about 2 to 3%. Prolonged treatment and
aftercare markedly increase the success rate.
Source: Roger Cicala, MD, Sep 2005
2490. Answer: E
Explanation:
There is no evidence that any SSRI is more effective than
another in its antidepressant efficacy. While an individual
patient may respond more favorably to a specific drug,
several controlled studies have shown equivalent effective
ness of these agents. However, SSRIs may be more effective
than tricyclic antidepressants in some patients.
Source: Katzung & Trevor’s Pharmacology, Examination
and Board Review, 6th Ed., McGraw Hill, New York, 1998
2491. Answer: D
Explanation:
ASIPP
524
Section 12 • Controlled Substance Management
Urine Drug Testing
Know the characteristics of testing procedures, since many
drugs are not routinely detected by all UDTs.
Although no aberrant behavior is pathognomonic of
abuse or addiction, such behavior should never be ignored.
Reliance on aberrant behavior to trigger a UDT will miss
more than 50% of those individuals using unprescribed or
illicit drugs.
Never prescribe “on-demand” for the patient until you are
comfortable with the situation.
A history of drug abuse does not preclude treatment with
a controlled substance, when indicated, but does require a
treatment plan with firmly defined boundaries.
Source: Laxmaiah Manchikanti, MD
2492. Answer: E
Explanation:
Watch for signs:
Progressive deterioration in personal appearance and
hygiene;
Uncharacteristic deterioration of handwriting and
charting;
QRS complex on the ECG is a major diagnostic feature of
cardia toxicity. Arrhythmias resulting from cardiac
conductivity (eg, lidocaine). There is no evidence that
hemodialysis (or hemoperfusion) increases the rate of
elimination of tricyclic antidepressants, presumably
because of their large volume of distribution and their
binding to tissue components.
Source: Katzung & Trevor’s Pharmacology, Examination
and Board Review, 6th Ed., McGraw Hill, New York, 1998
2496. Answer: E
Explanation:
Watch for signs:
Absenteeism, frequent disappearances or long unexplained
absences, making improbable excuses and taking frequent
or long trips to the bathroom or to the stockroom where
drugs are kept;
Excessive amounts of time spent near a drug supply.
Volunteer for overtime and at work when not scheduled;
Unreliability in keeping appointments and meeting
deadlines;_
Source: Erin Brisbay McMahon, JD, Sep 2005
2497. Answer: D
Explanation:
Food and Drug Acts
Wearing long sleeves when inappropriate;
Personality change - mood swings, anxiety, depression,
lack of impulse control, suicidal thoughts or gestures;
Patient and staff complaints about health care provider’s
changing attitude/behavior;
Increasing personal and professional isolation.
Source: Erin Brisbay McMahon, JD, Sep 2005
2493. Answer: D
Explanation:
Tricyclic drugs block the uptake of guanethidine into
sympathetic nerve endings, thus reversing its beneficial
effects on blood pressure. While the precise mechanism is
not defined, the tricyclics may also block the
antihypertensive effects of clonidine and methyldopa. All
of the other drug interactions have been reported.
Source: Katzung & Trevor’s Pharmacology, Examination
and Board Review, 6th Ed., McGraw Hill, New York, 1998
2494. Answer: D
2495. Answer: B
Explanation:
Tricyclic antidepressant overdose is a medical emergency.
The “three Cs”- coma, convulsions, and cardiac problemsare the most common causes of death. Widening of the
ASIPP
* 1906 - Wiley Act
- Founded the F. D. A.
- Centered on foods and meat packing
- Required ingredients and concentration standards for
all medications - largely to regulate patent medicines.
* 1938 - Food, Drugs, and Cosmetics Act
- Mandated premarket approval of drugs
- Required proof of safety
- Prohibited false therapeutic claims
* Marijuana Tax Act 1938
Source: Roger Cicala, MD, Sep 2005
2498. Answer: B (1 & 3)
Explanation:
Tramadol is metabolized by CYP2D6 to an active
metabolite that has more effect than the parent
compound, so that CYP2D6 inhibitors cause the effective
analgesia to decrease. However, CYP2D6 is also the
enzyme responsible for the excretion of tramadol, so that
inhibition leads to decreased excretion and the increased
possibility for seizures.
Source: Andrea M. Trescot, MD
2499. Answer: A (1,2, & 3)
Explanation:
Grapefruit inhibits CYP3A4, not 2D6, and has its action by
525
Section 12 • Controlled Substance Management
altering liver and intestinal 3A4 as well as inhibiting
intestinal transport by p-GP. Medicines with a low
bioavailability have the potential for increased absorption.
Source: Andrea M. Trescot, MD
2500. Answer: B (1 & 3)
Explanation:
Lidocaine is highly dependant upon liver excretion, while
methadone is not. Codeine is metabolized to morphine,
which is highly dependant upon liver excretion (though
M3G accumulates in renal insufficiency), while
meperidine requires renal excretion.
Source: Andrea M. Trescot, MD
2501. Answer: C (2 & 4)
Explanation:
Tramadol’s M1 metabolite has greater activity. Morphine
would have decreased metabolism in liver failure, but the
metabolites accumulate in renal failure.
Source: Andrea M. Trescot, MD
2502. Answer: D (4 only)
Explanation:
Caffeine (in coffee and tea) is metabolized by CYP1A2,
which is inhibited by ciprofloxin. Phenobarbital will
increase the risk of folate deficiency neuropathy. Smokers
metabolize propoxyphene faster and therefore have less
effect.
Source: Andrea M. Trescot, MD
2503. Answer: E (All)
Explanation:
The opiate immunoassay screens were designed to detect
heroin abuse, not adherence to a therapeutic opioid
regimen. These assays detect morphine and
codeine—heroin is rapidly metabolized to 6monoacetylmorphine (6-MAM) and then to morphine.
Performing opiate immunoassays at the federally
mandated level of 2000 ng/ml should eliminate nearly all
positive results due to morphine from foodstuffs. Only
specific detection of 6-MAM by GC/MS is proof of heroin
intake. Street heroin may be contaminated with codeine.
Remember that codeine may be metabolized to morphine.
Hydrocodone can also be produced as a minor metabolite
of codeine. J Anal Toxicol 2000;24:530-535
Source: Art Jordan, MD, Sep 2005
2504. Answer: E (All)
Explanation:
All of these actions can influence the drug-drug
interaction.
Source: Andrea M. Trescot, MD
2505. Answer: A (1,2, & 3)
Explanation:
Isoniazid and carbamazepine will increase the metabolism
of methadone (leading to decreased levels), while
cimetidine will slow down the metabolism (leading to
increased levels. While many CYP enzymes show ethnic
differences, methadone metabolism has not been
documented to have an ethnic variability.
Source: Andrea M. Trescot, MD
2506. Answer: D (4 only)
Explanation:
Drug clearance and plasma half-life are pharmacokinetic,
not pharmacodynamic, variables associated with the
elimination of drug through renal excretion and/or
hepatic metabolism.Bioequivalence compares different
formulations of the same drug and is not the same as
bioavailability or the fraction of a drug absorbed
following administration.Potency is a measure of the
amount or dose of drug required for an effect.
Evers AS, Maze M. Anesthetic Pharmacology: Physiologic
Principles and Clinical Practice. Churchill Livingstone,
Philadelphia, 2004.
Hardman JG, Limbird LE. Goodman and Gilman’s
Pharmacological Basis of Therapeutics. 10th ed. McGrawHill, New York, 2001.
Source: James D. Colson, MS, MD
2507. Answer: B (1 & 3)
Explanation:
A documented abnormality/pathology on MRI may be an
incidental finding and not causing pain. You should make
your decision based on examination, previous medical
records, imaging studies, and careful evaluation.
Just because another physician has been dispensing
narcotics to a patient, does not necessarily mean it was
appropriate or legal. The dispensing/prescribing must be
based on a legitimate medical purpose as determined by
the current prescribing physician, after careful evaluation
and examination.
Source: Art Jordan, MD, Sep 2005
2508. Answer: C (2 & 4)
Explanation:
The paleospinothalamic tract is rich in opioid receptors,
connects the thalamus to the cortex and reticular
activation system, and passes impulses from the C-fibers
to the 2nd order neurons.
Source: Andrea M. Trescot, MD
2509. Answer: E (All)
Explanation:
A patient compliance problem might suggest the choice of
a once a day medication or patch. If a patient is on other
BID mediations, a BID pain medication might be the best
choice. Drug interactions can affect choice of medication,
such as when a patient is already on Fiorinal, which will
decrease methadone levels. Side effects such as nausea
from hydroxylated opioids or constipation that might be
lessoned by a patch can also influence medication choice.
ASIPP
526
Section 12 • Controlled Substance Management
Source: Andrea M. Trescot, MD
2510. Answer: E (All)
Explanation:
Other options include changing route and adding adjuvant
meds.
Source: Andrea M. Trescot, MD
2511. Answer: C (2 & 4)
Explanation:
Morphine is primarily hepatically metabolized by
glucuronidation
Source: Andrea M. Trescot, MD
2512. Answer: A (1,2, & 3)
Source: Andrea M. Trescot, MD
2513. Answer: A (1,2, & 3)
2514. Answer: A (1,2, & 3)
Explanation:
Urine Drug Test Methods Opioids: Pitfalls
Tests for opiates are very responsive for morphine and
codeine . Do not distinguish which is present.
Show a low sensitivity for semisynthetic/synthetic opioids
such as oxycodone. A negative response does not exclude
oxycodone, or methadone use.
Source: Laxmaiah Manchikanti, MD
2515. Answer: B (1 & 3)
Explanation:
Urine Drug Test Methods
Amphetamines: Low Specificity
Tests for amphetamine/methamphetamine are highly
cross-reactive.
They will detect other sympathomimetic amines such as
ephedrine and pseudoephedrine
Not very predictive for amphetamine/methamphetamine
use.
Further testing is required.
Source: Laxmaiah Manchikanti, MD
2516. Answer: E (All)
Explanation:
http://buprenorphine.sahsa.gov/waiver _qualifications
21 U.S.C 823 (g)
Source: James D. Colson, MS, MD
2517. Answer: B (1 & 3)
Explanation:
Only dexmedetomidine preserves respiratory function
without potentiating opioid-induced respiratory
ASIPP
depression.
Only dexmedetomidine possesses selective alpha-2
adrenergic agonistic activity, while midazolam works
through a GABA2 receptor mechanism.
Evers AS, Maze M. Anesthetic Pharmacology: Physiologic
Principles and Clinical Practice. Churchill Livingstone,
Philadelphia, 2004.
Morgan GE, Mikhail MS, Murray MJ. Clinical Anesthesia.
3rd ed. McGraw-Hill, New York, 2002.
Source: James D. Colson, MS, MD
2518. Answer: E (All)
Explanation:
Norman R, Wells R, Neumann P, et al. A comparison of
peak vs cumulative physical loading factors for reported
low back pain in the automobile industry, Clinical
Biomechanics, 13(8): 561-573, 1998.
Source: Andrea M. Trescot, MD
2519. Answer: B (1 & 3)
Explanation:
Physical dependence is an occurrence that follows opioid,
benzodiazepine, or other controlled substance use, and
sometimes non-controlled substances, such as nicotine
and alcohol, particularly revealed when abruptly
discontinued. Titration is acceptable, and done slowly,
with the caveat that physicians that do not have a special
attachment to their DEA certificate do not use methadone
as a taper. Opioid antagonist drugs can abruptly throw
an individual into withdrawal.
Source: Hans C. Hansen, MD
2520. Answer: A (1,2, & 3)
Source: Stimmel, B
2521. Answer: E (All)
Explanation:
Federation of State Medical Boards Guidelines for the
Treatment of Pain
Use of controlled substances, including opiates may be
essential in the treatment of pain
Effective pain management is a part of quality medical
practice
Patients with a history of substance abuse may require
monitoring, consultation, referral and extra
documentation
MD’s should not fear disciplinary action for legitimate
medical purposes
Source: Laxmaiah Manchikanti, MD
2522. Answer: E (All)
Source: Mark V. Boswell, MD, KSIPP 2005
2523. Answer: B (1 & 3)
527
Section 12 • Controlled Substance Management
Explanation:
1.Phencyclidine was developed as an intravenous
anesthetic. However, the unique anesthesia it produced
was complicated by a prolonged emergence delirium,
leading to its demise as a clinically useful agent.
Phencyclidine also caused symptoms of sensory
deprivation, which is an excellent drug model of
schizophrenia.
2.The desirable anesthetic properties of phencyclidine
were retained in the short-acting arylcyclohexylamine
derivative ketamine or Ketalar, which produced a much
briefer emergence delirium. The term “dissociative
anesthetic” was coined to emphasize the anesthetized
patient was “disconnected” from his or her environment.
Ketamine subsequently was discovered by the drug abuse
community, where it is known as K, Super K, Special K,
and Kat Valium. Phencyclidine has been placed in
Schedule I of the Federal Controlled Substances Act, and
Ketamine in Schedule II.
3.MK-801 was developed as an anticonvulsant and
subsequently was used as a brain protective agent.
However, it was discarded because of its PCP-like effects.
Clinical trials of MK-801 have been extremely limited,
and the results are not publicly available.
4.Dextromethorphan is an antitussive agent. When taken
in very large amounts, it produces dysphoric mental effects
that can be related to its weak NMDA antagonistic
properties.
5.Amantadine and a related compound, memantidine,
have been shown to be weak NMDA receptor antagonists.
2524. Answer: B (1 & 3)
Explanation:
Alcohol absorption is slowed by food, but increased by
water, especially if carbonated. Alcohol goes directly into
the bloodstream from the stomach and it is distributed
throughout all tissues of the body. If somach alcohol
concentration becomes too high, mucus is secreted and
the pyloric valve closes, thereby slowing absorption.
Source: Psychiatry specialty Board Review By William M.
Easson, MD and Nicholas L. Rock, MD
2525. Answer: C (2 & 4)
Source: Reddy Etal. Pain Practice: Dec 2001, march 2002
2526. Answer: E (All)
Explanation:
The prefered medications for the management of alcohol
withdrawal delirium are the benzodiazepines
(chlordiazepoxide, diazepam, lorazepam, oxyazepam).
Multivitamins, particularly thiamine, B12, and folic acid,
should be used. Thiamine IV or IM should be given prior
to glucose loading. If seizures develop, using magnesium
sulfate. Clonidine, propranolol, chloral hydrate,
benzodiazepines, or barbiturates can be used dependin on
the total clinical picture.
Source: Psychiatry specialty Board Review By William M.
Easson, MD and Nicholas L. Rock, MD
2527. Answer: A (1,2, & 3)
Source: Stimmel B
2528. Answer: B (1 & 3)
Explanation:
1. Diacetyl morphine or heroine was first synthesized in
1874. It was produced in 1989 by the Bayer Company and
marketed under the name heroin. Heroin is synthesized
from morphine by acetylation at both the 3 and 6 position.
2. Heroin is more water soluble and also more potent than
morphine.
3. Heroin is metabolized in humans by de-acetylation to
6-mono-acetylmorphine and then further metabolized to
morphine
4. Heroin is classified as Schedule I drug. It is not
available for any therapeutic use in the United States. It is
prescribed in a few other countries as a pain medication or
for use in the management of heroin addiction.
2529. Answer: E (All)
Explanation:
Reference: Compton, et al. Am J Psychiatry 2001; 160:
890-895.
Co-existing Diseases and Drug Dependence
Alcohol dependence (63%)
Any psychiatric disorder (74%)
Antisocial personality disorder (44%)
Phobic disorder (41%)
Major depression (25%)
Source: Mark V. Boswell, MD, KSIPP 2005
2530. Answer: A (1,2, & 3)
Explanation:
The Controlled Substances Act (CSA), title 2 of the
Comprehensive Drug Abuse Prevention and Control Act
of 1970 is the legal foundation of the government’s fight
against the abuse of drugs and other substances. This law
is a consolidation of numerous laws regulating the
manufacture and distribution of narcotics, stimulants,
depressants, hallucinogens, anabolic steroids, and
chemicals used in the illicit production of controlled
substances.
All the substances that are regulated under existing federal
law are placed into I of V schedules. This placement is
based upon the substances’ medicinal value, harmfulness,
and potential for abuse or addiction.
Schedule I is reserved for the most dangerous drugs that
have no recognized medical use.
Schedule V is the classification used for the least
dangerous drugs.
The Act also provides a mechanism for substances to be
ASIPP
528
Section 12 • Controlled Substance Management
controlled, added to a schedule, decontrolled, removed
from control, rescheduled, or transferred from one
schedule to another.
Source: Manchikanti L, Board Review 2005
2531. Answer: A (1,2, & 3)
Source: Stimmel, B
2532. Answer: E (All)
Source: Stimmel, B
2533. Answer: E (All)
Explanation:
A slow metabolizer cannot detoxify drugs quickly, leading
to increased toxicity and possibly increased effect. If the
drug needs to be metabolized for increased effect (such as
with a pro-drug), slow metabolizers will have a decreased
effect. Since metabolism is necessary for most excretion,
slow metabolizers would have decreased excretion.
Source: Andrea M. Trescot, MD
2534. Answer: E (All)
Source: Arthur Thexton
2535. Answer: B (1 & 3)
Explanation:
Urine Drug Test Methods
Cocaine: Very specific
You might also want to order a serum oxycodone level to
get an idea of what the doses are achieving.
Reference: UDT in Clinical Practice: Purdue Pharma, 2005
Source: Art Jordan, MD, Sep 2005
2538. Answer: A (1,2, & 3)
Explanation:
There are restrictions on methadone and buprenorphine
use ONLY when prescribed to treat addiction; when
prescribed for pain or other conditions, they may be
prescribed by anyone authorized to prescribe other
controlled substances in the same schedule.
Source: Arthur Thexton
2539. Answer: B (1 & 3)
Source: Arthur Thexton
2540. Answer: E (All)
Source: Stimmel, B
2541. Answer: E (All)
Source: Arthur Thexton
2542. Answer: E (All)
Source: Stimmel, B
2543. Answer: E (All)
Source: Reddy Etal. Pain Practice: Dec 2001, march 2002
Tests for cocaine react principally with cocaine and its
primary metabolite, benzoylecgonine.
2544. Answer: A (1,2, & 3)
Source: Laxmaiah Manchikanti, MD
These tests have low cross-reactivity with other substances
2545. Answer: C (2 & 4)
Source: Stimmel, B
Very specific in predicting cocaine use.
Source: Laxmaiah Manchikanti, MD
2546. Answer: E (All)
Source: Stimmel, B
2536. Answer: C (2 & 4)
Explanation:
Drug interactions include drug-drug, drug-food, and
drug-condition interactions. Drug allergies influence the
choice of medicines, and drug doses are important in drug
treatment, but neither are specifically related to drug
metabolism.
Source: Andrea M. Trescot, MD
2537. Answer: D (4 Only)
Explanation:
Standard urine opiate immunoassay is designed to detect
only morphine and codeine, and will not detect
oxycodone.
GC/MS (Gas chromatography/mass spectrometry)will
specifically detect oxycodone, or other specific substances
as you indicate to the lab.
2547. Answer: D (4 Only)
Explanation:
A small number of patients lack cytochrone P450 2D6 and
cannot convert part of the codeine to morphine, a normal
metabolite of codeine
Source: Art Jordan, MD, Sep 2005
2548. Answer: D (4 only)
Explanation:
Gabapentin, morphine, and baclofen have their primary
action as the parent drug. Morphine’s metabolite, M6G,
has analgesic activity, but a pro-drug is a drug whose
parent compound has no activity. Codeine, which is a prodrug, has no activity until it is metabolized into morphine.
Source: Andrea M. Trescot, MD
2549. Answer: D (4 only)
Source: Arthur Thexton
2550. Answer: A (1, 2 & 3)
ASIPP
529
Section 12 • Controlled Substance Management
Explanation:
Explanation:
4 is incorrect. A physician may issue multiple
prescriptions authorizing the patient to receive a total of
up to a 90-day supply of a Schedule II drug.
Source: 71 Fed. Reg. 52,724.
Source: Erin Brisbay McMahon, JD
2551. Answer: B (1 & 3)
Source: Laxmaiah Manchikanti, MD
2552. Answer: B (1 & 3)
Explanation:
1. The term “addict” by CSA means any individual who
habitually uses any narcotic drug so as to endanger the
public morals, health, safety, or welfare, or who is so far
addicted to the use of narcotic drugs as to have lost the
power of self-control with reference to his or her
addiction.
2. There is no definition for addiction in DSM-IV.
DSM-IV defines substance abuse with at least 1 in 12
month period.
3. Maladaptive pattern leading to distress or impairment.
Recurrent failure to field role.
Recurrent physically undesirous behavior.
Recurrent legal problems.
Continued use despite social problems.
Never met dependence criteria
DSM-IV definition for substance dependence is as follows:
Tolerance
Withdrawal
Larger Amounts/Longer periods
Efforts or desire to cut down
Large amount of time using/obtaining/recovering
Activities given up: social/work/recreation
Continued use despite problems
Need 3 of above in 12 months
An alternate definition from the American Society of
Addiction Medicine for addiction is as follows:
Addiction
A primary, chronic neurobiologic disease with
genetic, psychosocial and environmental factors
effecting its course and presentation
Characterized by one or more of the following
Impaired control of drug use
Compulsive use
Craving
Continued use despite harm
4. The federation recommends several additional steps in
patients with addiciton or abuse
Source: Manchikanti L, Board Review 2005
2553. Answer: B (1 & 3)
Explanation:
1, 2. Schedule I
The drug or other substance has a high potential for
abuse.
The drug or other substances has no currently accepted
medical use in treatment in the United
States.
There is a lack of accepted safety for use of the drug or
other substance under medical
supervision.
3. Schedule II
The drug or other substance has a high potential for
abuse.
The drug or other substances has no currently accepted
medical use in treatment in the United
States or a currently accepted medical use with severe
restrictions.
Abuse of the drug or other substances may lead to
severe psychological or physical dependence.
Other
Schedule III
The drug or other substance has a potential for abuse
less than the drugs or other substances in
schedules I and II.
The drug or other substances has no currently accepted
medical use in treatment in the United
States.
Abuse of the drug or other substance may lead to
moderate or low physical dependence or high
psychological dependence.
Schedule IV
The drug or other substance has a low potential for
abuse relative to the drugs or other
substances in schedule III.
The drug or other substance has a currently accepted
medical use in treatment in the United
States.
Abuse of the drug or other substances may lead to
limited physical dependence or psychological
dependence relative to the drugs or other substances in
schedule III.
4. Schedule V
The drug or other substance has a low potential for
abuse relative to the drugs or other
substances in schedule IV.
The drug or other substance has a currently accepted
medical use in treatment in the United
States.
Abuse of the drug or other substances may lead to
limited physical dependence or psychological
dependence relative to the drugs or other substances in
schedule IV.
Source: Manchikanti L, Board Review 2005
ASIPP
530
Section 12 • Controlled Substance Management
2554. Answer: A (1, 2 & 3)
Explanation:
1. The CSA created a closed system of distribution for
those authorized to handle controlled substances.
2. The system is the registration of all those authorized by
the DEA to handle controlled substances.
3. Only the individuals and practices that dispense directly
to the patients from their clinics are required to maintain a
DEA license.
4. All individuals and firms that are registered are
required to maintain complete and accurate inventories
and records of all transactions involving controlled
substances, as well as the security for the storage of
controlled substances.
The attorney general may limit revocation or
suspension of a registration to the particular
controlled substance. However, the Board of Medical
Licensure may also limit this indirectly by
means of requesting the limitation by DEA and
reaching an agreement with the practitioner.
Source: Manchikanti L, Board Review 2005
2555. Answer: A (1,2, & 3)
Explanation:
Opioids increase colonic motility but reduce peristaltic
acitivity; in addition, the defecation reflex is impaired.
Bulk laxatives are a poor choice because adequate water
intake is required and the bulk can make the stool hard
and precipate impaction.
Source: Oxford Textbook of Palliative Medicine, 2nd Ed
Schedule an appointment to discuss abnormal/unexpected
results with the patient; discuss in a positive, supportive
fashion to enhance readiness to change/motivational
enhancement therapy (MET) opportunities.
Use results to strengthen physician-patient relationship
and support positive behavior change.
Chart results and interpretation.
Source: Laxmaiah Manchikanti, MD
2558. Answer: A ( 1, 2, & 3)
Explanation:
Morphine is in the same class of opioids (phenanthrenes)
as oxycodone, but morphine has a 6-OH group (associated
with more nausea). Fentanyl, meperidine, propoxyphene,
and methadone are completely different classes of opioids
Source: Trescot AM, Board Review 2004
2559. Answer: A (1,2, & 3)
Explanation:
Ballantyne J, Mao J. Opioids for Chronic Pain. NEJM.
2003; 349: 1943-1953
Source: Mark V. Boswell, MD, KSIPP 2005
2560. Answer: E (All)
Explanation:
* Prolonged, high dose therapy may have adverse
consequences
* The opioid formulation does not reduce development of
tolerance
* Abuse potential of long acting and short acting
formulations are the same
2556. Answer: E (All)
Explanation:
Clinical Recommendations
* Limit the dose
- Daily doses above 180 mg/day of morphine have not
been validated
- Dose escalation beyond the stabilization phase may
predict a problem
* Drug formulations
- Formulation does not influence tolerance
* Long term opioids produce adverse physiologic changes
(immune, hormonal, pain, etc)
* Opioid rotation may help reduce the need for dose
escalation and improve efficacy
Source: Mark V. Boswell, MD, KSIPP 2005
2561. Answer: E (All)
Explanation:
Continued controlled substance prescribing requires:
* Diagnosis
* Opioid rotation
- Concept of incomplete cross-tolerance
- Rotation may restore efficacy
Source: Mark V. Boswell, MD, KSIPP 2005
2557. Answer: B (1 & 3)
Explanation:
UDT results:
Consult with laboratory regarding ANY unexpected
results.
ASIPP
* Reduction in pain
* Improvement in functional status
* Lack of evidence of drug abuse
* Documented informed consent
Source: Mark V. Boswell, MD, KSIPP 2005
2562. Answer: C (2 & 4)
531
Section 12 • Controlled Substance Management
Explanation:
Half life is 12 hours, as compared to diazepam which is
21-37 hours. Is indicated for short term use of anxiety. The
kinetics are linear.
It is converted to lorazepam glucuronide (75%) abd NOT
to benzoylecgonine, a major metabolite of cocaine.
Reference: Disposition of Toxic Drugs and Chemicals in
Man. Fifth Edition. Randall C. Baselt. 2000 page 483
Source: Art Jordan, MD, Sep 2005
2563. Answer: A (1,2, & 3)
Explanation:
1, 2, & 3. Random drug screens, narcotic contracts, and
aggressive refill policies (no early refills) have been felt to
help control aberrant drug behaviors.
4. Opioid rotation tries to address the issue of drug
tolerance.
Source: Trescot A, Board Review 2003
2564. Answer: A (1, 2, & 3)
Explanation:
Alcoholism is the third largest health problem after heart
disease and cancer. In males 25-44 years old, alcohol plays
a major role in all four leading causes of death: accidents,
homicides, suicides, and alcoholic cirrhosis. The chronic
use of alcohol produces psychological, interpersonal, and
medical problems, which include violence, absence from
work, loss of job, and legal difficulties. Alcohol is a factor
associated with at least 50% of traffic fatalities, 50% of
homicides, and 25% of suicides.
Source: Psychiatry specialty Board Review By William M.
Easson, MD and Nicholas L. Rock, MD
2565. Answer: E (All)
Explanation:
Explanation:All of these are condemned behaviors; the
more occurring in combination, the more likely a
conviction is.
Source: 71 Fed. Reg. 52,724.
Source: Erin Brisbay McMahon, JD
2566. Answer: C (2 & 4)
Source: Cole EB, Board Review 2003
2567. Answer: A (1,2, & 3)
Explanation:
Patients being maintained with methadone require special
consideration for acute pain management in surgical or
trauma situations.
1. Maintenance patients develop full tolerance to the
analgesic effects of the maintenance dose of methadone.
2.During opioid maintenance treatment, a cross-tolerance
develops to all opioid agonists drugs, accounting for the
“blockade effect. Early research has demonstrated that
stable opioid maintenance treatment patients could not
distinguish 20 mg of intravenous morphine from
intravenous saline.
3. The usual maintenance dose does not provide any
analgesia, and adequate analgesia will require higher doses
of opioid agonists given more frequently than in the nontolerant patient.
Methadone has a half-life of 24 to 36 hours, but its
analgesic effects range from 4 to 6 hours,
which is similar to morphine in both potency and
duration.
Morphine, Dilaudid, codeine, and other agonist drugs
are appropriate for opioid maintenance
treatment patients.
Mixed agonist-antagonists (pentazocine, butorphanol,
nalbuphine) and partial agonists
(buprenorphine) must not be used, as they will
precipitate an opioid withdrawal syndrome.
Meperidine and propoxyphene should be avoided
because of the risk of seizures at the higher
doses required to produce analgesia in these patients.
4. Maintenance doses of opioids do not provide adequate
analgesia in acute pain.
2568. Answer: D (4 Only)
Explanation:
Alcohol withdrawal delirium (delirium tremens, DT’s) is
characterized by confusion, disorientation, fluctuating or
clouded consciousness, perceptual disturbances, delusions,
vivid hallucinations, agitation, insomnia, mild fever, and
marked autonomic arousal. Problems may apepar
suddently or two or three days after cessation or
redduction of heavy drinking, with a peak at the fourth or
fifth day. Symptoms may last four to five weeks, but in the
majority of patients, problems subside after three days.
About one-third who develop alcohol withdrawal seizures
(“rum fits”) go into delirium tremens. The best treatment
is to prevent withdrawal by the use of benzodiazepines and
a hig-calorie, high carbohydrate diet with supplemental
vitamins.
Source: Psychiatry specialty Board Review By William M.
Easson, MD and Nicholas L. Rock, MD
2569. Answer: E (All)
Explanation:
Barbiturate withdrawal (especially short acting) usually
results in weakness, insomnia, anxiety, tremulousness,
abdominal discomfort, nausea and vomiting. With
preexisting cardiovascular problems, there may be fatal
reactions. Seizures generally precede delirium. Symptoms
are more marked with secobarbital an dleast with
phenobarbital withdrawal (due to its long half-life).
Source: Psychiatry specialty Board Review By William M.
Easson, MD and Nicholas L. Rock, MD
ASIPP
532
Section 12 • Controlled Substance Management
2570. Answer: D (4 Only)
Explanation:
In Acute opioid overdose, the drug of choice is naloxone
HCI (Narcan), 0.4-2.0 mg, preferably IV, every 2 to 3
minutes, to a maximum dose of 10 mg. Nalocone is an
opioid antagonist that blocks opioid receptors. Other
opioid antagonists are nalorphine and levellorphane. In
an opioid withdrawal procedure, naltrexone HCI (Trexan),
clonidine, and methadone may be used as they have longer
acting effects.
Source: Psychiatry specialty Board Review By William M.
Easson, MD and Nicholas L. Rock, MD
prescribed)
Source: Art Jordan, MD, Sep 2005
2576. Answer: E (All)
Explanation:
21 U.S.C 823 (g)
Source: James D. Colson, MS, MD
2577. Answer: E (All)
Explanation:
History of Opium
* Eber’s Papyrus, 1500 B. C. - pain relief
2571. Answer: B (1 & 3)
Explanation:
A documented abnormality/pathology on MRI may be an
incidental finding and not causing pain . . . treat the
patient, not the MRI.
Just because another physician has been dispensing
narcotics to a patient, does not necessarily mean it was
appropriate or legal. The dispensing/prescribing must be
based on a legitimate medical purpose as determined by
the current prescribing physician, after careful evaluation
and examination.
Source: Art Jordan, MD, Sep 2005
2572. Answer: B (1 & 3)
Explanation:
2 and 4 are incorrect. A prescription cannot be issued in
order for an individual practitioner to obtain controlled
substances for supplying the individual practitioner for
the purpose of general dispensing to patients. Schedule II
drugs cannot be prescribed for narcotic addiction
treatment.
Reference: 21 USC 823(g); 21 CFR 1306.04
Source: Erin Brisbay McMahon, JD, Sep 2005
* Greece - mixed with wine 100 B. C.
* Galen - 100 A. D. - ‘great cure-all’
* Arabia - (600-900 A. D.) used medicinally. When the
Koran forbade alcohol, Opium and Hashish became the
primary social drugs.
Source: Roger Cicala, MD, Sep 2005
2578. Answer: A (1,2, & 3)
Explanation:
Food and Drug Act Amendments
* Durham-Humphrey 1951 - make OTCs require
prescriptions.
* Boggs Act (1951)
* Narcotic Control Act (1956)
* Drug Abuse Control Acts of 1965 _and 1968 labelled
‘potential drugs of abuse’ and gave power to DEA
precursors to regulate.
Source: Roger Cicala, MD, Sep 2005
2573. Answer: B (1 & 3)
Source: Reddy Etal. Pain Practice: Dec 2001, march 2002
2579. Answer: A (1,2, & 3)
Source: Stimmel, B
2574. Answer: A (1,2, & 3)
Explanation:
Grapefruit juice will increase methadone levels.
Source: Andrea M. Trescot, MD
2580. Answer: C (2 & 4)
Explanation:
Urine Drug Test Methods: Cocaine
2575. Answer: A (1,2, & 3)
Explanation:
Financial contracts are not included.
ASIPP
Cocaine, a topical anesthetic, is clinically used in certain
trauma, dental, ophthalmoscopic, and otolaryngologic
procedures.
In addition, the complete listing includes:
A patient’s urine may test positive for the cocaine
metabolite benzoylecgonine after such a
procedure for up to 2 to 3 days.
1: Medical history and physical examination
2: Diagnostic, therapeutic and laboratory results
3: Evaluations and consultations
4: Discussion of risks and benefits
5: Informed consent
6: Treatments
7: Medications (including date, type, dosage and quantity
There is no structural similarity between other topical
anesthetics that end in “caine” (eg,
Novocaine, lidocaine) and cocaine or benzoylecgonine;
therefore, cross-reaction does not occur.
A positive UDT result for the cocaine metabolite, in the
533
Section 12 • Controlled Substance Management
absence of a medical explanation, should
be interpreted as due to deliberate use.
Source: Laxmaiah Manchikanti, MD
2581. Answer: A (1,2, & 3)
Source: Laxmaiah Manchikanti, MD
2582. Answer: E (All)
Explanation:
Drug Testing may be performed by any of the following:
Urine Drug Screening
Specific drug analysis (blood)
Hair Samples
Saliva Testing
Serum Levels
Source: Laxmaiah Manchikanti, MD
2583. Answer: B (1 & 3)
Explanation:
Watch for signs:
Work performance alternating between periods of high
and low productivity and mistakes made due to
inattention, poor judgment and bad decisions;
Confusion, memory loss, and difficulty concentrating or
recalling details and instructions. Ordinary tasks require
greater effort and consume more time;
Interpersonal relations with colleagues, staff and patients
suffer. Rarely admits errors or accepts blame for errors or
oversights;
Heavy “wastage” of drugs;
Sloppy recordkeeping, suspect ledger entries and drug
shortages;
Inappropriate prescriptions for large narcotic doses;
Insistence on personal administration of injected narcotics
to patients;
Source: Erin Brisbay McMahon, JD, Sep 2005
2584. Answer: E (All)
Explanation:
1. Methadone is an opioid receptor agonist. It is used as an
analgesic and to treat opioid abstinence and heroin users
(methadone maintenance).
2. Methadone has greater oral efficacy than morphine and
a much longer biologic half-life;
3. Methadone produces milder but more protracted
abstinence syndrome associated with methadone.
4. Adverse reactions may include constipation, respiratory
depression, and light headedness.
2585. Answer: B (1 & 3)
Explanation:
Opiates are naturally occurring alkaloids such as
morphine. Opioids are natural or synthetic compounds
that work at the opioid receptor. All opiates are opioids,
and all opioids are narcotics. However, not all opioids are
opiates. All narcotics are opioids; examples of nonopioid
narcotics include marijuana and LSD.
Source: Andrea M. Trescot, MD
2586. Answer: C (2 & 4)
Explanation:
Cocaine withdrawal has no specific physiological signs,
but there are physical problems (“crash”) that peak in two
to four days. Depression and irritability can persist for
weeks. These patients show a desire for sleep, often with
insomnia, with disturbed sleep and increased dreaming,
general fatigue, and suicidal ideation. Drug-seekingbehavior usually occurs after bein drug-free for a few days.
Source: Psychiatry specialty Board Review By William M.
Easson, MD and Nicholas L. Rock, MD
2587. Answer: D (4 Only)
Explanation:
Amphetamines or “speed” are stimulants with reinforcing
effects similar to cocaine. Chronic amphetamine use
causes tachycardia, elevated BP, pupillary dilation,
agitation, elations, and hypervigilance. Adverse side effects
include insomnia, fever, headaches, confusion, irritability,
hostility, and visual hallucinations.
Source: Psychiatry specialty Board Review By William M.
Easson, MD and Nicholas L. Rock, MD
2588. Answer: A (1,2, & 3)
Explanation:
Some opioids, such as morphine, are metabolized by
glucuronidation, while other opioids, such as
propoxyphene are metabolized by the P450 system. M6G
is analgesic while M3G is probably produces hyperalgesia.
Reference: Sjogren P, Jensen NH, Jensen TS. Disappearance
of morphine-induced hyperalgesia after discontinuing or
substituting morphine with other opioid agonists. Pain
1994 Nov;59(2):313-6.
Source: Andrea M. Trescot, MD
2589. Answer: B (1 & 3)
Explanation:
Patient is taking 4950mg acetaminophen per day (Lortab
has 500mg per tablet = 3000mg, Fioricet has 325mg per
tablet = 1950mg). Toxic acetaminophen levels range from
3 to 4 gm per day. The equivalent dose of methadone
would be 10mg TID (hydrocodone 60mg per day =
morphine 60mg; decrease dose by 1/2 =30mg,
divide by 3 and dose q 8hr would be 10mg
q 8hrs). Paroxetine is a potent CYP2D6 inhibitor
and will inhibit the metabolism of hydrocodone
to hydromorphone, leading to decreased analgesia.
Neither escitalopram (Lexapro) or citalopram
ASIPP
534
Section 12 • Controlled Substance Management
(Celexa), which are considered “super-selective SSRIs”,
will inhibit CYP2D6. Butalbital and triptons can cause
rebound headaches, and should be changed; prophylactic
medications may be needed. Methadone gives good relief
of headaches, but butalbital will decrease methadone
levels.
Source: Andrea M. Trescot, MD
2590. Answer: A (1,2, & 3)
Explanation:
Equipotent doses of opioids are equipotent on the
respiratory system, and there is a direst effect on the
medulla. The cough suppression is a different mechanism
than the respiratory response. However, respiratory rate
will only drop after the CO2 and hypoxia responses have
decreased.
Source: Andrea M. Trescot, MD
2591. Answer: A (1,2, & 3)
2592. Answer: E (All)
Explanation:
Harrison Narcotics Tax Act of 1914:
* Tax on all opium and coca.
* Required physicians to register and keep records of
prescribed medications.
* Outlaws sale and distribution except prescribed by
physician.
* Created Bureau of Narcotics of Treasury Department
(and Federally run Heroin Clinics for addicts).
* The Bureau of Narcotics became the Prohibition Unit
after passage of the Volstead Act. After Prohibition ended
it became the Federal Narcotics Bureau and later the D. E.
A.
-does your Medical Licensure Board have guidelines on
prescribing controlled substances?
-what does DEA say about your state on its website?
http://www.dea.gov/pubs/state_factsheets.html
Source: Erin Brisbay McMahon, JD, Sep 2005
2594. Answer: D (4 only)
Explanation:
Nociceptive pain is caused by the activation of nociceptors
in the tissues, and is divided into somatic and visceral
causes. Deafferentation and automatic firing are
characteristics of neuropathic pain, and respond to alpha 2
antagonists. NSAIDs, which block prostaglandins, are
often useful in nociceptive pain.
Source: Andrea M. Trescot, MD
2595. Answer: E (All)
Source: temp
2596. Answer: A (1, 2 & 3)
2597. Answer: D (4 only)
2598. Answer: A (1,2, & 3)
Explanation:
Level of Care - Necessary to Achieve & Maintain
Abstinence
* Medically managed inpatient treatment
- Medical/surgical hospital
- Psychiatric hospital
* Medically supervised inpatient treatment
* Partial hospitalization
Harrison Act Case Law:
* Intensive outpatient program
* Three physician cases: Webb (1919), Moy (1920), and
Behrman (1921).
- Behrman: Physicians could not prescribe to ‘habitual
users.
- Webb and Moy: Must be “in the usual course of
practice” to a “legitimate patient”.
- 3,000 physician arrests during 1920s.
* Residential treatment program
Source: Kennison Roy, MD
* By 1930 “addict pattern” was male, minority, criminal.
But much smaller (?20,000).
* Payment issues
- The opioid model works well from a business
standpoint
* 1925 Linder case allowed physicians to prescribe long
term or in addicted persons - but few would.
Source: Roger Cicala, MD, Sep 2005
* System difficulties
- Hard to assemble a multidisciplinary team
2593. Answer: A (1,2, & 3)
Explanation:
AVOIDING TROUBLE
Know your state
-what laws and regulations are in place?
ASIPP
2599. Answer: B (1 & 3)
Explanation:
Barriers to Non-Opioid Management
* Issues of detoxification
- “Rebound pain” phenomenon
Source: Kennison Roy, MD
2600. Answer: D (4 Only)
Explanation:
* Outcome studies are becoming available
535
Section 12 • Controlled Substance Management
- One year post completion outcome data:
All patients still had some pain - most had much less
70% had achieved continuous sobriety
Pain was not ever worse without narcotics than it was
while taking narcotics.
Source: Kennison Roy, MD
2601. Answer: A (1,2, & 3)
Explanation:
State Board Rules You Might Not Know
Case Study: Eastern Kentucky
* Almost half a ton of prescription narcotics reached six
counties in Eastern Kentucky from 1998-2001, equating to
.75 pound for every adult in those counties.
* On a per capita basis, Eastern Kentucky drugstores,
hospitals, and legal outlets receive more prescription
painkillers than anywhere else in the United States.
The Escalating Problem: Hydrocodone
Cannot Rx Schedule II or III for family members
Can provide samples of unscheduled drugs for family, but
MUST document in a medical record
Cannot Rx for anyone in sexual relationship, EVER.
Cannot Rx for yourself, EVER.
Cannot Rx to anyone (including friends) if you have not
documented their H&P and have a current chart on file.
Source: Laxmaiah Manchikanti, MD
2602. Answer: E (All)
Explanation:
Source: Manchikanti et al., National All Schedules
Prescription Electronic Reporting Act (NASPER):
Balancing Substance Abuse and Medical Necessity, Pain
Physician 2002
GRIM NATIONAL STATISTICS
Opioid abuse increased 85% from 1994-2000
Oxycodone abuse increased 166% since 1994
Hydrocodone abuse increased 116% since 1994
OxyContin suspected in 282 overdose deaths during a 19month period
Source: Erin Brisbay McMahon, JD, Sep 2005
2603. Answer: B (1 & 3)
Explanation:
Federal Guidelines allow for use of opioids for analgesia in
persons with substance abuse disorder for “legitimate
medical reasons”.
Clear documentation of the pain problem is needed to
demonstrate the physician without proper credentials is
not attempting to detoxify an opiate abuser.
State regulations in certain states do no allow for this, and
consider prescribing opioids in known substance abusers
malprescribing.
Source: Laxmaiah Manchikanti, MD
2604. Answer: A (1,2, & 3)
Explanation:
Source: Linda Johnson, Eastern Kentucky: Painkiller
Capital, Lexington Herald-Leader, Jan.19, 2003 and Linda
Johnson, Lesser-Known Favorites Cheap, Abundant,
Lexington Herald-Leader, Jan.19, 2003.
* Nationally, emergency room visits for hydrocodone
overdoses increased 500 percent from 1990-2000
* Three Eastern Kentucky counties had enough Lortab,
Lorcet, and Vicodin pills in 2001 to provide every adult in
those counties with 156 pills
* Oxycontin sells on the street for about $40/pill; Lortabs
sell for $20/pill and Lorcets for $9/pill
Source: Erin Brisbay McMahon, JD, Sep 2005
2605. Answer: A (1,2, & 3)
Explanation:
Practitioner may administer or dispense directly, but not
prescribe, a narcotic drug in any schedule to a narcotic
dependent person for the purposes of detoxification or
maintenance treatment only if the practitioner is
registered as a narcotic treatment program and compliant
with DEA regulations21 U.S.C 823 (g)
Source: James D. Colson, MS, MD
2606. Answer: E (All)
Explanation:
U.S. enlisted soldiers addicted to opioids in RVN did not
follow the pattern of addicted civilians: on return to U.S.;
the great majority no longer used the drug (heroin).
Source: Psychiatry specialty Board Review By William M.
Easson, MD and Nicholas L. Rock, MD
2607. Answer: B (1 & 3)
Source: Renee R. Lamm, MD, Sep 2005
2608. Answer: D (4 Only)
Explanation:
Ergotamine actions are mediated by agonist actions at
both
serotonin (5HT)-receptors and ?-adrenoceptor.
Ergotamine causes vasoconstriction and is contraindicated
during pregnancy.
2609. Answer: E (All)
Explanation:
Acute dystonia, gynecomastia, sedation and loss of libido
may all be seen in patients being treated with neuroleptic
agents.
2610. Answer: E (All)
Explanation:
Biotransformation generally produces metabolites that are
more water soluble than the parent drug. The metabolites
are less lipophilic and, hence, are poorly reabsorbed in the
ASIPP
536
Section 12 • Controlled Substance Management
kidney, thus facilitating elimination.
2611. Answer: A (1,2, & 3)
Explanation:
Side Effects are Common - Opioids are often discontinued
because of adverse event:
Constipation
Nausea
Somnolence
Vomiting
Source: Mark V. Boswell, MD, KSIPP 2005
Up to 20% of prescribed opioids are diverted.
567 physician arrests and sanctions in 2002.
No more Federal diversion program for malprescribing
physicians.
State Boards may react to arrests of physicians, increasing
sanctions.
Source: Roger Cicala, MD, Sep 2005
2619. Answer: A (1,2, & 3)
Explanation:
1. Goals of pharmacotherapy include prevention or
reduction of withdrawal symptoms
2612. Answer: C (2 & 4)
Explanation:
Ref: Frogen and Avram. Chapter 15. Nonopioid
Intravenous Anesthetics. In: Clinical Anesthesia, 2nd
Edition. Barash, Cullen, Stolling; Lippencott, 1992, pg 388
Source: Day MR, Board Review 2003
2. Goals of pharmacotherapy include prevention or
reduction of drug craving
2613. Answer: B (1 & 3)
Explanation:
Risks of Malprescribing
4. Goals of pharmacotherapy also include prevention of
relapse to use of addictive drugs.
Legal charges, probably jail time
Conviction rate is currently almost 90%
Felony conviction will likely prevent or at least severely
limit future practice
Duped and Dated aren’t viable defense
Source: Laxmaiah Manchikanti, MD
2614. Answer: C (2 & 4)
Source: Reddy Etal. Pain Practice: Dec 2001, march 2002
2615. Answer: B (1 & 3)
Explanation:
What is the public need?
Patients and payers call for more accountability
- Changing patient-physician relationship
- Concern about increasing costs
Clinicians (and patients) coping with information
overload need tools for better decision making
- Synthesis of alternative diagnostic and treatment options
- Quantification of outcomes
Source: Laxmaiah Manchikanti, MD
2616. Answer: E (All)
Source: Reddy Etal. Pain Practice: Dec 2001, march 2002
2617. Answer: A (1,2, & 3)
Source: Jackson KC. Board Review 2003
2618. Answer: A (1,2, & 3)
Explanation:
ASIPP
3. Goals of pharmacotherapy include restoration to or
toward normalcy of any physiologic function disrupted by
chronic drug use.
2620. Answer: E (All)
Source: Renee R. Lamm, MD, Sep 2005
2621. Answer: A (1,2, & 3)
Explanation:
Prescriptions for pain relief are receiving special attention
by the DEA and even the OIG because of the significant
increase in the types of pain prescriptions available today,
the significant increase in the number of prescriptions for
these medications, and the evidence of doctor shopping by
persons who obtain pain prescriptions from multiple
doctors for their own abusive use or for illegal resale to
others.
Source: U.S. Drug Enforcement Administration News
Release, October 23, 2001
(http://www.usdoj.gov/dea/pubs/pressrel/pr102301.html).
Source: Erin Brisbay McMahon, JD, Sep 2005
2622. Answer: E (All)
Explanation:
The temperature should be 90-100 degrees F. within 4
minutes of voiding. The pH should remain with the range
of 4.5 to 8.0. The urinary creatinine should be greater than
20 mg/dl; less than 20 mg/dl is considered dilute and less
than 5 mg/dl is not consistent with human urine. Color
may be a result of substances from food pigments,
medications, or disease states.
Ideally, the collection room should not contain a basin
with running water, to reduce potential for specimen
dilution, and blue pigment should be added to the toilet
water.
537
Section 12 • Controlled Substance Management
Reference: Cook, Caplan et al; The characterization of
human urine for specimen validity determination in
workplace drug testing: a review. J Anal Toxicol.
2000;24:579-588.
Urine Drug Testing in Clinical Practice: Pearls & Pitfalls:
Purdue Pharma, 2005 (available free on request)
Source: Art Jordan, MD, Sep 2005
2623. Answer: A (1,2, & 3)
2624. Answer: D (4 Only)
Explanation:
It is important that the physician know that there are
many available devices to avoid detection of improper or
illegal substances in the urine, including the
“Whizzinator”; an artificial penis which contains urine
“guaranteed” to be drug free.
Source: Art Jordan, MD, Sep 2005
2625. Answer: E (All)
Explanation:
It is advisable that an allopathic physician stay true to
scientific and well-validated approaches when treating
pain. Many times the pain management physician is a
referral of desperation, and expectations of the patient
may be unrealistically high. Alternative therapies such as
naturopathic medicine may be trialed, in conjunction with
well-established treatment to enhance positive outcome.
The patient should understand from the beginning the
treatment style, policy and procedures of the clinic, and
expectations, particularly if controlled substances are
being used.
Source: Hans C. Hansen, MD
2626. Answer: A (1,2, & 3)
Source: Hansen HC, Board Review 2004
2627. Answer: A (1,2, & 3)
Explanation:
The Federation of State Medical Boards requires specific
documentation in the medical record to define legitimate
medical need when controlled substances are used. This
includes: medical history, substance or chemical abuse,
pain history, appropriate studies, working diagnosis,
treatment plan, rationale for treatment selected, patient
education, and documentation that the patient and
physician understand the treatment goals. The standard
medical history course will cover allergies to medications,
but not necessarily relevant to the concept of legitimate
medical need. For appropriate prescription habitry to be
realized, diagnosis, historical features and exam must meet
the need for the medication, clearly outlined and
understood in the medical record.
Source: Hans C. Hansen, MD
2628. Answer: A (1,2, & 3)
Explanation:
Methadone, is not included in the federal five.
The correct federal five are:
1: Marijuana (delta-9-THC acid)
2: Cocaine (benzoylecgonine)
3: Opiates
4: Phencyclidine
5: Amphetamine/methamphetamine
Source: Art Jordan, MD, Sep 2005
2629. Answer: B (1 & 3)
Explanation:
The state of hydration on an isolated sample would be of
no use in determining dosage. Likewise, the half life of the
medication could not be determined from an isolated
urine sample, while only checking for drugs present
It is most important that the ordering physician know
exactly which drugs are included in the specific test, and
which drugs may not be detected. Examples include
oxycodone, fentanyl, and methadone which may not be
detected in many basic urine drug screens on the market.
Source: Art Jordan, MD, Sep 2005
2630. Answer: C (2 & 4)
Explanation:
Reference: “Interim Policy Statement” November 2004
Source: Art Jordan, MD, Sep 2005
2631. Answer: C (2 & 4)
Explanation:
“1” was commonly referred to as alternate dating prior to
the F.A.Q.’s and was generally accepted by the D.E.A. as a
legal but not recommended activity. The F.A.Q.’s actually
stated that this action was acceptable, however this was
specifically addressed as illegal in the “Interim Policy”
statement in November of 2004.
Source: Art Jordan, MD, Sep 2005
2632. Answer: E (All)
Explanation:
Explanation:
All of the above answers are listed in the interim policy.
In addition, the following are also listed as “certain
recurring concomitance of condemned behavior”:
1:An inordinately large quantity of controlled substances
was prescribed
2: Large numbers of prescriptions were written
3: No physical examination was given
4: The physician prescribed controlled drugs at intervals
inconsistent with legitimate medical treatment
5: The physician wrote more than one prescription on
occasions in order to spread them out
**You will frequently see some of these behaviors listed as
ASIPP
538
Section 12 • Controlled Substance Management
“Red Flags”
Source: Art Jordan, MD, Sep 2005
2633. Answer: C (2 & 4)
Source: Stimmel, B
2634. Answer: C (2 & 4)
Explanation:
Reference: Kalso E, Edwards JE, Moore A, McQuay JH.
Opioids in chronic non-cancer pain: systematic review of
efficacy and safety. Pain 2004; 112;372-380.
2639. Answer: B (1 & 3)
Source: Laxmaiah Manchikanti, MD
2640. Answer: A (1,2, & 3)
Explanation:
Addiction: Definitions
DSM-IV - None
A primary, chronic neurobiologic disease with
genetic,psychosocial and environmental factors
effecting its course and presentation
Morphine and oxycodone equally effective
Morphine 20mg/day and oxycodone 30 mg/day not
effective
All patients in studies were on opioids previously
Opioids did not improve depression
Improved functional status unclear
Source: Mark V. Boswell, MD, KSIPP 2005
2635. Answer: B (1 & 3)
Explanation:
Drug Dependent (Addict)
Uses another doctor’s DEA number
2641. Answer: D (4 Only)
Explanation:
Morphine is used to relieve the pain associated with
myocardial infarction. It can suppress the opioid
withdrawal syndrome but not the withdrawal syndrome
associated with other classes of central nervous system
depressants. Morphine and other opioids induce
constipation and can be used to treat diarrhea. Miosis is
an adverse effect of morphine.
Calls in scripts in names of family members or fictitious
patients and picks them up himself
Source: Laxmaiah Manchikanti, MD
2642. Answer: E (All)
Explanation:
SUBSTANCE DEPENDENCE: DSM-IV DEFINITION
Starts by taking controlled drug samples
Asks staff to pick up medications in their names
2636. Answer: A (1,2, & 3)
Explanation:
NEED SLIDE 13
Source: Laxmaiah Manchikanti, MD
2637. Answer: A (1,2, & 3)
Explanation:
Phase II reactions typically yield water-soluble
metabolites. Active centers are introduced during phase I,
not phase II, biotransformation. Glucuronyl transferase is
the only phase II enzyme inducible by drug
administration.
Prodrugs are often activated by phase I, not phase II,
reactions; phase II reactions generally terminate drug
action.
2638. Answer: E (All)
Explanation:
All of the choices are possible consequences of drug
administration. Many enzymes involved in drug
biotransformation also catalyze the metabolism of
endogenous compounds such as steroids.
ASIPP
Characterized by one or more of the following
Impaired control of drug use
Compulsive use
Craving
Continued use despite harm4-Cs
Loss of Control
Craving
Compulsive Use
Continued use despite consequences
Source: Laxmaiah Manchikanti, MD
Need 3 in 12 months
Tolerance
Withdrawal
Larger Amounts/Longer periods
Efforts or desire to cut down
Large Amount of time using/obtaining/recovering
Activities given up: social/work/recreation
Continued use despite problems
Source: Laxmaiah Manchikanti, MD
2643. Answer: A (1,2, & 3)
Explanation:
Clinical Implications:
* Difficult to distinguish pharmacologic tolerance from
opioid-induced abnormal pain sensitivity
* Treating increasing pain with increasing doses may be
futile
* High dose therapy may have adverse consequences
Source: Mark V. Boswell, MD, KSIPP 2005
539
Section 12 • Controlled Substance Management
2644. Answer: B (1 & 3)
Explanation:
Prolonged, High-Dose Opioid Therapy:
No evidence to support high dose therapy (> 200 to 300
mg/day or more)
Anecdotal evidence that pain relief not better
Opioid doses should be limited to improve efficacy and
safety
Source: Mark V. Boswell, MD, KSIPP 2005
2645. Answer: A (1,2, & 3)
Explanation:
Physician Opioid of Choice
1990s
Hydrocodone 70%
Fentanyl class 10%
Dilaudid < 5%
Oxycodone < 5%
Codeine < 5%
2002
Ultram 70%
Hydrocodone 40%
Oxycodone 25%
Fentanyl class < 5%
Dilaudid < 5%
Source: Roger Cicala, MD, Sep 2005
2646. Answer: C (2 & 4)
Explanation:
Opioid toxicity or overdose should be suspected in any
undiagnosed coma patient or patients with respiratory
depression (pulmonary edema), shock (hypothermia),
pupillary construction, and needle marks. Grand mal
seizures can occur with meperidine overdose.
Source: Psychiatry specialty Board Review By William M.
Easson, MD and Nicholas L. Rock, MD
2647. Answer: C (2 & 4)
Explanation:
Physician Opioid of Choice
1990s
Hydrocodone 70%
Fentanyl class 10%
Dilaudid < 5%
Oxycodone < 5%
Codeine < 5%
2002
Ultram 70%
Hydrocodone 40%
Oxycodone 25%
Fentanyl class < 5%
Dilaudid < 5%
Source: Roger Cicala, MD, Sep 2005
2648. Answer: D (4 Only)
Explanation:
Monoamine oxidase (MAO) inhibitors such as
tranylcypromine may precipitate a hypertensive crisis
when used in the presence of certain foods that contain
tyramine, or in the presence of certain sympathomimetic
agents. Note also that opioids, particularly meperidine,
may also (although rarely) precipitate a hypertensive crisis
when used with MAO inhibitors.
2649. Answer: D (4 Only)
Explanation:
Tricyclic antidepressants can precipitate narrow-angle
glaucoma through their muscarinic-cholinoreceptor
antagonist activity. They may cause hypotension and may
block neuronal uptake of guanethidine and thus decrease
its antihypertensive action. They may also decrease the
gastrointestinal absorption of levodopa.
2650. Answer: E (All)
Explanation:
Short-acting benzodiazepines are used as preanesthetic
medications because of their anxiolytic, sedative, and
amnestic effects. Daytime drowsiness and ataxia are
commonly produced by benzodiazepines and may impair
judgment and interfere with motor skills. Paradoxical
excitement is a rare adverse effect of these drugs.
2651. Answer: E (All)
Explanation:
Morphine produced all of the above effects. It decreases
the sensitivity of the respiratory center to carbon dioxide
and directly inhibits the respiratory center, leading to
respiratory depression. This effect on respiration may be
responsible for morphine’s beneficial effect when used to
treat dyspnea and for cerebral vasodilation. Morphine
produced miosis by stimulation or the Edinger-Westphal
nucleus of the oculomotor nerve; this is mediated by
acetylcholine and can be blocked by atropine.
2652. Answer: A (1,2, & 3)
Explanation:
Carbamazepine is an inducer of 2D6 and will reduce
oxycodone levels.
Source: Boswell MV, Board Review 2004
2653. Answer: E (All)
Explanation:
SUBSTANCE ABUSE: DSM-IV DEFINITION
At least one in 12 months:
Maladaptive pattern leading to distress or impairment
Recurrent failure to fill role
Recurrent physically hazardous behavior
Recurrent legal problems
Continued use despite social problems
Never met dependence criteria
Source: Laxmaiah Manchikanti, MD
ASIPP
540
Section 12 • Controlled Substance Management
2654. Answer: E (All)
2655. Answer: A (1,2, & 3)
Explanation:
Meperidine is contraindicated for lactation because
normeperidine collects in the neonate
Source: Boswell MV, Board Review 2005
2656. Answer: A (1,2, & 3)
Explanation:
Open Label Studies
44% on opioids at 2 years
80% of patients experienced at least one adverse event
No firm conclusions about tolerance and addiction
4. Poor documentation
5. No policies – No agreements
6. Ignore complaints
7. Focus on negative aspects of regulations and
reimbursement
8. Not nice to investigators from Board, DEA!!
9. Reckless disregard to law with prescription pads and
regulations
10. Know it all – Do it all
Source: Laxmaiah Manchikanti, MD
2660. Answer: D (4 Only)
Explanation:
Barbiturates induce liver microsomal enzymes that
increase porphyrin synthesis and increase the metabolism
and inactivation of certain anticoagulants. Barbiturates
show cross-dependence with other sedative-hypnotic
drugs but not with opioids.
Patients in trials were highly selected
Results not generally applicable
5% of 1000 screened patients included
Source: Mark V. Boswell, MD, KSIPP 2005
2657. Answer: E (All)
Explanation:
Deficient (Dated Practitioner)
Too busy to keep up with CME
Unaware of controlled drug categories
Only aware of a few treatments or medications
Prescribes for friends or family without a patient record
2661. Answer: A (1,2, & 3)
Explanation:
Risks of Malprescribing
Loss of “Provider Status”
Insurors frequently report to Boards Insurors frequently
report to Boards now
Several plans have removed providers for
“overprescribing.” Seems more common with more
expensive agents (Duh!).
Insurors can report to a separate national data bank, not
available to public, but available to hospitals and other
insurors.
Source: Laxmaiah Manchikanti, MD
Unaware of symptoms of addiction
Remains isolated with peers
Only education from reps
Source: Laxmaiah Manchikanti, MD
2658. Answer: D (4 Only)
Explanation:
2002 and 2003 saw a significant increase in non-medical
use of pain relievers. This seems to be a trend over the
past decade. Hydrocodone leads at 15 million, followed by
oxycodone (Oxycontin? specifically mentioned as a subset
of oxycodone), methadone, and tramadol.
Source: Hans C. Hansen, MD
2659. Answer: A (1,2, & 3)
Explanation:
Top 10 Pitfalls
1. 4 D’s – Deficient, Duped, Deliberate, Dependent
Practioner
2. Weak heart – pretend addiction doesn’t exist
3. Never say “NO” – Family, Friends, Patients
ASIPP
2662. Answer: E (All)
Explanation:
Symptoms of withdrawal include:
Opiate craving
Rhinorrhea
Anxiety/Dysphoria
Sweating
Dilated pupils
Piloerection
Restlessness
Yawning
Muscle twitching
Nervousness
Increased respiration
Headache
Fatigue
Irritability
Anorexia
Fever
Cutaneous hypersensitivity
Insomnia
Tachycardia
541
Section 12 • Controlled Substance Management
Hypertension
Hot/cold flashes
Nausea
Vomiting
Muscle aches/spasms
Abdominal cramps
Bone pain
Diarrhea
2663. Answer: B (1 & 3)
Explanation:
Physician Drugs of Abuse 2001
Alcohol 50% to 60%
Opioids 30%
Benzodiazepines 20% (40% for females)
Marijuana 20%
Cocaine 10%
Amphetamines, Ritalin 10%
Source: Roger Cicala, MD, Sep 2005
2664. Answer: D (4 Only)
Explanation:
Benzotropine (Cogentin) has atropine-like side effects;
dilated pupils, dry mouth, urinary retention, restlessness,
confusion and toxic psychosis.1. Phenelzine, a MAOI,
reacts with tyramine-containing substances causing a
“cheese reaction,” which consists of sweating, palpitations,
headache, and increased blood pressure resulting in a
possible intracerebral hemorrhage.2. Disulfiram
(Antabuse), if taken with alcohol, cuases flushing,
throbbing, sweating, thirst, respiratory difficulty, nausea,
vomiting, tachycardia, hypotension, vertigo, blurred
vision, and confusion.3.Alprazolam, a benzodiazepine,
causes sedation, impairment of performance, and
dependency.
Source: Psychiatry specialty Board Review By William M.
Easson, MD and Nicholas L. Rock, MD
2665. Answer: E (All)
Explanation:
Use in Europe and America
* Opium knowledge was lost during Dark Ages,
reintroduced after Crusades.
* Paracelsus 1500 called laudanum (opium, cloves, and
alcohol) the “stone of immortality”.
* Thomas Sydenham brought to England about 1700.
* 1831 Sertuener given Nobel Prize for isolating
individual opiates.
* 1853 - hypodermic syringe invented. The American Civil
War and Prussian Wars of 1860-1870 led to widespread
morphine injection.
Source: Roger Cicala, MD, Sep 2005
2666. Answer: B (1 & 3)
Explanation:
Opiate Abuse
* “Soldier’s Disease” - morphine addiction - reported
during American Civil War and Prussian Wars of 18601870.
* Widespread laudanum and opiate abuse in U. S. in late
1800s. Sears carried syringe kits in catalogs. Estimated
500,000 - 1,000,000 addicted in U. S.
* 4% of U. S. population used laudanum nonmedicinally.
* Laudanum addiction was considered fashionable.
Typically described as ‘middle-aged, upper-class white
female’. Samuel Taylor Coleridge, Elizabeth Barrett
Browning, Thomas De Quincy all quite open about their
addiction.
* Heroin introduced in 1898 as “non-addicting”
morphine. Used as cure for “Morphinism” similar to
methadone today.
Source: Roger Cicala, MD, Sep 2005
2667. Answer: A (1, 2, & 3)
Explanation:
Factors determinative of control or removal from
schedules by the attorney general are as follows:
1. Its actual or relative potential for abuse.
2. Scientific evidence for its pharmacological effect, if
known.
The state of current scientific knowledge regarding the
drug or other substance.
3. Potential and current pattern of abuse
The scope, duration, and significance of abuse.
What, if any, risk there is to public health.
Its physic or physiological dependence liability.
Whether the substance is an immediate precursor of a
substance already controlled under this title.
4. Price has no impact.
2668. Answer: B (1 & 3)
Explanation:
Tolerance occurs when the same dosage of drug has a
reduced effect and increased amounts of the drug are
needed to achieve the desired effect. Physical dependence,
the need to take the drug to prevent withdrawal, tends to
develop in parallel with tolerance.
Source: Psychiatry specialty Board Review By William M.
Easson, MD and Nicholas L. Rock, MD
2669. Answer: C (2 & 4)
Explanation:
Heroin abusers tend to start in late teens and early 20s
(most common 18-25 years old), with the majority in the
ASIPP
542
Section 12 • Controlled Substance Management
mid-30s. There is a 3:1 male to female ratio. Suicide in
abusers is three times greater than in the general
population. They also have a 20 times greater death rate,
as well as higher rates of hepatitis B and HIV III viral
infections.
Source: Psychiatry specialty Board Review By William M.
Easson, MD and Nicholas L. Rock, MD
2670. Answer: E (All)
Explanation:
Heroin withdrawal symptoms are similar to a influenzalike syndrome along with anxiety and dysphoria. Physical
symptoms include yawning, sweating, rhinorrhea,
lacrimation, pupillary dilation, piloerection, hypertension,
waves of gooseflesh, twitching movements, deep muscle
and joint pains, nausea, diarrhea, vomiting, abdominal
pains, fever, and hot and cold flashes.
Source: Psychiatry specialty Board Review By William M.
Easson, MD and Nicholas L. Rock, MD
2671. Answer: A (1, 2, & 3)
Explanation:
Therapeutic communities for substance abusers have as
their goals a complete change of lifestyle and abstinence
from drugs. If the patient’s stay is more than 90 days, there
is a long-term decrease in illicit drugs use, antisocial
behavior, and arrests, and increased employment. With a
12-month stay, subjects fare even better at five years postprogram follow-up.
Source: Psychiatry specialty Board Review By William M.
Easson, MD and Nicholas L. Rock, MD
2672. Answer: B (1 & 3)
Explanation:
Withdrawal convulsions can occur with alcohol, certain
benzodiazepines, and barbiturates.
Source: Psychiatry specialty Board Review By William M.
Easson, MD and Nicholas L. Rock, MD
2673. Answer: B (1 & 3)
Explanation:
1. Neuropsychologic testing can document the
physician is mentally acute while on chronic opioid
therapy.
2. Without adequate documentation of the physician
patient’s mental ability and the requirements of the
hospital, the pain treatment physician could place him or
herself at risk by making such a statement.
3. The pain treatment physician cannot directly inform
the interested parties of the patient’s medications, but can
cease treating the patient if he or she feels the situation is
not acceptable.
4. There is no law in most states that prevents practicing
medicine while taking opioids.
Source: Roger Cicala, MD, Sep 2005
2674. Answer: B (1 & 3)
Explanation:
Those who either stop or control drug use after a period of
addiction are more likely to be older than age 40, have a
ASIPP
normal premorbid personality, never be arrested for
substance abuse, and to undergo substance abuse
treatment / rehabilitation. Developing medical
complications from abuse rarely leads to abstinence.
Source: Roger Cicala, MD, Sep 2005
2675. Answer: E (All)
Explanation:
Simple Chronic Pain
Pain lasting longer than 6 months.
Tend to have no more distress or psychopathology than
what is expected in the general population.
Tend to continue working.
Tend not to become overly reliant on medications, i.e.,
have various ways to self-manage pain.
Tend to maintain meaningful relationships
Tend to maintain a sense of meaning and direction to their
lives.
Source: Murray McAllister, PsyD, LP - Spring 2004
2676. Answer: A (1,2, & 3)
Explanation:
1. 10% of prescription drugs are used for non-medical
purposes.
2. Over 6 million people over the age of 12 years in the
United States have been reported to use controlled
prescription drugs for non-medical purposes.
3. Opioid abuse is seen in as high as 18% to 24% of the
patients in chronic pain. Illicit drug use is seen in 14% to
32% of chronic pain patients.
4. Marijuana is the most commonly used illicit drug,
followed by cocaine.
Source: Laxmaiah Manchikanti, MD
2677. Answer: B (1 & 3)
Explanation:
All of the above are symptoms of opiate withdrawal along
with numerous others. Fever, Hypertension, and
Tachycardia are considered the most consistent symptoms
indicated severe withdrawal reaction.
Source: Roger Cicala, MD, Sep 2005
2678. Answer: B (1 & 3)
Explanation:
The patient’s total dose of hydrocodone is 60mg per day,
which is equal to 60mg oral morphine per day. This
converts to 20mg IV morphine per day (30mg morphine =
10mg IV), which is divided by 24 to get the hourly dose of
1mg per hour. This is equivalent to 3mg per day of
hydromorphone (10mg IV morphine = 1.5mg IV
hydromorphone so 20mg IV morphine = 3mg IV
hydromorphone), which, divided into 6 doses (q4hrs) =
0.5mg per dose. Meperidine is not appropriate, and
fentanyl is too slow an onset for the initial conversion.
Source: Andrea M. Trescot, MD
543
Section 12 • Controlled Substance Management
2679. Answer: B (1 & 3)
Explanation:
Alcohol is a significant problem, particularly when
controlled substances are being used, and those that have
significant risks associated with them, particularly
pharmacokinetically long acting drugs. An example that
underscores this issue is that the FDA recently removed
Palladone?, a pharmacokinetically long acting
hydromorphone preparation from the market. Palladone?
has been considered for pain control for a number of
years, recently introduced, and to be found an
unacceptably high risk when mixing alcohol and
this drug. 50% of the American population
consider themselves alcohol consumers, and
22% of those participated in a binge drinking
episode one month prior to the survey. 16.1
million are heavy drinkers, and at significant risk.
Source: Hans C. Hansen, MD
2680. Answer: A (1,2, & 3)
Explanation:
Personal and family history of substance abuse, personal
and family history of psychiatric illness, male gender, age
under 40, and nonmarried status are all associated with
increased prevalence of substance abuse.
Source: Roger Cicala, MD, Sep 2005
2681. Answer: E (All)
Explanation:
The above are the 4 listed characteristics in the Consensus
Statement.
Source: Roger Cicala, MD, Sep 2005
2682. Answer: A (1,2, & 3)
Explanation:
U. S. Opiate Abuse Patterns
* Change from morphine to heroin in 50s-60s. Dramatic
increase in numbers 60s-70s.
* Reduction in opiate abuse during 80s and early 90s
(?cocaine effect?)
* Prescription opiate abuse increases in late 1990s (?or
increased awareness?)
* Heroin resurgence begins late 1990s.
Source: Roger Cicala, MD, Sep 2005
* Coca products began widespread use in 1880s in patent
medicines and ‘soft drinks’. Endorsed by the Surgeon
General in 1886.
* Amphetamines 1920s, used by military, physicians,
widespread public use in 30s.
* Barbiturates and amphetamines began widespread use
in 1940s, originally as OTC medications and in patent
medications.
* Marijuana use reported in 1880s. Became popular
during prohibition and through the great depression.
* Hallucinogens popularized in 1960s. Declined by late
1970s.
Source: Roger Cicala, MD, Sep 2005
2686. Answer: A (1,2, & 3)
Source: Stimmel, B
2687. Answer: E (All)
Source: Stimmel, B
2688. Answer: A (1,2, & 3)
Source: Stimmel, B
2689. Answer: E (All)
Explanation:
Prescription drug abuse, particularly of opioid pain
killers, has increased at an alarming rate over the last ten
years.
Non-medical use of narcotic pain relievers, tranquilizers,
stimulants, and sedatives ranks second (behind marijuana)
as a category of illicit drug abuse among adults and youth.
In 2003, 6.3 million Americans were current abusers of
prescription drugs, with 4.7 million using pain relievers.
100,000 E.R. visits for Benzodiazepine abuse in 2002
119,000 E.R. visits for 0pioid abuse in 2002
SAMSHA, 2003 Survey
DAWN, 2004 Report
Source: Laxmaiah Manchikanti, MD
2690. Answer: A (1, 2, & 3)
2683. Answer: E (All)
Source: Stimmel, B
2684. Answer: E (All)
Source: Stimmel, B
2685. Answer: E (All)
Explanation:
Other Drugs of Abuse
2691. Answer: B (1 & 3)
Explanation:
Seizure can occur with stimulant overdose or
benzodiazepine withdrawal. It is not a reported part of the
syndrome of opiate withdrawal or of cocaine withdrawal.
Given the positive drug screen for benzodiazepines and
cocaine, either scenario is possible.
Source: Roger Cicala, MD, Sep 2005
ASIPP
544
Section 12 • Controlled Substance Management
2692. Answer: D (4 Only)
Explanation:
Answer (a) is wrong because the court cannot grant
probation, parole, or a suspended sentence if death or
serious bodily injury results with respect to a S I or II drug.
Reference: 21 USC 841.
Source: Erin Brisbay McMahon, JD, Sep 2005
2693. Answer: A (1,2, & 3)
Explanation:
(Stoelting, Anesthesia and Co-Existing Disease, 3/e, pp
528-529.)
Acute cocaine overdose will increase in central
catecholamine levels are increased. Cocaine inhibits the
reuptake of norepinephrine. Increased circulating
norepinephrine levels have numerous effects on the
cardiac system, including coronary artery vasospasm, an
increase in myocardial oxygen consumption, and an
increase in systemic vascular resistance. These effects can
cause high-output cardiac failure and cardiac ischemia.
Source: Curry S.
2694. Answer: E (All)
Explanation:
The active ingredient in marijuana is Delta-9tetrahydrocannabinol.
In general, marijuana is a CNS stimulant causing
tachycardia, giddiness, and , at high doses, visual
hallucinations.
1. Potential therapeutic uses include antiemesis in cancer
chemotherapy and reduction of intraocular pressure in
glaucoma.
2. Acute intoxication is characterized by reddening of the
conjunctiva (bloodshot eyes) owing to local vasodilation.
3. Potential therapeutic uses include antiemesis in cancer
chemotherapy and reduction of intraocular pressure in
glaucoma.
4. Chronic use has been associated with an amotivational
syndrome and with a reduction in serum testosterone and
sperm count.
2695. Answer: B (1 & 3)
Explanation:
Chronic alcoholism is associated with retrobulbar optic
neuropathy, cerebellar anterior lobe degeneration,
encephalopathy (Wernicke’s), subdural hematoma,
amnestic disorder (Korsakoff ’s syndrome), dementia,
peripheral neuropathy, pancreatitis, esophageal varices,
duodenal ulcer, cardiomyopathy, pulmonary infections
(especially tuberculosis), cirrhosis, and fetal alcohol
syndrome.
Source: Psychiatry specialty Board Review By William M.
Easson, MD and Nicholas L. Rock, MD
2696. Answer: E (All)
ASIPP
Explanation:
Although equipotent charts may vary, in general, 30mg of
oral MSO4 is equivalent to 10mg MSO4 IV, 20mg of oral
oxycodone, 1.5mg of IV hydromorphone, or 20mg of
methadone.
Source: Trescot A, Board Review 2003
2697. Answer: E (All)
Source: Reddy Etal. Pain Practice: Dec 2001, march 2002
2698. Answer: C (2 & 4)
Explanation:
* The attorney general may suspend or revoke a
registration to manufacture, distribute, or dispense a
controlled substance upon finding that the registrant:
- Has materially falsified any application filed.
- Has been convicted of a felony.
- Has had his state license or registration suspended,
revoked, or denied.
* Has committed such acts as wound render his
registration inconsistent with the public interest.
* Has been excluded or directed to be excluded from
participation in a program pursuant to Section 1128 (A)
of the Social Security Act.
2699. Answer: C (2 & 4)
Explanation:
Answer (1) is wrong because no Schedule II prescription
can be refilled. Answer (3) is wrong because there is no
exception for dispensing drugs to another physician.
Reference: 21 USC 829(a).
Source: Erin Brisbay McMahon, JD, Sep 2005
2700. Answer: A (1,2, & 3)
Explanation:
Answer (4) is wrong as the physician does not need to
determine that the patient is complaining of extreme and
unremitting pain before finding that an emergency
situation exists.
Reference: 21 CFR 290.10, 1306.11(d).
Source: Erin Brisbay McMahon, JD, Sep 2005
2701. Answer: B (1 & 3)
Explanation:
Interviews with family members may be helpful under
certain circumstances if clinically indicated and if within
legal bounds of privacy, however, this is not listed by the
Board.
Periodic reports from law enforcement, without patient
consent, would be inappropriate and possibly a violation
of privacy laws.
Source: Art Jordan, MD, Sep 2005
545
Section 12 • Controlled Substance Management
2702. Answer: A (1,2, & 3)
Explanation:
Answer (4) is wrong because a fax will not serve as the
original prescription for a Schedule II controlled substance
for any terminally ill patient.
Reference: 21 CFR 1306.11.
Source: Erin Brisbay McMahon, JD, Sep 2005
2703. Answer: B (1 & 3)
Explanation:
Answers (2) and (4) are wrong because both the
prescribing physician and the pharmacist have the
responsibility to make sure the controlled substance is for
a terminally ill patient when the partial filling is for a
terminally ill patient.
Reference: 21 CFR 1306.13.
Source: Erin Brisbay McMahon, JD, Sep 2005
2704. Answer: A (1,2, & 3)
Explanation:
Answer (1) is wrong; that is not a requirement for a partial
filling of a prescription for a Schedule III, IV, or V
substance.
Reference: 21 CFR 1306.23.
Source: Erin Brisbay McMahon, JD, Sep 2005
2705. Answer: D (4 Only)
Explanation:
Cocaine blocks neuronal dopamine, serotonin, and
norepinephrine reuptake. With prolonged cocaine use and
abuse, a delusional psychosis similar to paranoid
schizophrenia may develop.
Source: Psychiatry specialty Board Review By William M.
Easson, MD and Nicholas L. Rock, MD
2706. Answer: E (All)
Source: Laxmaiah Manchikanti, MD
2707. Answer: A (1,2, & 3)
Explanation:
Alcohol withdrawal occurs when there is a relative drop in
blood alcohol levels; therefore, it can develop while still
drinking. The patients are likely to show a coarse, fastfrequency generalized tremor that is made worse by motor
activity or stress and is easily observed when the hands or
tongue are extended. Withdrawal is manifested by
autonomic hyperactivity (increased BP, tachycardia,
sweating), malaise, vomiting with anxiety, depression,
irritability, cognitive changes, and possible seizures.
Source: Psychiatry specialty Board Review By William M.
Easson, MD and Nicholas L. Rock, MD
2708. Answer: E (All)
Explanation:
All of the above are required services for an OTP.
Refernece: 42 CFR 8.12.
Source: Erin Brisbay McMahon, JD, Sep 2005
2709. Answer: A (1,2, & 3)
Source: Renee R. Lamm, MD, Sep 2005
2710. Answer: E (All)
Explanation:
Reference: The Federation of State Medical Boards’ Model
Policy for the Use of Controlled Substances for the
Treatment of Pain (May 2004); www.fsmb.org.
Explanation:
The correct answer is E, all of the above, because each of
these items plays into the minimum standards related to
the taking of a patient history and the performance of a
physical examination prior to prescribing controlled
substances for the treatment of pain. A provider should not
omit any of these questions from his/her interaction with
the patient. Many of these questions can be set forth in a
general history form. Providers may also develop special
forms on substance abuse issues to use with patients prior
to prescribing them controlled substances. In all cases,
providers should cover these areas and more with their
patients. Consult your state materials on the use of
controlled substances for the treatment of pain.
Source: Jennifer Bolen, JD, Sep 2005
2711. Answer: A (1,2, & 3)
Explanation:
Reference: Collins and Streltzer 2003, Am J. Addict 12:2,
Covington an dKoltz, 2003 Prin of Addict Med
Source: Renee R. Lamm, MD, Sep 2005
2712. Answer: E (All)
Explanation:
Cimetidine, and Ciprofloxin are CYP3A4 inhibitors, and
will increase methadone levels. Ciprofloxin can inhibit
CYP3A4 by up to 65%. Phenytoin and butalbital will
induce the CYP3A4 enzyme and decrease blood levels.
Source: Andrea M. Trescot, MD
2713. Answer: D (4 Only)
Explanation:
The most likely problem is that the paroxetine (Paxil®) (a
potent CYP2D6 inhibitor) is preventing the metabolism
of tramadol to the active M1 metabolite. He is therefore
not drug seeking or withdrawing from illicit drugs, but
instead has had his previously working opioid made
ineffective by the drug interaction.
Source: Andrea M. Trescot, MD
2714. Answer: E (All)
2715. Answer: E (All)
Explanation:
History of Opium
* Arabic traders spread use and cultivation to China by
900 A. D.
* 1644 China outlawed tobacco, opium smoking became
endemic
ASIPP
546
Section 12 • Controlled Substance Management
* 1700 British East India company smuggled opium from
India to China for tea (China refused legitimate trade).
* 1839 - Because of “rampant addiction” Chinese emperor
destroyed 45,000 pounds of British opium in Canton,
beginning the Opium war.
* Britain won the ensuing war, receiving Hong Kong and
forcing China to accept opium in trade.
* Opium finally outlawed in China by 1913.
Source: Roger Cicala, MD, Sep 2005
2716. Answer: E (All)
2717. Answer: C (2 & 4)
Explanation:
Itching from opioids (usually the naturally occurring such
as codeine and morphine) is usually not an
antibody/antigen reaction but rather a direct histamine
release from the mast cells, as well as a central µ receptor
stimulation. Changing to a synthetic opioid such as
hydromorphone will usually resolve the problem. In the
face of a “true allergy”, there is usually no cross reactivity
across classes.
Source: Andrea M. Trescot, MD
2718. Answer: E (All)
Explanation:
Different opioids have different receptor affinity, so that
switching from a low affinity opioid like hydrocodone to a
high affinity opioid like fentanyl may allow for better
analgesia at a lower than equipotent dose. Different
metabolic pathways may explain why hydrocodone (a prodrug metabolized by CYP2D6) might not be effective
when propoxyphene is, especially in a patient who is
CYP2D6 deficient. High doses of morphine can lead to
accumulation of M3G which is hyperalgesic; switching to
an opioid without this type of antagonist would give better
analgesia. Inhibition of one enzyme system (such as 3A4
and methadone) would have no effect on an opioid
metabolized by another system (such as 2D6 and
hydrocodone).
Source: Andrea M. Trescot, MD
ASIPP
2719. Answer: E (All)
Explanation:
Opioid receptors, concentrated in the ventral tegmental
and periaqueductal grey areas, presynaptically inhibit the
transmission of excitatory pathways: acetylcholine,
catecholamine, serotonin, and substance P. Activation of
the opioid receptor inhibits adenylate cyclase. All opioid
receptors are G protein-linked structures embedded in the
plasma membrane of neurons; activation releases a
portion of the G protein, which moves in the membrane
until it reaches its target (either an enzyme or an ion
channel). These targets alter protein phosphorylation and/
or gene transcription. Opioids and endogenous opioids
activate presynaptic receptors on GABA neurons, which
inhibit the release of GABA in the ventral tegmental area.
This allows dopaminergic neurons to fire more vigorously
, and the extra dopamine in the nucleus accumbens is
intensely pleasurable.
Source: Andrea M. Trescot, MD
2720. Answer: A (1,2, & 3)
Explanation:
Urine drug testing may be helpful, however is not required
by law nor in the usual course of professional practice.
Source: Art Jordan, MD, Sep 2005